September Questions (

You might also like

Download as pdf or txt
Download as pdf or txt
You are on page 1of 232

September 2020 SLLE Questions

IHC for adenocarcinoma taken from a breast tissue -1

Carcinoemberyonic ag
Epithelial tissue ag

Tissue stayed in formalin for more than 80 hours and later on FISH was tested and -2
it was negative when the last test was positive. What is the reason

Beca e i a dela ed ample


Enzymes digestion at higher rate
Different probes were used

What would affect the patient if the Laboratory had staff shortage -3
no calibration will be done
exhaustion and less focus
Delayed samples

What preventable error can cause fatalities -4


misidentification of the patient

What is considered a violation of patient rights -5


recording or taking photos of the patient
Treating the patient
Patient confidentiality
Avoid accidental injury to the patient is called -6
near miss
Close shave
Close thing
Narrow (something)

- AHG: + autocontrol -7
SC1: IS PHASE AND 37 PHASE NO REACTION BUT AHG SHOWS REACTION
+2
SC2: IS PHASE AND 37 PHASE NO REACTION BUT AHG SHOWS REACTION
+2
SC3: IS PHASE AND 37 PHASE NO REACTION BUT AHG SHOWS REACTION
+2

low incidence ag
High incidence ag
Multiple autoantibodies
Single autoantibodies

Some ec ion ere hick on H&E ain e en ho gh he micro ome was fixed -8
on 2-3 microns. why
block was loose
Recalibrate and do maintenance to the microtome

In pa ien afe em, Pa ien afe c l re mean -


safety report system
Patient confidentiality

Heterophil test -10


EBV
case: abdominal pain and diarrhea no ova, cyst or trophozoite in stool but string -11
test was positive what is the pathogen
Giardia

Hainey
cell
wekemia
G
HCL marker -12
CD11c
24g103
123
19 gHc
CD125

B-CLL markers -13


19g20122 20 1 5 4 -
20 1 7 3
13 33

Antibody against different species -14

xenoantibody
Zeroantibody
Alloantibody
Autoantibody

A 16 years old boy broke his leg on football game and urine sample showed 2+ -15
protein and dark turbid urine why

Kidney injury
Kidney failure
Sternous exercise
Energy drink

A woman urine sample on immuonoassay showed positive for opitaes but in -16
advanced chromatography it was negative why
some over the counter medications contain codeine
Vitamins and orange juice affected the results

A man had a car accident and he has history of heroin use. Acetylcodiene and -17
acetylmorphine were tested. Blood sample showed negative but urine showed pos
why
blood can be easily manipulated
Blood is not the sample of choice in this case
Drug markers are more stable in urine

Delivered AB+ baby needs exchange which blood is suitable -18


+AB
-O
-AB
+B

A AB- patient needed 4 units of blood and the technologist found only 3 AB- -1
what other blood unit can be given to him
-A-, O- or B
+A- O+ B

Which HLA antibodies is the marker for Behçet's disease -20


HLA-B51

...ANA, AMA and ASMA are markers for -21


Autoimmunity
Immunodeficiency
Bacteria causes Endocarditis -22
Alphahemolysis Viridans
Strep A
hemolysis
p agalactiae Strep B
Strep c

Which an er i correc for polic of pa holog req e -23


must be filled by nurse
Must be handled by doctor in schedule
Must be accessible for health workers who transport samples
Must be handled by receiving section in the lab (something like that)

What should you put on acid spill to neutralize it -24


Baking soda
Sodium hypochlorite
Vinegar

Amount of T cell of whole lymphocytes in whole blood -25


%20
%70
%100
%1

Temperature of paraffin wax -26


C 20-10
C 60-30
C 70-40
C100- 0
What Causes the reaction in GVHD -27
T cells
B cells
NK cells
Neutrophils

What to give patient to avoid GVHD -28


Irradiated blood

Sickle cell anemia is due to which nucleotide substitution -2

A T
T A
resultin
valine G C
substitution
C G

material safelydata sheets


What is the content of MSDS in chemical safety program -30

Product information
Chemical use

Bence jones in urine -31


Multiple myeloma

What is indicative of multiple myeloma -32


Bence jones protiens

Case: hb high, wbc high, platelets high -33


JAK 617F mutation

APTT: N / PT: N / TT: prolonged -34


what is the most probable factor
X
XI
XII
I

Aptt prolonged whats the next step -35


Mixing study

Vit K deficiency what is the affected factor -36


II
V
VIII
XIII

A2B with Anti-A1 discrepancy case -37

A2 with Anti-A1 discrepancy case -38

Pos DAT indicative of -3


HDFN
incompatible Crossmatch

Case: patient came with fever and the pic shows schistocytes -40
a
Hb
Platelets
TTP
ITP
HUS

Vancomycin resistant bacteria -41


Enterococcus faecalis

Pic with sickle cells and lab findings indicate -42


sickle cell anemia

Case with fatigue and paleness -43


Lab findings
Ferritin
Iron
TIBC

IDA

Bacteriophage T4 on E.Coli -44


destruction
inhibition

Woman pregnant with the first child has a cat at home which test should be done -45
CMV
Toxoplasma

What ensures long use of lab instruments -46


training
Maintenance
Good inventory

Troponin control sample exceeded +/- 3SD what should u do before reporting -47

report the results


Rerun the controls
Calibrate the newly calibrated instrument
Use new reagent lot

Brea i ec b he pa hologi didn fi in he ca e e ha ho ld do 48-

process it anyway it will shrink later


ask pathologist to cut smaller sections
use larger cassette
put it in formalin

Case: patient came with tenderness in the upper right quadrant of abdomen -4

Lab findings: all normal but lipase is very high

pancreatitis
Appendicitis

Case: fever with chest pain -51

pneumonia
TB
Myocardial infarction
Case: Patient came with fever, vomiting and pain and has history of acute -52
hepatitis

Lab findings
AST
ALT
ALP: normal
Bilirubin

acute hepatitis
Chronic hepatitis

Case with Gamma-beta bridging -53


liver cirrhosis

benigntumor ofadrenal
grand
What test is confirmatory of Pheochromocytoma -54
serum catecholamines
Serum vanillylmandelic acid

hours urine catecholamines 24

The relatively new implementation of liquid chromatography coupled with -55


tandem mass spectrometry (LC- MSMS), have disadvantage which is matrix effects
phenomenon, how can you avoid this

Dilute the sample 1/10 times

Who is prohibited to see the patient record -56


intern
Patient
Doctor of the case
The pa ien legal g ardian

Xenograft definition -57

Urease positive found in stool -58

we shigella

weSalmonella typhi
we Proteus mirabilis
ve Kelbsiella Pneumonia

Which biochemical reaction is positive for pseudomonas aueroginosa -5


Oxidase

What causes diarrhea due to eating rice -60


Bacillus cereus

Case: a man cam with history of drinking raw sheep milk -61
Brucella

What is a nocturnal parasite -62


Wucherichia bancrofti
burgyei malayyi

Low parasitemia what do u do -63


thick film
Thin film
CBC
RBC count
Case: infant with meningitis -64
Lab findings of CSF
Low glucose high protein
Wha he mo probable ca a i e pa hogen
group B streptococcus

Case with findings of -65


HCO3
PH
CO2: N

Metabolic Alkalosis

Case with findings of -66

HCO3: N
PH
CO2

compensated respiratory alkalosis


Compensated metabolic alkalosis

because Uncompensated respiratory alkalosis


the PH Uncompensated metabolic alkalosis
is abnormal

Case: woman with vaginal itching, dysuria and white discharge like cottage -67
cheese what is the most probable pathogen

candida albicans

Case: woman with green frothy vaginal discharge, dysuria and lab findings -68
showed jerky motility what is the most probable organism
Trichomonas vaginalis

What is found in type 2 Diabetes -6


high glucagon
Pancrea doe n prod ce in lin

What is the nature of reaction that happens of ab agains ABO incompatibility -70
complement sensitize antibody on the antigens on the cells and cause cell
lysis/destruction

What is the nature of Monoclonal Antibody used in DAT -71


IgG and cd3
IgG and cb3 and cd3
Mural IgG

MIC on bacteria by using ampicillin gave 1 mg and by pencilling gave 8 but -72
MIC after using them together was 0.3 what is this combination named
synergism
Antagonism

A table of four patients with different CDEce expression. Which patient will -73
show R2r? (Check table of wiener terminology in blood bank ppt)
Patient No. 4 -

To calculate BUN -74


2.14
What is the correct formula of the anion gap -75
(Na + K) - (cl + hco3)

Case: patient who is a vegetarian had fatigue. Blood smear showed ovalocytes -76
and target cells. What should be seen in blood smear
hypersegmented neutrophil

What is howell jolly body composed of -77


DNA

What is heinz body composed of -78


denatured Hb

...Immunized individual against HBV by vaccine should show -7


Anti-Hbc negative, anti-HBs positive) (

A donor came with.. pulse: 140bpm, blood perssure: 100/70mmhg, 37 C, 51 kg -80


Why was he deffered
bpm 140

A woman came to donate she was in london between 1993 and 1994, italy in -81
1999 to 2000, france in 2005 to 2006 and Australia from 2009 to 2015. Why was she
deferred
italy
Australia
france
London

A Nigerian man came to saudi arabia two years ago and became a nurse in -82
Riyadh. He is healthy
he should be deferred for 1 year
He should be deferred for 2 years
He is healthy and can donate
A female had grey patches around cornea -83
Lab findings
LDL
Cholestrol
Triglycerides N
What does she have

Familial cholestrolemia

Wha ill o find in rine in il on di ea e -84


increased copper

Pic of basophilic stippling -85

Baby hemoglobin type on 7th month -86


A

A patient came after a week of her appendici i rger and he o nd ha n -87


healed and i ill bleeding ha i he ca a i e fac or
XIII
X

Immunoglobulin seen in tears and mother milk -88

igE
igA
IgG
IgM

Immunoglobulin in recent infections -8


IgM
Patient showed positive VDRL what is the next test to be done - 0
RPR
FTA-ABS
Darkfeild

Whats the next step after clearing - 1


infiltration

In which step does pathologist explain and cuts the tissue - 2


grossing

A blood sample showed cold agglutinins what do u do - 3


dilute and rerun
Warm to 37 and rerun
Report normally

What type of tube do you use in serology - 4


plain
edta
Heparin
Citrate

Pic: mucoid fungus on blood agar - 5


Why mucoid
capsule production

Which HIV sample wont cause transmission - 6


CSF
Blood
Urine
pleural fluid
Which organism has no cell wall - 7
mycoplasma

Case: a woman came with joint pain and deformity when she stands up what test - 8
should be done
Rheumatoid Factor

A technologist was doing validation on an instrument and the company -


representative asked him to advertise the instrument for other laboratories staff what
is this called

conflict of interest
False diagnosis
Working in a hostile environment

Saliva sample for biochemical test -100


shaking
Heating
centrifugation

What causes low ESR -101


grossly hemolyzed sample
Tilted tube
.High room temp

Pic: haemagglutination test -102

Last step in ELISA-103


add substrate

Smallest amount can cause death -104


Poison

Findings in gilbert syndrome -105


increased unconjugated bilirubin

CPD Shelf life -106


days 21

Auer rods found in -107


Myeloblasts
lymphocyte

Usually seen in urine of young females -108


s. Aureus
S. Saprophyticus

A echnologi didn appl an i ep ic before dra ing blood from a bo and he -10
boy developed bacterial infection what is the most probable cause
s. Epidermidis

Sterilization -110
autoclave
Dry heat oven

What PPE to use when handling hazardous sample -111


gown goggles and gloves
Gown goggles
Gown gloves
Gloves mask

Normal hb and normal mcv -112


normocytic normochromic

pic: microsporum macroconidia -113

Pic: histoplasma capsulatum macroconidia -114

Marker for thyroid carinoma -115


calcitonin
hurthle cells

Marker for alcoholic liver -116


GGT-

color of amorphous phosphate microscopically -117


black
Yellow
Pink
White

Pic: rouleaux formation -118

gram negative bacteria that are diplococci -11


N. Menengitidis/gonorrhea (according to the case wether its from csf or
cervix sample)

most common strain of H.influenzae -120


A
B
C

Which antigen shows dosage


I
P
Kidd
Lutheran

turbid semen means -121


bacteria
WBC
which is a Kell ag -122
JKa
Jkb
Ka

glanzzman disease is associated with -123


platelet aggregation

retention of patient record in blood bank -124


years 10
year 1
years 2

what is suitable for enterobactericiae growth -125


Anaerobically
Aerobically
Aerobically + 15% CO2
Aerobically + 10% CO2

antibodies formed by bombay individuals -126


Anti-A, B and H

used for quantitation of electrolytes -127


ion selective electrode
most random urine sample -128
random
hrs 24
Clean catch
steatorrhea in stool due to -12
fat

meaning of CBAHI -130


Saudi Central Board for Accreditation of Healthcare Institutions
urine formation comprised of -131
reabsorption, filtration and excretion

Case: A woman came from india with severe vomiting and diagnosis showed -132
) ( 50 immature eggs
hook worm
Pin worm
Whip worm

donation of autologous patient requires -133


Hb at least 11 g/dl
donation and transfusion in same facility

cryoprecipitate used for what deficiency -134


Fibrinogen

bacteria grows in 43 -136


campylobacter

niacin and tryptophan deficiency -137


pellagra

how is pleural fluid preserved -138


c4
.Room temp
...hrs in 48

donor came with skin lesions -13


send to the doctor for physical examination

protein that aid in iron-binding -140


ferritin
Transferrin
Albumin

false negative/positive (?) DAT -141


wash cells

delivered AB( +/-?.. can remember)bab ha need blood e change ha i -142


the suitable blood group
+AB
-A
+B
+O

In phenylketonuria there is a deficiency in -143


tyrosine

patient with lockjaw what neurotoxin to look for -144


Tetani/tetanus

high ALP -145


page di ea e

normal urine PH -146


6-5
what condition is suitable for rbc transfusion -147
aplastic anemia

‫ا‬ ‫ا‬ ‫د‬- ‫ا‬


1. Which of the following methods may be employed to remove IgG
antibodies that are coating a patient s red blood cells
a. Adsorption
b. Elution
c. Neutralization
d. Titration

2. The fecal-oral route is common in transmitting which of these


hepatitis viruses?
a. HAV
b. HBV
c. HDV
d. HCV

3. Which of the following is the most frequently transmitted virus


from mother to fetus?
a. HIV
b. Hepatitis
c. CMV
d. EBV

4. RhIG is indicated in which of the following circumstances?


a. Mother D-positive, infant D-positive
b. Mother D-negative, infant D-positive
c. Mother D-positive, infant D-negative
d. Mother D-negative, infant D-negative

5. Which complement pathway is activated by the formation of


antigen-antibody complexes?
a. Classical
b. Alternative
c. Lectin
d. Retro
6. An individual has R1r Using Fisher-Race terminology, what is the
most likely Rh genotype?
a. DCE/Dce
b. DCE/dce
c. DCe/dce
d. DCe/DcE

7. Cryoprecipitate that has been pooled must be storage at______ and


transfused within ______ hours.
a. 24 hr at 20-24 °C
b. 6 hr at 1-6 °C
c. 4 hr at 20-24 °C
d. 8 hr at -18 °C

8. Which of the following can pass through lipid layer easily ?


a. CO2
b. Protein
c. Glucose
d. Starch

9. Tumors marker conceder as :


a. Diagnostic test
b. Confirmatory test
c. Screening test
d. Monitoring after diagnosis

10. After blood transfusion for 30 min , patient fever rise from
37 °C to 39°C . what is the type of reaction?
a. Delayed hemolytic transfusion reaction
b. Acute hemolytic transfusion reaction
c. Allergic reaction
d. Febrile nonhemolytic reaction
11. Hemophilia C , deficiency of :
a. Factor VIII A
b. Factor IX B
c. Factor XI c
d. Factor IIV

12. Patient come to clinic with signs of increase blood sugar


and he has ketoacidosis, what test should do for him ?
a. Insulin level
b. random sugar
c. HA1C
d. Fasting sugar

13. Redness , swelling and itching :


a. Adaptive response
b. Innate response
c. Passive response

14. Children came to clinic with high fever , abdominal pain ,


diarrhea , microbiology plate show positive for shigella .What type
of leukocyte will increase in his blood ?
a. Neutrophil
b. Eosinophil
c. Basophil
d. Lymphocyte

15. Most antibody reagent (antisera) used in blood bank:


a. A1
b. B
c. O
d. A
16. Crossmatch is :
a. Direct atiglobulin test
b. Indirect atiglobulin test
17. Chemical that use as disinfectant for lab benches?
a. sodium hypochlorite
b. 30% ethanol

18. Storage for cytology sample:


a. 2 years
b. 10years
c. Permanent

19. To confirm negative occult blood test :


a. Three samples from the edge of the specimen
b. two samples from the middle of the specimen
c. Three samples from the middle of the specimen
d. two samples from the edge of the specimen

20. What blood banker should give for patient taking blood from
relatives?
a. Irradiation blood
21. Donor come with skin lesion in both arms what should phlebotomist
do:
a. Send him to physician to evaluate
b. Tell him to come after 2 weeks
c. Permanent deferral
d. Ask blood bank supervisor

22. Patient come to histopathology lab , he request for his wife report ,
he has family ID . what you should do ?
a. Send him to medical report
b. Don t give him
c. Give him
d. Send him to physician office
23. Lewis antibodies:
a. IgG , cause HDN
b. IgG, not cause HDN
c. IgM, cause HDN
d. IgM, not cause HDN

24. Most abundant Haemophilus influenza:


a. A
b. B
c. C
d. D

25. Staff did big mistake which harm the patient at night sift .
what supervisor sould do ?
a. Finishing his contract
b. Schedule him in morning shift
c. Give him 3 months training

26. Most abundant mineralocorticoid hormones :


a. Aldosterone

27. Blood Lab test give indication of integrity of the liver:


a. ALT
b. AST
c. Albumin
d. Ammonia

28. Patient has high amylase and lipase level ,positive for CA19-
9 . what is the diagnosis?
a. acute pancreatitis
b. chronic pancreatitis
c. pancreatic cancer
29. to prevent GVHD :
a. give patient irradiated blood .

30. Tumor marker for breast cancer ?


CEA CA15 3
31. confirmation test for CLL?
flowcytometry
32. DNA stain in electrophoresis?
Ethedium bromide
33. If transfusion reaction happened in allo transplantation ,
which cell will react :
a. B cell produce antibodies
b. T cytotoxic will lyse the cells
c. T helper
d. LK production

34. Hormone involve in increase osmolality and water


reabsorption:

35. Holly Jolly bodies contain of :


a. Iron
b. DNA
c. RNA
d. Hemoglobin

36. In which condition we can found pappenheimer bodies and


what it is contain of ?
a. Iron
b. DNA
c. RNA
d. Hemoglobin
37. heinz bodies contain of what , and in which condition we
found it ?

38. Thalassimia is :
a. Macrocytic , hypochromic

39. In histopathology, what kind of stain used for Biopsy from


skin ?

40. If the cut can not fit the cassette what you will do ?
a. Call the physician to re-cut
b. Put it in the cassette because it will shrink anyway
c. Cut it by yourself

Whey we can not use GGt,ALT,AST alone for diagnosis


d. Expinsive reagent
e. They are indication for other things ( non-specific)

41. What to do with this ABO discrebancy :

Patient Cells With Patient Serum With


Anti-A Anti-B A1 cells B cells
4+ 4+ 4+ 4+

a. Mix sample and rerun


b. Wash RBC

42. You run CBC sample and the machine give you flag for very
low platelet count , what you will do ?
a. Rerun the sample
b. Run QC
c. Stop the machine and do calibration
d. Report the result
43. lipemic sample for CBC ,what of the following will affected
a. RBC
b. WBC
c. Platelet
d. Hemoglobin

44. Blood gases sample transported with non-anaerobic


condition , how will affect CO2,PH,O2
a. CO2 decreas,O2increase,PH decrease
b. CO2 increase ,O2decrease ,PH increase
c. CO2 increase ,O2increase,PH decrease
d. CO2 decreas,O2decrease ,PH increase

45. probability of sickle cell trait parent to have their first baby
with sickle cell disease?
a. 50%
b. 90%
c. 70%
d. 10%

46. If the mother O and the father B , what is the most likely
blood group for the baby?
a. B
b. O
c. B or O
d. AB

47. Proper volume for two blood culture ?


a. 10-20ml
b. 15 ml for each
c. 20-25ml
48. Gram positive Beta hemolytic Bacteria and sensitive for
bacitracin ?
a. Group A S.pyogenes

49. Principle of catalase test .

50. Bacteria cause bacillary dysentery ?


a. Shigella

51. Grme negative bacilli bacteria cause UTI, oxidase neg,


lactose neg, urease +
a. Proteus mirabilis

52. Bacteria can grow in maconky agar:


a. Neisseria
b. S.pyogenes
c. S.aureus
d. Pneumonia

53. Positive complement test :


a. Adherence.
b. Agglutination.
c. Hemolysis.
d. Precipitation.

54. Screening test for syphilis :


a. PCR
b. Serology

55. What Class of Biological Safety Cabinet need for TTB test ?
56. A fire broke out in the laboratory and the fire extinguisher
in the other lab, what you should do ?
a. Scape , activate the code
b. Informs the supervisor
c. Activate the code , close the room , tell others , scape
d. bring the fire extinguisher and fighting with the fire

57. factor will initiate coagulation cascade:


a. tissue factor
b. factor IIX
c. factor IX
d. factorXI

58. increase ESR observe in :


a. inflammation

59. which of the following is normal in urin :


a. bilirubin
b. glucose
c. urobilinogen
d. protein

60. Bacteria that can cause Abortion:

61. Sample must processes it as soon as possible in histo :


a. Biopsy
b. Immunohistochemistry

62. What does Titration (1:5) mean ? how many drops of


patient blood and how many drops of the reagent ?
63. Type of hepatitis come along with hepatitis B ?

64. Effect of increase blood urea in coagulation (PT . PTT,BT,..)

65. Which factor will convert prothrombin to thrombin ?

66. Diagnostic test for breast cancer?

67. Hormone that can extract fatty acid from adipose tissue?

68. Which of the following is non direct antigen ?


a. Kull
b. Lewis
c. Fya
d. kidd

69. Drug that cause permanent deferral in blood donation.


a. Paracetamol
b. Vaccine

70. When you receive Amniotic fluid sample what is the first
thing to do ?
a. Heating
b. Centrifuge

71. Purpose of washing the RBCs


a. To facilitate sensitization of RBCs
b. To remove free antibody

72. Road of transmission for TTB


73. Where we can found the target cells

74. Specific CK for heart :


a. CK-MM
b. CK-MB
c. CK-BB

75. Percentage of anticoagulant in sodium citrate (blue tube):


a. 3.2%

76. Lab specialist accept sodium citrate (blue tube) with


insufficient volume , who this will affect the result of coagulation
test PT,PTT ?

77. Definition of : Standard deviation , Accuracy, toxicology ,


affinity.
Standard deviation:

Accuracy:
is how close results are to what is expected from a test.
Toxicology:
the branch of science concerned with the nature, effects, and
detection of poisons

Affinity:

78. Calculation :
MCHC (g/dL) = Hb / HCT x 100.
HDL
79. What is the blood group for this patien ?
a. A, Rh +
b. B,Rh+
c. O,Rh-
d. AB,Rh-

80. Identify this picture :

giant platelets

81. One Question about lead poisoning.


82. 4 questions in virology , one of them picture of virus and
you have to identify .
83. 2 questions in microbiology about media contents.
84. Cause of DM type 1 and Type 2, lab findings for both.

85. Donor come to blood donation area with :

pulse 170
Weight 50 kg
Blood pressure 80/50 mm/hg
fever 37 C
He is temporary deferral because of :

a. Pulse
b. Wight
c. Blood pressure
d. Fever
86. Normal alkaline urine crystal:
a. uric acid
b. Calcium oxalate
c. amorphous urates
d. amorphous phosphates
e.
87. One question about Transfusion-related acute lung injury
(TRALI).

88. One question about Intrauterine transfusion (IUT) and


neonatal transfusion

89. One question about Acid-base and arterial blood gases

90. Increase A2 hemoglobin observe in :


a. Thalassemia
b. Sickle cell anemia
c. Hemoglobin C disease

91. Hemophilia A , deficiency of :


a. Factor VIII
b. Factor IX
c. Factor XI
d. Factor IIV
1.To deferential between group B and A streptococcus ➡

CAMP test
*Group A and B.Streptococcus are B.hemolytic.

2. Bence Jones protein found in ➡patient with multiple myloma.

*Bence Jones protein is Kappa or Lamda light chain of antibody.


*Bence Jones protein detect in urine by heat(45-60C) and
disappear when boiling.

3.Bacteria cause gas gangrene(in Diabetes mellitus patients) ➡


Clostridium perfringes .

4. Anti-coagulant use in blood culture to prevent colt

SPS (Sodium polyanethole sulphate)

5.Cylindrical parasite ➡ Ascaris lumbricoides.. Hook warm

***cylindrical larvae ➡ strongyloid stercoralis

6.Anticoagulant use for coagulation profile (PT,PTT,TT)➡3.2%


Na citrate
***3.8% for ESR.

7.Is statement describe of fungi ➡ has 80 ribosoms for eukaryote


like fungi
***70s for prokaryote like bacteria.
***Fungi:
*Eukaryote.(no mesosome)
*Yeast(unicellular,oval/round,sexuall/metosis).

1
*Molds(multicellular,produce hypha,Asexuall/mycelial
fragmentation).
*Membrane contain chitin.

8.Hematuria causes by parasite ➡ Schistosoma haematobium

9.To detect anemia in for student school use ➡ Hb and PCV or


CBC.

10.Type of water uses in lab ➡ type II

* distal water.
*Filter water-Tap water
‫حد‬ ‫ر‬ ‫تر‬ Deionized water Distal water

11.device uses in Lab to sedimentation of suspention➡ centrifuge

12.Mycology is study of ➡ fungi

13.Cell in parasitic infection ➡increase eosinophil

14.Cell in bacteria infection ➡ increase neutrophil

15.Cell in viral infection ➡ increase lymphocyte

16.Deionization of water➡ exchange to proton and electron

‫ز ا‬ ‫ا تباد‬ ‫ر ب حص‬ ‫ا‬ ‫ات ت ر با د ا‬ ‫ازا ا‬ ‫ا‬

17.In obstructive jaundice➡ pale stool and increase in ALP


enzyme
**Alt/Ast slight elevated
liver ‫ا‬ ‫بد‬ ‫ا ر ا ا سداد‬ ‫ا را ب‬ ‫**دا‬
stone or pancreatic cancer ‫ا ا‬ ‫اا ش‬

2
‫ا ست ب ا‬ ‫ب‬ ‫ا‬ ‫ا زا ا‬ ‫د ب‬ ‫ا جر ا ب د ا بد شا‬
‫ا ب‬ ‫ا دا‬ ‫اتح‬
***In obstructive jaundice.. Direct bilirubin return to the liver
lead to liver necrosis (some cell) ▶abnormal function ▶increase
unconjugated bilirubin.. But direct is more elevated.. ALP more
specific to obstructive jaundice Alt/Ast slight elevated.
ALT/AST.. ‫ ا‬obstructive .. ‫ا دا ر ت با ر ب ا ا در‬ ‫ا ر ا‬ ‫اب‬
‫دا ب‬ liver necrosis.. ‫ت ا شا‬ ‫ا ادر ت ب‬ ‫ تب‬..‫ا‬ ‫ب ضا‬
‫ ا‬..‫ بس ا دا ر ت ا تر‬..‫ا دا ر ت‬ ‫ز اد برض‬ ‫ د ب‬alp.. ‫ز ا تر‬
.. ‫ر ا ا سداد‬

19.To convert of fibrinogen to fibrin ➡ by coagulase enzyme

‫ز ا زا ب سا د ا ثر ب‬ ‫ا‬ ‫ا‬ ‫ب اس‬ ‫بر‬ ‫ت‬ ‫ج‬ ‫بر‬ ‫شا اح‬

20.Martin bell syndrome ➡ defect in x chromosome(fragile x


chromosome).
‫باد‬ ‫ر س‬ ‫ب‬ ‫ رض ج‬mental retardation
‫باس ا ا ا تش‬ ‫س‬

21.Scotch tape test used to detect ➡ ova of enterobius


vermicularis.

3
22.India ink stain➡ capsule mainly for Cryptococcus neoformans

23.Hypothyroidism ➡ T3T4 low.. TSH high

‫ات ا‬ ‫ر‬ ‫ص‬ ‫ب‬ ‫ا د‬ ‫ش‬

24.CPD storage blood for➡ 14__21 days

‫د‬ ‫اا‬
*CPDA for 35 days only Citrate phosphate dextrose adenine.
**CPD for 14__21 days Citrate phosphate dextrose.

25.Deferential between plasma and serum➡ Serum contain


fibrin.Plasma contain fibrinogen

26.Not found in eukaryotic➡ mesosome

* ‫ا بر ار ت‬ ‫ج د‬ ‫ا بر ار ت‬ ‫ار ت‬ ‫ا ت ات ا‬ ‫احد‬

27.HDN causes by➡ igg antiD(Rhesus HDN)

HDN ‫اسباب‬ ‫ا‬ ‫سا ب‬ ‫حب ا‬ ‫ابب‬

28.Defrential between AML and ALL ➡ Auer rods

AML ‫ا‬ ‫ص ر بت‬ ‫ا ر ر د دا ز ابر ا‬

4
29.Temp of denaturation in DNA ➡ 94__96 C

*Denaturation ➡ 94__96C

**Annealing ➡ 50 65C and Extension ➡ 72 C ‫ات‬ ‫ا‬ ‫د با‬.


‫درجت حرارت‬

30.Shigella on XLD with phenol red ➡ give red colour .

31.platelets storage in blood bank ➡ 22__27 C for 5 days

32.Nucleated RBCs found in➡hemolytic anemia and


megaloblastic anemia
‫تد‬ ‫ا‬ ‫ا ت‬

33.cushing syndrome ➡ increase cortisol hormone and ACTH

34.anemia with thrombocytopenia ➡Aplastic anemia &


megaloblastic anemis
‫صا بتا ص ا ح د‬ ‫بت‬ ‫د ا ت‬

35.Fishy smell in media characteristic for ➡ Proteus

‫بر ت س‬ ‫ا‬ ‫د‬ ‫ارح اس‬ ‫ت‬

36.Malaria routinely diagnostic by ➡ thick blood film

And use thin blood film to deferential between malaria spp

*Plasmodium malarai life cycle ➡72 h

37.visceral leishmania cause by ➡ leishmania donavi

38.Monocopy mean ➡ one copy

‫احد‬ ‫س‬ ‫ب‬ ‫ا حاج د ادت ا‬ pcr ‫ات ا‬ ‫د‬


5
39.colour of gram positive bacteria ➡ violet colour

40.colour of gram negative bacteria ➡ red colour

41.glucose-6-phosphate dehydrogenase (g6pd) deficiency is type of➡


heamolytic anemia.

42.Known amount of analyte ➡ STD Or stander

‫ا ستا در‬ ‫ز‬ ‫ا تر‬ ‫اد‬

43.Bacteria oxidase positive ➡ vibrio.. pseudomonas.. nesseria ..


h influenzae
‫ا س د س ب زت‬ ‫د‬

44.specific enzyme to diagnosis of acute pancreatitis ➡ lipase

.‫ب ز ا تارا ا ز‬ ‫جاب ا‬ ‫ار‬ ‫ا‬ ‫جا‬ ‫ا ا ز اسبس‬


،‫ص‬ ‫ال‬ ‫قب ل‬ ‫ر دق‬ ، ‫طاق اس‬ ‫ز اح‬ ‫أ از ا‬ ‫الر‬
.‫ص ال اب الب ر اس الحاد‬ ‫لل‬ ‫ال‬ ‫ا ز ال باز‬

‫ا‬ ‫ات ا بتج‬ ‫ا‬ ‫ا ت س ب ز ا حاد‬ ‫سؤ ا‬

45.Intermediat host of toxoplasma ➡ Human

Definitive host ➡ cat

‫ا‬ Diagnosis Serology or ELISA‫ تش ص ب‬Protoza ‫ب‬


Intracellular parasite-multi intracellular- infective stage oocyte in
stool
fetal cerebral infection ‫اج اض س‬abortion ‫ش بت‬ ‫ب اا‬

46.Rh antigen has 5 antigen➡CeDEc

6
**Rhesus blood group have more than 50AG,most important
(D,c,C,E,e)
‫حر‬ ‫ا ا تج ات بد‬ ‫ا ر صص بت‬ ‫ش‬ ‫اس ا ب‬
. ‫د‬ ‫س ا حر‬ ‫ا‬ ‫ا‬

47.CD2 bind with ➡CD58

**Tcell (CD2) bind with Antigen presenten cell CD58(APC).

48.PCR 2 cycle ➡4 copies.

49.One cycle give 2 copies.


**Numbers of copies calculated by 2n(n=number of cycle)
‫ر‬ ‫ات‬

50.DNA storge in ➡DNA bank

51.Analytical error ➡caliberation.. Proceduer..reagent

‫ا سس‬ ‫اث ا‬ ‫ا تحدث‬ ‫ا‬ ‫ا‬ ‫د‬

7
52.Post analytical error ➡result interpretation Or report

53.Albert stain ➡stain for Corynebacterium diphtheriae

‫ا د تر ا‬ ‫را ب اتر ا بتسس‬ ‫ا ب تر ا ا‬ ‫اص‬

**Eleck test ➡for Corynebacterium diphtheriae

Chinese letter ‫ا ا‬ ‫با ضا‬ ‫اص ات ا د تر ا تست دا ا ست‬ ‫د ا‬

***Lowfer serum media for ➡ Corynebacterium diphtheriae

‫برض د تب‬

54.Harmful cholesterol carry by ➡LDL( Low density lipoprotein)

atherosclerosis ‫ار بسبب‬ ‫ا‬ ‫ستر‬ ‫دا ا‬

55.trachoma causes ➡ by chlamydia

‫دا‬ ‫ا‬ ‫ت ر با ر د بتسبب‬ ‫ا‬ ‫دا رض‬

*Elementary body ➡ is infective stage of chlamydia

*Reticulate body ➡ Active metabolic body of chlamydia

‫دا‬ ‫اا‬ ‫دا ا ت ا‬

56.septicemia and gas gangrene caused by ➡Clostridium


perfringens

57.Reed–Sternberg cells ➡ hodgkin lymphoma

8
‫د‬ ‫ا‬ ‫ز‬ ‫اب‬ ‫ا‬ ‫ا بتشب‬ ‫اا‬ ‫د‬

58.hydatid cyst disease is caused by ➡ echinococcus granulosus

‫ا بد بث ا تر ش ب ص ب ا بد‬ ‫ا‬ ‫ار‬ ‫س ست ح ا‬ ‫برصا ت بسبب‬

59.leishmania transmitted by ➡sand fly

❤‫حس اء ا حشرات‬ ‫ا سا د‬ ‫ا‬ ‫ا ش‬ ‫ا ا‬

60.Dwarf warm ➡ H. nana

‫با‬ ‫ا صر د د‬

61.characteristic of mycobacterium tuberculosis ➡acid fast bacilli


stained by Z N stain

62.pinworm ➡ enterobius vermicularis

‫ا د د ا دب س‬

63.Food poisoning causes by ➡salmonella enteritidis

‫ا ذا‬ ‫ا تس‬ ‫بتسبب‬ ‫ا سا‬

64.Amoeba move by ➡ pseudopod

‫ا دا ا اذب‬ ‫با بتحر ب اس‬ ‫ا‬

65.Gardia move by ➡ flagella

66.‫ ا‬Capsulated bacteria ➡

9
67.Gravet segment in stool detect of ➡ teaina saginata or soluim

**Teania solium and saginata ..differentiated by their gravid


segment
‫ب ب ر ب ات ب ا ت‬

By raw meat
T. saginata in beef meat and T. solium in pork meat

68.Hashimoto disease is ➡autoimmune and is hypothyroidism

10
Also called, Hashimoto's thyroiditis is an autoimmune disease, a disorder in which the immune system turns against
the body's own tissues. In people with Hashimoto's, the immune system attacks the thyroid. This can lead
to hypothyroidism

‫ات‬ ‫ا ر‬ ‫ت ا ت ث‬ ‫ا د ب‬ ‫بصب‬ ‫رض ا‬ ‫ا اش ات‬


‫ا بت رز‬
gravis same as hashimato?!
Hashimoto hypo gravite hyper ‫بث ب ت‬auto immune disease ‫ا ت‬

69.Candida can be stain by ➡ gram stain and give blue colour

70.latency period ➡ herpes vires


‫رض‬ ‫ب‬ ‫حت‬ ‫اا‬ ‫ت اثر ب‬ ‫اد‬ ‫ا د ا ب ا د ا ا ر س دا جس ا سا‬ ‫ا ت س بر د‬
. ‫ز ب ا ا ر سات ا ربس‬ pathogenic ‫حت ب‬ ‫شج ا ساس‬

71.corynebacterium diphtheriae ➡appear as Chinese litter

72.secondary structure of protein ➡ alpha helix and beta sheet.


11
‫‪73.Dimorphic fungi ➡ yeast and mold‬‬

‫درج ا حرار‬ ‫ثابت با ت‬ ‫ش‬ ‫د‬ ‫ش‬ ‫دب ا‬ ‫ا ست ا‬ ‫ا‬


‫‪budding‬؟ا‬ ‫ب‬ ‫ا ت‬
‫ا ا ا ر اب بتا ت ‪DNA‬‬ ‫ا س ا ا ب د حارس‬
‫‪12‬‬
74.In DNA

purine ➡ A and G

Pyrimidine ➡ T and C

double bond ➡ between A--T

Triple bond ➡ between C---G

75.In RNA

Purine ➡Aand G

Pyrimidine ➡ U andC

Double bond➡ between A--U

Triple bond ➡between G---C

76,Most important organ in buffering system ➡ lung and kidny

Acid base balance ‫ا‬ ‫بد‬ ‫ات‬ ‫د ا‬

77.virus like sand ➡ arena virus.

‫ا ر ا ا رس‬ ‫بات ا ر‬ ‫ا رس ب شب‬

13
78.B lymphocyte mature in bone marrow
79.T lymphocyte mature in Thymus gland
. ‫د ات ت‬ ‫ات‬ ‫ر‬ ‫اب‬ ‫ا ات ر ش‬ ‫ا ب ب حص‬

80.Anisocytosis ➡variation in size of RBCs

81.Poikilocytosis ➡ variation in shape of RBCs

RBCs ‫ا ا‬ ‫اش ا ا‬ ‫احجا‬ ‫ا ت‬ ‫ص حب‬

82.site of absorption of iron ➡duodenum and upper of jejenum.

. ‫ا ا تصاص ا حد د‬ ‫د ا حت ا بت‬

83.philadelphia chromosome positive in ➡CML Choronic myloid


leukemia

84.In direct comb test or in direct antiglobulin test ➡ to detect


Ab in vitro
use in cross matching

85.Direct coomb test or direct antiglobuline test➡to detect


sensitize RBCs coated ab In vivo

14
86.western blot test ➡is confirmatory test of HIV

87.hyperparathyroidism ➡ causes hypercalcemia

‫ا د بد ت ز اد ا س‬ ‫شا زا د‬

88.Thayer martin agar ➡ is media contain antibiotic for neisseria


spp mainly for n gonorrhoeae

89.Antibiotic inhibition of RNA ➡Rifampicin

90.antibiotic inhibition of protein ➡Aminoglycosides

91.Hormone important in fertility ➡progesterone

❤‫دا‬ ‫را‬ ‫د‬ ‫بث اس ا‬ ‫ات‬ ‫ا ر‬ ‫ا‬

92.Syphilis causes by ➡ treponema pallidum

‫ا‬ ‫ا تر‬ ‫بتسبب ا‬ ‫رض ا ز‬

93.cell increase in allergic reaction or hypersensitivity ➡ basophil

94.ALL ➡ 2__10 years

15
95.AML ➡50__ 65

96.CML ➡40-60

97.CLL➡60-80

98.LDH 1 more than LDH2 ➡called a flipped pattern

**LDH has 5 isoenzyme

Different LDH isoenzymes are found in different body tissues. The areas
of highest concentration for each type of isoenzyme are:

LDH-1: heart and red blood cells . LDH-2: white blood cells,serum,RES .
LDH-3: lungs . LDH-4: kidneys, placenta, and pancreas . LDH-5: liver and
skeletal muscle

‫احد‬ ‫ا‬
LDH 1 in heart
LDH2 in Serum
‫احد ا تر‬ ‫احد‬ ‫ا تر‬ ‫ا ات‬ ‫ب‬ ‫ا ض ا ب‬ ‫اص‬
‫حا‬ ‫ا ر ضد د‬ ‫ا‬ ‫با رب ا ب‬ ‫بد‬ ‫ات ب‬
***Myocardial infraction .

99.Cutanuoes mycosis causes by➡ Microsporum - trichophyton -


epidermophyton
‫اجد‬ ‫ا راض‬ ‫ر ات بتسبب‬ ‫د ت ت‬

RBCs indices ‫ا‬ ‫ ب با سب‬.100

16
101.Acetyle co add to...... Substance to give citric acid in cytreic cycle
➡oxaloacetate

102.Virus causes burkitt lymphoma ➡ EBV Epstein-Barr virus

17
‫تست‬ ‫ا تر‬ ‫بذ‬ ‫ا ا رس دا د‬ ‫ا ب سبب‬ ‫ا راض ا‬ ‫احد‬
***The mononuclear spot test or monospot test, a form of the
heterophile antibody test, is a rapid test for infectious
mononuclosis due to Epstein Barr virus (EBV). It is an
improvement on the Paul-Bunnell test.
‫ا بسبب‬ EBV ‫ا ا رس ا‬ ‫ب دت ش‬ ‫ا ت ب د تست شا ا‬
(Paul-Bunnell test) ‫تست حس‬ ‫سس‬ ‫ا ش ش‬

103.Taq polymerase use in ➡ PCR

taq polymerase is ➡ enzyme create DNA by assembling


nucleotides
‫أ زا ب رب‬ ‫ت ش‬ ‫سا‬ ‫ا ب س ار ا‬ ‫است ا ا‬ ‫سا‬
‫اج ا س ا‬ ‫ ا‬DNA amplification ‫ا تا د‬
PCR

104.Target cell seen in ➡ thalassemia and in iron deficiency in


chronic case
‫ا ث ث ا ا تار ت د‬ ‫ز جدا‬ ‫با‬

105.steatorrhea ➡means fat in stool or fatty stool

‫اد‬ ‫ا تحا دا ص ح‬ ‫ر ز ا ا ص ح دا رات ب ج ب ا‬


‫ا ست‬

106.Turbidity estimated by ➡ Nephlometry


Nephelometry is a technique used in immunology to determine the levels of several blood
plasma proteins. For example the total levels of antibodies isotypes or classes: Immunoglobulin
M, Immunoglobulin G, and Immunoglobulin A

*Turbidimetry measure transmitted light.


**Nephlometery measure scatter light.

18
107.Increase growth hormone in adult cause ➡ acromegaly

108.increase growth hormone in children cause ➡ gigantism

‫ا اس ا بار ب س ب‬ ‫زد‬ ‫ا ا‬ ‫ب سبب ا‬ ‫ز اد ر ر ا‬


) ‫را‬ ‫ض (ض ا ة ا‬ ‫اات‬ ‫ا احد بب‬ ‫ا‬ ‫ا‬

109.Decreas growth hormone cause ➡ dwarfism

‫ا ز ا ا صر‬

110.parameter sensitive to light or photosensitive ➡ bilirubin

.‫ء‬ ‫ا‬ ‫ا‬ ‫سر ا احا‬ ‫ات ا ب شت‬ ‫ا‬ ‫د ا با ر ب‬ ‫شا‬


‫ت ا‬ ‫ح‬ ‫ا ت ت ح ت بتا ت ا بتا تس ز‬ ‫ش‬ ‫ ج‬.111

1- Vaccine ➡ HBs Ag & antiHBc -ve /antiHBs +ve

2-Active infection ➡ HBs Ag & antiHBc &Igm antiHBc +ve


/antiHBs ve
3-C c c ..HB A &a HBc + /a HB -ve

4-Resolved(immunization) HBV infection➡ HBs Ag -ve / antiHBc


& antiHBs +ve

112.To defernaite between sickle cell disease (HBSS) and sickle


cell trait (HBAS) ➡ use of Hb electrophoresis

‫ر سس‬ ‫تر‬ ‫ا‬ ‫شا ا در ا ر ب ا ب ات‬ ‫ر‬ ‫ا‬

113.gene consist of ➡Promoter /Etron/Intron.

‫ات ا ج‬ ‫د‬ ‫د برض تح‬

114.Decrease in all blood element ➡ called pancytopenia

115.Bacillary dysentery causes by ➡ shigella

19
116.Amoebic dysentery causes by ➡ Entamoeba histolytica

117.shigella on Xld agar ➡red colored or colourless without


indicator

**shigella on macconkey agar ➡colourless with small colony

118.Salmonella in XlD ➡ pink colour with black


center(ParatyphiB)

**salmonella on macconkey agar ➡ colourless or pale yellow

119.Gilbert's syndrome is ➡ autosomal dominant characterized


increased unconjugated bilirubin.
***C Na a d .D c in uridine diphospho
glucuronyltransferas

120-Reduce in line of wbcs cell➡called leukopenia

x linked autosomal dominant ‫ا ر‬ ‫ ا ز ا ر‬121.


**X- d .. a d XC .( : a)
**A a . a d S a cc .(22 c )
**A a D a O c a
cause Disease.
**A a c .T c c ac a ca
Disease.

**Autosomal dominmt➡50%

50 % ‫ت ر ث ا سا د‬ ‫سب‬ ‫اا‬

122.Result of negative ASO ➡ no agglutination there is no react


with antibody

123.Before take sample for patient ➡assure of patient

20
‫تتا د ا دا ا ب ش ا ر ض تش‬ ‫ا ج د ضب ا ج د ا‬ ‫ات ا ش‬ ‫ا‬ ‫احد‬
. ‫ا‬

124.Reticulocyte found in ➡Hemolytic anemia

‫ا‬ ‫ت ا‬ ‫ا‬ ‫ز جدا‬

125.Deficency of Vit B12and folate➡megaloblastic anemai

Pernicious anemia ‫بذ‬ ‫** د‬


Pernicious anemia is auto immune diseases ..Ab attack parital cell
if stomach (cell that responsible for production of intrinsic factor
(IF) which bind with vit B12 to facilitate absorption in jejunum)
so lead to deficiency of vit B12 .which lead eventually to
Megaloblastic anemia.

Auto immune disease ‫ا ا‬ ‫ر‬

126.Pateint with heprine tratment➡ptt prolonged

**Heparin treatment patients monitoring by PTT.

21
Warfarine ‫ج با را ر‬ ‫ ب در‬Prolonged pt ‫د‬ ‫ا اس ا ب‬
‫س ا‬ ‫س‬ ‫ش‬ ‫تج‬
**Warfarine treatment patients monitoring by PT.
127.Negler reaction positve in collesteridum perfringes
‫سترد‬ ‫ا‬ ‫ص‬ ‫ص‬ ‫تست‬

128.negri body ➡ is eosinophilic inclusion in nerves cell found in


rabies virus
‫د ا اس ا ب صااب ا ب داء ا س ر‬ ‫ا ا صب بت‬ ‫ا‬ ‫ش‬ ‫ا ر د‬

‫ا‬ ‫رب د‬ ‫د‬ ‫ر ض ا ربس‬ ‫ا ست د ا‬ ‫ةا‬ ‫ا‬ ‫ح‬

129.herpes virus ➡ tzanck cells

130.protein transport iron ➡ transferrine.


22
131.Gene responsible for familial hypercholesterolemia
(autosomal dominant)➡ LDL receptor

**mutation in LDLr protein which normally remove LDL(bad


cholesterol) from circulation.

‫بتا ا د ا ا ب بدا‬ ‫ا ا سترا د ا ا شر‬ ‫ جا ا س ا‬.132


‫ت ت‬ ‫ت ت حت ب رج‬ ‫س‬ ‫ب بدا س اتجا‬ ‫ا س ا ا برا ا تاب‬
‫سة‬
DNA double helix DNA prime
‫س‬ ‫ب ش‬3_5 ‫ تات‬5__3 ‫ا س‬ ‫بد‬ ‫د‬

133.Cryoprecite plasma contain factors ➡


fibrinogen..VIII..XIII..VWF
‫ب ا بر ج‬ ‫ات ا ر برس بت د ا تر ش‬ ‫د‬
‫ ت ت رات بتا ت‬centerfuge of plasma ‫ش ب‬ ‫ا ر برس بت د بار‬
‫ا ر برس بت د‬ ‫ا ا ترات د بس‬ ‫ا‬ deposition ‫د ر ا‬

134.Relapsinge fever causes by➡ Borrelia recurentti.

135.Harmful cholesterol ➡ LDL

136.useful cholesterol ➡ HDL

hdl ‫د‬ ‫ا‬ ‫ستر‬ ‫ ا‬ldl ‫ار ب ح‬ ‫ا‬ ‫ستر‬ ‫ا‬

137.Defect in VIT D causes ➡Rickets in children And


Osteoporosis
in adult
‫د ا اس ا بار ب سبب‬ ‫ا ر تس ا‬ ‫ا ب سبب ا ساح ا‬ ‫دا‬ ‫دا‬ ‫ت‬ ‫صا‬
‫شاش ا ا‬

138.Albunism ➡defect in enzyme tyrosinase

23
‫با رب ا د ا ح ر‬ ‫ا‬ ‫صب‬ ‫ب ش‬ ‫ا زا‬ ‫ا اس ا د‬

acid base balance ‫حا ت‬139

*Respiratory acidosis ➡ low ph+ high pco2 + normal Hco3

*Respiratory alkalosis ➡ high ph+low pco2+ normal Hco3

*Metabolic acidosis ➡ low ph+ normal pco2+low Hco3

*Metabolic alkalosis ➡ high ph+ normal pco2 + high Hco3

140.Double zone in media contain beef blood ➡ cl.perfrenges and


s.pyogen in anaerobic.
. ‫دب ز‬ ‫بتب‬ ‫ا‬ ‫ض‬ ‫ت‬ ‫سس بس‬ ‫ب تا‬ ‫بت‬ ‫*اص ا با ج‬
**141.Viral classification according to their Nucleic acids:
DNA viruses :-
1-Poxviridae
2-herpesvirdiae
3-adenovirdiae
4-hepadnavirdiae
5-papilomavirdiae
6-parvovirdiae
7-polyomvirdiae

RNA viruses :-

24
1-picornavirdiae
2-coronavirdiae
3-retrovirdiae
4-reovirdiae
5-orthomyxovirdiae
6-paramyxovirdiae
7-rhabdovirdiae

142.Type of G6PD deficiency anemia➡heamoltyic anemia

**Favism anemia is deficiency in ➡ enzyme G6 PD

RBCs ‫ا زا ذا دا‬ ‫د ت‬ ‫د‬ ‫ب‬ ‫اس ا ا ا‬ ‫د‬


143.excess antibody = prozone phenomena
144.excess antigene = postzone phenomena
‫ب ستاز‬ ‫ت ر بس‬ ‫ا تج‬ ‫برز‬ ‫ت ر بس‬ ‫ا ت ب د‬
‫حا ا را ش‬ ‫د‬

145.Women has Rh negative and baby positive ➡ give anti D for


women( during 72h after first delivery).
‫ا‬ ‫ا‬ HDN ‫حص‬ ‫ا ب حص ش حاجا ب ا‬ ‫ا‬
‫ح ات د‬ ‫ ب ا بد ا‬HDN ‫حص‬ ‫شا‬ ‫ش‬

146.How to preper 1%agar from agrose➡1 gram

100 ‫ را‬1 ‫ا ا‬ ‫ا ة ا تر ز ابتا‬ 1 ‫ا‬

147.heavy metal measure by ➡atomic absorption

148.selective media for bordetella ➡bordetella gengou agar

25
149.gout disease increase of ➡ uric acid

150.One gene one enzyme ➡ one gene one poly peptide

One gene one enzyme ‫شرح‬


‫د‬ ‫ا زا ا جا ا تا‬ ‫ج س‬ ‫ا اس ب دج ا ا‬ ‫ر‬ ‫د‬
‫اص‬ ‫ رج ا بر ت‬protin ‫اص ا ا‬ ‫ا ر ا ا ااا رج ا زا‬
‫ببتا د‬ ‫ب‬ ‫ج س‬ ‫ ب ت ا ر‬poly peptid ‫ا‬
**One gene one poly peptide ‫ر‬ ‫ب تد ات ر ا‬
151.I .1000
** 1 1000000

152.concentration of dntp in pcr ➡10 um

building blocks of DNA in pcr ‫ د ا‬dNTP


‫جرا‬ 10 ‫ا ب س ار‬ ‫دا ا تر ز ا بست د‬
‫ر‬ ‫اد‬ ‫ا تاج ا ا‬ ‫ا‬ ‫اسد ا ب د‬ ‫ا ا‬ ‫د‬153
‫ارب د ت‬

Ketogenic amino acid➡leucine and lysine(L a ac d)

**K ca ac d K b d .
**G c ca ac d ..G c .

154.Media contain antibiotic ➡Thayer-Martin agar

155.Gram stain give false result ➡ due to over-decolorization

156.Syphilis causes by ➡treponema pallidum

157.Iso enzyme elevated in myocardial infarction ➡


CKmb/AST/LDH

26
158.Ammonia transfer from tissue to liver as ➡ alanine

‫اب‬ ‫ر ا تت‬ ‫ا بد شا تتح‬ ‫ا‬ ‫ااب‬

159.Early fasting glucose obtained from ➡ Hepatic


Glycogenolysis.
‫ا‬ ‫ز‬ ‫ا‬ ‫ج‬ ‫ا بد بت سر ا ج‬ ‫حا ا ص ا ا ص ر ا ا بتج ا‬

**Prolonged fasting or in starvation obtain glucose from ➡


lipolysis
‫ات‬ ‫ز ح‬ ‫ا‬ ‫ج‬ ‫اج‬ ‫ا تاج ا ا‬ ‫اد‬ ‫ا جس بتج‬ ‫ا‬
‫ت‬ ‫ج‬ ‫بتا ت ت س ر ا ج‬ ‫اسا‬

Glycolysis➡break down of glucose to obtain ATP**

Glycogensis➡synthesis of glycogen from glucose

glycogenolysis➡break down of glycogen to obtain glucose

Gluconeogensis ➡formation of glucose from non CHO source

lipolysis➡formation of glucose from lipid

160.Cystic fibrosis ➡ is autosomal recessive diseases

‫رض ت ح‬ ‫راث ب ص ب ا ر‬ ‫ا‬ ‫رض ج‬

161.Function of neutrophil ➡phagocytosis or granule circulation

162.FSH / LH hormon ➡ is Glycoprotein hormones

‫ر‬ ‫بر ت‬ ‫ج‬ ‫بار‬ ‫ب ت‬ ‫د‬

163.osmotic fragility test ➡to detect hereditary spherocytosis

Heamoltytic anemia ‫ا س ر شا ت سس ا ا تب‬ ‫تست بست د ا‬

164.Cold antibody ➡Igm1

27
145.Warm antibody ➡ Igg

146.Buffer consist of ➡ weak acid + weak base or alkaline

147.Systemic lupus erythematosus (SlE) disease diagnosis by ➡


ANA antinuclear antibody test

148.Grow in charcoal ➡ legionell

Bacteria in cat bite ➡ pasteurella(Yersinia pestis)

.‫ب‬ ‫ا دس ا‬ ‫ت‬ ‫جرح‬ ‫ا ب تر ا ا ب ا ا‬ ‫د‬


high HBf ‫ا‬ ‫ا‬ low CBC ‫ا‬ ‫سبش‬ ‫ ا ا‬149
‫ ا ا‬F ‫ب‬ Major ‫ا ا‬ ‫حددا‬ Thalssemia ‫حت‬ ‫اا‬
‫شد د‬

150.heinz body ➡ rement of denaturation of Hb

‫ا اد‬ ‫س ح‬ ‫تب‬ ‫ب‬ ‫باا‬ ‫ا‬ ‫ش‬ ‫جاب‬ ‫دا ت ر‬


‫ب ا ا‬ ‫ا‬ ‫ا ح راء‬In heamolytic anemia maily G6 PD And
meglaoblastic anemia

151.Diabetic patient treated by insulin should be monitored ➡ k

‫ا شا‬ ‫ا ب تاس‬ ‫تاب‬ ‫سر ب‬ ‫د‬ ‫ا اس ا ت ا ج با س‬


Hypokalemia ‫ب ا‬ ‫ا ا ب تاس‬ ‫بد‬

152.lactose fermenter ( red to pink ) e.g : E.coli , Klebsilla ,


Enterobacter
153.Non lactose fermenter (colorless) e.g proteus , salmonella ,
shigella

154.organ depend on glucose as fuel ➡brain

28
155.anemia with thrombocytopenia➡ aplastic&megaloblastic
anemia
‫صصاح‬ ‫ب‬ ‫ا ت‬ ‫ت‬ ‫د ا‬

156.Neutrophil with two lobe ➡ pulger huet anomaly

hyposegmented neutrophil ‫ب‬ ‫رض راث ب ص ب ا تر‬

157.Type of infection that igg present ➡ secondery infection

158.Type of infection that igm present ➡ primary infection

igg ‫بت‬ ‫ ا صاب ا تا‬igm ‫ب‬ ‫ا صاب ا‬

159.Amount of blood enter kidney➡1200__1500 ml/ min

160.Lamb of spectrophotometer ➡tungsten lamp

‫ة‬ ‫دا ا تصارات س اء اح اض ا‬161


Phenylalanine aspartate arginine ?
Phenylalanine=P /aspartate=D /arginine=R

162.Increase absorption of iron ➡Vit C. high TIBC .HCL .

163.Decrease absorption of iron ➡ low TIBC .

ferric state of iron. Tannic acid,low Hcl


‫ا تصاص‬ ‫ا تصاص ا حد د ت‬ ‫ا بتز د‬ ‫ا‬ ‫ا‬ ‫د‬

164.Hashimoto and G a disease antibody against ➡TSH

TSH ‫ب ض جا ب‬ ‫دا ج‬ ‫د ا ت ب د بس اس ا‬ ‫رض‬ ‫ا‬

Causes HDN ➡ igg antiD

165.Pancrietic enzyme➡lipase and amaylase


29
**Acute pancreitites diagnosis by➡lipase

Salivery gland ‫ب رز‬ ‫ اسبس‬Amaylase

166.Hormone responsible for water absorption ➡ADH antidiuretic


hormone

167.Hemophilia A defect in ➡ factor VIII

178.Hemophilia B ➡ defect in factor IX

179.Cortisol in urine ➡ free cortisol or unbound cortisol in


plasma

180.Candida can stain with ➡gram stain give blue colour

181.Hb function ➡carry O2 from lunge to tissue

182.Non glucose ferment ➡pseudomonas

**.All enterobacteriaceae ferment glucose except pseudomonas


oxidizing of glucose

183.closeness of the measure value to true value ➡ high accuracy

‫صح ح‬ ‫بت‬ ‫اتا تج ح‬

184.Enzyme effect with alcohol ➡ ggt gamma glutamate transferase

185.Study of chromosome and gene➡ cytogenetic

186.Study of genetic ➡ study of inheritance.

187.Hormone control adrenal gland ➡ACTH

‫ا د‬ Pituitary gland ‫ا‬ ‫ا ب ت رز‬ ‫دا ا ر‬


188.Pituitary gland called master gland

189.Most abundant hb in infants ➡ HBf

30
Hb A ‫ش ر ب تح‬ ‫ب د ا سب‬ ‫ا‬ ‫ب‬ ‫ا بدا ب‬

190.vibrio on TCBS yellow colonies due to ➡ fermentation of


sucrose
191.the most important to check in blood bag in case of
transfusion➡ expire date .

192.the most important to check in blood bag in case of donation


➡anticoagulant .

‫ت‬ ‫ا‬ ‫ا ت‬ ‫تبر ا د ب ح‬ ‫اج اش‬ ‫ا‬


‫ا سب ر ا تار ا ت ا ا د‬ ‫تبر ب ح‬ ‫اج اد ا د ا ش ت ا‬
‫ا ت ا ا ست د‬ ‫حسب ا ت‬ ‫سحب ا ب احسب ا ا‬

193.virus cause gastritis ➡ rotavirus or enteroviruses

194.Blast in acute leukemia ➡ more than 20 % blast

195.positive in complement test ➡no hemolysis

**Negative in complement test ➡ hemolysis

196.test before transfusion ➡IAT indirect antiglobulin test

‫ رت ا س ا‬197
Heamolysis ‫ارات‬ ‫ا‬ ‫ض‬ ‫اص احاد‬ ‫ب ص ا‬ ‫سبب ا‬ ‫اتب‬
‫ا‬ ‫جا‬ ‫ا ا‬ ‫ا ص ا ح ز ا اس ا بت ا ج با ش‬ ‫بت ص‬

197.Vires found in stool ➡ polo vires

‫ا‬ ‫ب ا دا ا ا رس ا بسبب ش ا‬

31
198.donar who is donate 500 ml of blood why platate can not be
transfused to other patient ➡ platelets don't separated by
apheresis
‫ب در د‬ ‫بث ا‬ ‫تس‬ ‫ز‬ ‫ د‬500 ‫اتبر ب‬ ‫ا ز‬ ‫ب‬
‫؟ ا ا ا سبب ا ر سس‬ ‫ب ش تاات ب سا ا سبب ش‬ ‫اص احاد‬
Aphoresis ‫اا‬ ‫صص‬ ‫ا ب ت ت ا ص ت با ج از ا‬

199.Dark microscopic can be diagnosis ➡ syphilis

treponema pallidum ‫ب‬ ‫اش‬

200.KalaAzar or vesral leishmaniasis ➡causes by leishmania


donovani

201.oriental sore ➡caused by leishmania tropica

202.heavy metal measure by ➡atomic absorption

203.function of epinephrine ➡fight and flight

‫دا‬ ‫ا ر‬ ‫ا ب رت‬ ‫حا‬ ‫ب ر‬ ‫ا ر ا ر‬ ‫دا ر‬

204.Parasite form of crescent shaped ring ➡plasmodium


falciparum

205.Step of blood collection ➡ select vein.. Torniqate .. clean with


alcohol.. Then punctur

206.Normocytic normochromic anemia ➡aplastic


anemia/Hemolytic anaemia

Temp of complement➡37 C

207.zoonotic diseases ➡yersinia

‫ا اس ر‬ ‫بر س‬ ‫رض ا ا‬ ‫اح ا ز‬ ‫ا راض ا بتص ب ا سا‬

208.Function of fluoride oxalate ➡inhibition of glycolysis


32
**anticoagulant of glucose.
‫د ا را‬ ‫ز‬ ‫ت سراج‬ ‫بح‬

209.Function of sodium dodocyl in electrophoresis gel ➡


denaturation of proteins.

210.Sterlization in hot air oven in➡160 C for 30 min

211.water through pores ➡ ultrafiltration

212.electrolyte measurement by ➡flame photometer or ion


selective electrode

213. natural killer cell / T.cytotoxic called cytotoxic cell ➡


produce toxin to kill foreign body
‫تت ا‬ ‫ا‬ ‫اد س ة دا‬ ‫ا راز‬ ‫ر‬ ‫ا‬ ‫ش‬ ‫ا ت‬

214.Liver fluke cause by ➡fasciola hepatica

215.In direct Elisa to detect ➡ antibody

216.Direct Elisa to detect ➡ antigen

217.Universal donor ➡ O negative

218.Universal recipient ➡ AB+ve

219.Most of crystal in renal stone ➡ ca oxalate

220.Parasite in urin with terminal spine ➡schistosoma


haematobium

221.Most WBC cell 40% _75% has(3-5 lobes) ➡neutrophil

222.B. anthrax cause ➡wool sorter disease a type of pneumonia

‫ا ب تر ا‬ ‫ا ح ا ات ب ت اد‬ ‫ص‬ ‫ا‬ ‫ا اس ا ب شت‬ ‫دا ا ت اب بج‬


‫ر ا ست شا‬
33
223.Immunoglobulin in body fluid ➡ IgA

‫ ا ا س رش تا‬Urine tear milk ‫ب ا‬

224.reticulocyte count normal range ➡ less than 2 %

225,Rice watery diarohea➡causes by vibrio cholorei

226.Stem cell is first cell produce RBCs WBS megakaryocyte


.‫ا‬ ‫ا‬ ‫ا بتبدا تث با‬ ‫ا‬ ‫ا‬
‫ا ا‬ ‫ا‬

227.PTprolonged➡extrensic path way+common pathway

228.PTT prolonged➡interensic path way+common pathway

**VitK dependent factors...X...VII..IX..II (1972)


**V K d d C/S

**Exrensic pathway➡Factor VII..X..V..I..II

**Intrnsic pathway➡VIII..IX..XI..XII..X..V..I..II

***C a a X V II I

34
229.tumor marker of hepatocellular carcinoma ➡alpha
fetoprotein

230.croup disease causes by ➡ parainfluenza virus

‫ا حصا‬ ‫ص ت از ص‬ ‫ب‬ ‫از‬ ‫ا دا رض ا ا‬

231.hypocalcemia ➡ less than 7,5 mg/ dk

HBA1C ‫تا ج‬ ‫بتا‬ ‫جد‬ ‫ا ر ش حارس‬232

35
233.wright stain ➡ can use for WBCs deferential

234.whooping cough caused ▶bordetella pertussis

‫اس ا اد‬

235.Un flexible immunoglobulin ➡ Ige

236.Immunoglobulin againstABO blood group ➡ Igm

236.Immunoglobulin against Rhesus system ➡ Igg

237.ALT➡ specific for liver only

238.AST➡ specific for liver and heart

239.ALP➡ specific for liver and bone

240.Baby has HDN give➡ Fresh blood RH _ ve less than 7 days

‫؟؟؟‬ ‫ش‬ ‫اتش د ا ا ر ض اد‬ ‫ت حاص‬ ‫ا بب ا‬


Fresh blood RH _ ve less than 7 days

241.Patient with gastroctomy ➡ causes deficiency of B12 vitamin


lead to megaloblastic anemia
242.Accreditation is non governmental agency that concerning
with evaluation of the labs and give certification
Accreditation
. ‫تص با ج د‬ ‫رح‬ ‫ا ت‬ ‫صدر ا‬ ‫ا‬ ‫ت اد ا ب د ا‬ ‫د ش اد ا‬

36
243.Spectrophotometer ➡ measure the light.. The light
proportional directional to concentration

244.principle of flame photometer ➡By the thermal energy of the


flame, the atoms get excited and there after return to ground
state.

245.flurometer▶absorbed short wavelength and emit long


wavelength

246.electrophoresis ▶ depend on charge and molecuuler weight

247.nephelometer principle ▶ measure scatter light

‫د ا بر س ب بتا ت ا ج ز‬

gametocyte of plasmodium
falciparum

37
The stages of gametocyte development in Plasmodium falciparum.

38
39
40
41
42
43
Paracoccidioidomycosis

44
45
46
mycobacterium tuberculosis acid fast stain

Mycobacterium tuberculosis visualization using the Ziehl–Neelsen stain.

47
Streptococcus pyogenes . Bl d aga h i g -hemolysis

Beta-hemolysis on blood agar (Streptococcus pyogenes)

48
Alpha-hemolysis on blood agar (Streptococcus pneumoniae)

Anaerobic culture of
Clostridium perfringens on blood agar. The characteristic double zone of
clear beta-hemolysis around a colony is clearly seen (arrow).

49
Anaerobic (double z one of beta hemolys is )

Egg of E. vermicularis in an iodine-


stained wet mount from a formalin concentrate.

Eggs of E. vermicularis in a wet


mount.

50
larvae of strongyloides
stercoralis

51
babesia

babesia

What is Babesia?
52
‫‪A: Human babesiosis, more commonly known as babesia is a rare, blood-borne‬‬
‫‪disease which is transmitted by ticks and found in both North‬‬
‫‪America and Europe. Babesia is caused by‬‬
‫‪the Babesia microti and Babesia divergens parasites, both of which are members‬‬
‫‪of the protozoan kingdom.‬‬
‫ا ح ا ات‬ ‫ر ا راد ص ب ج ة ب رة‬ ‫رض ت‬ ‫ا باب ز ا بارة‬ ‫د‬
‫رات ا د‬ ‫ح د ا ة ش دا‬ ‫أح ا ادرة د ص ب ا سا ‪ ،‬سبب ا‬ ‫ا ة ا بر ة‬
‫ا ح راء ج س باب ز ا‪.‬‬
‫حشرات‬ ‫ا س‬ ‫ا ت ز ا ج را‬ ‫ا ذ تح‬ ‫ا ا ا ا‬ ‫ا رض اس ا تشار‬
‫ا ا ة‪.‬‬
‫ذ‬ ‫ا باب ز ا‬ ‫ا ثا ر ا ا ذ شتر‬ ‫ا باب ز ا شد د ا رتبا ب رات ا د ا ح راء‬
‫ا ص ح ا ب ر ب ز ا ‪.Piroplasms‬‬ ‫ا اص ة‬
‫‪Host‬‬ ‫ا باب ز ا ت صصة ا ا‬ ‫أ ا ا ا ت ة‬ ‫ا ا دة ا ا ة‬
‫‪B.‬‬ ‫ذ‬ ‫ا ارض‬ ‫د تص ب ا سا‬ ‫‪ B. microti‬ا ت‬ ‫‪ Specific‬باستث اء‬
‫‪ divergens‬ا ت د تص ب ا ب ار ا سا ا جرب ‪.‬‬
‫ا سبب ا راض ا ح ا ات‬ ‫‪1‬‬ ‫ب‬ ‫‪،‬‬ ‫ا باب ز ا ر ا‬ ‫ا‬ ‫‪71‬‬ ‫ا أ ثر‬
‫ا ستأ سة‪.‬‬
‫ا ب ار إ أ ا صابات ب ا‬ ‫أ ا سا ر ا تصاد ة ا ت سبب ا ا رض تتر ز‬ ‫ار‬
‫ب ض ا ا ‪ .‬ا باب ز ا ادرا ا ت‬ ‫ا ا ا ا ز ا از ر ا ب ا أ ة ت ا تة‬
‫ش اص ذ ا ا ة ا ثب ة ‪Immuno Suppressed‬‬ ‫رض ة سا تسبب رض‬
‫رج ز‪ ،‬باب ز ا ب س‪ ،‬باب ز ا‬ ‫باب ز ا دا‬ ‫‪ Persons‬أ ا ا باب ز ا ا ت د تص ب ا سا‬
‫ا ز‪ ،‬باب ز ا ا ا‬

‫‪S taphylococcus Aureus‬‬

‫‪53‬‬
‫‪Cases:‬‬
‫تجا ب‬ ‫ا تاح ش‬ ‫تش‬ ‫** ز ت ر ا س برااااح‬
‫ر ض بث تاح ا‬ ‫سط‬ ‫حج‬
‫‪1.Patient has cat bite➡ Pasturella‬‬
‫ضت ا د س ات ب تر ا‬ ‫ار‬ ‫د‬ ‫د‬ ‫د د‬ ‫بش‬ ‫ح‬
‫ا بستر‬ ‫سببت د‬ ‫حت‬

‫‪CBC .‬‬ ‫ا‬ ‫‪ 2.‬س ا ب ش‬


‫د ‪Low Hb‬‬ ‫ا‬
‫‪HBF‬‬ ‫ا تاح‬
‫‪thalassemia‬‬ ‫ح تار ا‬ ‫ط اا‬
‫ش‬ ‫‪Type of Hb Hbf‬ا‬
‫ر‬ ‫ا ث س ا جر‬ ‫ا ر‬ ‫طب‬

‫‪54‬‬
‫د‬ ‫ز‬ ‫ة‬ ‫راض‬ ‫ا ر ا با ً ا‬ ‫راض برض ا‬ ‫با‬

‫‪55‬‬
‫ا‬ Alcoholic pateint ‫ ا ا‬3.

Cultur of sputum And stain with weak acid fast bacilli➡


nocardia
Waek acid fast ‫ارد ا‬ ‫ت تار‬ ‫ط ا‬
Weak acid bacilli ‫ا س دا‬ ‫ا تاح‬

4.patient with UTI lab culture gram negative bacilli and motile
indole positive and lactose ferment ➡ E coli
‫ا تستات‬ ‫با‬ ‫تا‬ ‫ ا د‬UTI ‫تاح‬

5.Peptic ulcer/urease positive/grow in chocalte➡ h .pylori


‫را‬ ‫با‬ ‫ت ا رح ط ا‬ ‫ط ا‬

6.Septecemia ➡ sallmonella
‫سبتس ا‬ ‫ا ب تر ات ا بت‬ ‫احد‬
‫تس‬ ‫ حص‬eggOr chicken ‫ث‬ ‫سا بش ا‬ ‫ب‬

56
‫ا‪spleenoactony‬‬ ‫ا‬ ‫د ا ‪ITP‬‬ ‫را جات‬ ‫ا‬ ‫…‪7‬دا س ب‬
‫اش احس ؟؟‬ ‫ش ا ا ر د‬ ‫د‬ ‫ار‬ ‫ارات ا ا ا ض‬ ‫د‬
‫ا جاب ا ا ار‬ ‫ا‬
‫‪patient count of platelet 170000 on corticosteroid‬‬
‫ا ا ب ش اش تحس ا ص ائ زادت ب د ا دت ا ج‬ ‫ب‬
‫ا صاح‬ ‫ت ب با ر‬ ‫س‬ ‫ا س دا‬ ‫ا تب ا‬
‫‪ITP➡ idiopathic thrombocytopenic purpura‬‬
‫اصاح‬ ‫ا ت ب د ضد ا ص ائح ا د‬ ‫ا ج از ا ا ح ا ب‬
‫ازا‬ ‫ا‬ ‫ا رتب با ت ب د ب ت ازا ت ا ب اس ا حا ا ا‬
‫اش تحس‬ ‫ا ارات ا ب ر ا ا ر د‬ ‫اسب ‪ .‬ا ا احس ش‬
‫؟‬ ‫ش‬
‫‪170‬‬ ‫ز اد ب ت ا تر‬ ‫اش‬ ‫ا سب ار ا ت ت ا ص ائح بتا ت‬
‫ا ب د ا دت ا ج ‪.‬‬

‫‪8.sample culture in charcoal media ➡ legionella‬‬


‫داا ح‬ ‫زرا‬ ‫ا‬ ‫ا‬ ‫د ا ت اب ا‬ ‫ط ب ادا ا ب ش‬
‫‪.‬‬ ‫د ط اا‬
‫‪9.Patient take chohol‬‬
‫ت ا ‪Liver damge‬‬ ‫د‬ ‫ا ا ح‬ ‫ش‬ ‫از‬
‫طب ا ‪ALT‬‬ ‫‪liver damge‬‬ ‫ا‬ ‫د‬ ‫د‬ ‫ا‬ ‫ات ا زا ب ا‬

‫‪57‬‬
‫از ر‬ ‫دتا ا ح‬ ‫ا ز ا برت‬ ‫حا ة سئ‬ :‫ح ة‬
.GGT ‫ا حا ة ر ضة ب‬ ‫بد‬ ‫ح‬ ‫ات ا‬

‫ا‬ ‫ا بد تح‬ ‫ج‬ ‫اج‬ ‫ةت ز‬ ‫ا س ا ا س ر ا ائض ب د‬.10


Fatty acid.

CSF culture in chocolate agar


Low glucose high protein ‫ا ا‬ ‫ا ت تا ج ا‬
‫زر ا ا‬ ‫ب ت ر ا طا ا ا‬ meningitis ‫ا سبب ا‬ ‫د ط ا‬
H influenza ‫ش ت حت‬
meningited ‫بتسبب‬ ‫ر‬ ‫دا‬ ‫ا‬ ‫بت‬ ‫ا د‬

11.Pateint has cardic surgry


‫ا ا ب تر ا د‬ ‫ ا‬UTI ‫ا احت ا‬ ‫ا ت اب ا‬ ‫جا‬
Resist of vacomicine and panciline
: ‫ ط ا‬Grame positive cocci
Enterococci
‫ب زت‬ ‫ت‬ ‫را‬ ‫ط‬ ‫ت ب‬ ‫ا ر‬
All G+ve bacteria are vancomycin sensitive except VRSA
(Vancomycin Resistant Staphylococcus aureus) and VRE
(Vancomycin Resistant Enterococci )

58
All G-ve bacteria are vancomycin resistant innately.

12.Sample culture in anaerobic condition?


Eye swab )swab ‫(بص ة ا ة‬
swab ‫اس ا‬ ‫ ا ا‬pleural fluid ‫اس ا‬

‫ة ش ؟؟؟؟‬ ‫ ا ب ت ر ا ا ت‬synovial fluid ‫ة‬ ‫ س ا ش ا‬13.


strepto pyogene ‫ا ت اب اص‬ ‫ا بت‬

‫ا‬ ‫ا‬ ‫ا ر‬... ‫ا ر‬ ‫ز‬ ‫ ر سج‬3 ‫د‬ ‫ ا ا ز‬14.


‫ ) ش ؟‬glucoseuria) ‫دا‬
Glucagon hormone
hyperglycemic hormones ‫اج‬ ‫اج‬
**Insulin is hypoglycemic hormone.
**Glucagon is hyperglycemic hormone.

15.Patient with otitis media bacteria that causes ➡ S.aureus


.‫ا رس‬ ‫ش استا‬ ‫اب تراا ت‬ ‫س‬ ‫د ا ت اب اذ‬ ‫بش‬

59
16.Bacterial infection has oxidase positive➡ vibrio
Psedumonas – Nesseria - H influenzi
‫ا س‬ ‫حسب ا ار ا بج‬ ‫ب‬ ‫تا‬ ‫ا س د س ب زت‬ ‫د‬
17.Hormone regulation of Na K and water➡ aldosterone
hormone

18.ALT and AST ALP raised

GGT slight rise ➡ chronic hepatitis

Has blood in urine ‫ا ب د ا ا ب ش‬.19


‫ش ؟؟‬ ‫ ا ت‬stone ‫ ا ا‬ultra sound ‫ا‬
schistosoma haematobium ‫د‬ ‫ا ح‬

Urine general And ultra sound ‫ا‬ ‫اب د ا بش‬ .20


Crystal like coffin lid ‫د‬ ‫ا‬
Mg phosphate ‫ا ا تاب ت‬ ‫ر ستا تشب‬

21.Post hepatic hyperbilirubinemia ➡ pancreatic cancer


.‫ا ب ر اس‬ ‫سرطا‬ ‫د‬ ‫اسباب ا ر ا ا سداد ا اس ا ب‬ ‫احد‬

60
‫ا سبب‬... ‫ا ة س اد‬ ‫ا د اش ا‬ ulcer ‫د‬ ‫ ر ض س ر‬-22
‫ش ؟‬
Have gas gangrene caused by Cl.perfringens
23-Neutrophil with Two segmented nucleus or
hyposegmented neutrophil? pelger huet anomaly

24-Watery diarrhea ➡ shigella.

25-Rice watery diarrhea ➡ vibrio cholerae

26.Women all parameters low➡ diluted sample


‫ا‬ ‫ا‬ ‫ت با اد ا درب‬ ‫ار‬ ‫ا س دا ب ا ا ر جات ا‬
low ‫ا ح صات ط ت‬
‫ا ا درب‬ ‫ا دا‬ ‫شا ا ا‬ ‫ا‬ ‫حص ت‬ ‫ا سبب ش‬
61
27.Children or boy in urine glucose given ++++ by depstic ➡
renal threshold Or tubular dysfunction
‫د ارب‬ ‫ا‬ ‫ج ز ب ا ستر ب‬ ‫ا‬ ‫ا س دا ط ا د‬
‫ات ب‬ ‫ص ب س ر ا د طب بس ا ر ا ثر ش د ا ب‬
‫ا بت تص ت ا س ر‬

‫ت‬ ‫ا س دا ب‬.28
Women present with fatiga and headic
hb 8 mg/ dl- McV= 85 -Retic = 10% ‫د ا‬ ‫ا‬ ‫صات‬ ‫اا‬ ‫ا‬
‫ا ت ا ارات‬ ‫ب ش‬ ‫بست د‬ ‫ا ح صا د‬ ‫ات تست‬
Coom test
Sickling test
Hb ekectrophoresis
Serum bilrubin
S iron
‫ا جاب ا صاح طب ا‬ ‫ا ا ا صاح ش ت ت ر‬
Serum iron
‫شد د‬ 8 ‫ا‬ ‫رب‬ ‫ا‬
‫ا صاح‬ ‫ت ا‬ ‫ا‬ ‫شر‬ ‫ا ت‬ ‫د‬10 ‫رت ز‬
62
‫احد احد‬ ‫س‬
‫ا ا ب ش دا د ا ت ا ت ب د‬ ‫شا ا ر‬ ‫ا‬
‫ص ا تش ب‬ ‫ا‬ ‫سس سر‬
‫ب سط‬ ‫حت ا سج‬ ‫ا‬ ‫اس‬ ‫ت ا ا ز‬ ‫ا‬ ‫س‬ ‫اس‬
‫سس‬ ‫حص‬ ‫برض‬ ‫ا‬ ‫تت سر‬
‫س‬ ‫اا ا‬ ‫ت ا‬ ‫ا ا ا‬ ‫ا تر ر سس ا شا ا در ا ر ب‬
‫س ا س س ا‬
‫بر ض‬ ‫ا س ر با ر ب‬
‫ا‬ ‫ا ‪ Rbcs‬تت سر‬ ‫ب رت‬ ‫سس‬ ‫ا‬ ‫ر‬
‫‪S Iron‬‬ ‫ش‬ ‫ا تست ا احست د‬ ‫ب‬

‫ا ح صات د‬ ‫ا‬ ‫بش‬ ‫‪29‬دا س‬


‫‪ALP 50‬‬ ‫‪- Ca 10 - ALT 25‬‬
‫را‬ ‫ا د‬ ‫با ا‬
‫تار ‪thyroiditis‬‬ ‫ب‬ ‫‪Bone‬برض طب‬ ‫‪ liver‬طب‬
‫اد‬ ‫ا ت اب د‬
‫ا س‪.‬‬ ‫راض ا د ا ب ش‬ ‫ا‬

‫‪30.High CKmb➡ in myocardial infaction.‬‬

‫‪63‬‬
‫د س ر ا ش صا‬ ‫ ط‬Fastinge ‫ا‬ ‫ ا ا ب ش‬.31
Dibetic
‫ا تش ص ش طب ا حا تار‬ result ‫ا ض‬
More than 126 mg/ dl
Fastinge normal 75__110 mg) dl

‫اش‬ ‫ بس ا ا بس‬renal threshold ‫ س س ب بتا ا د ا د‬.32


Renal glucose urea ‫ا‬

33.Hypernatremia + hypokalemia➡ conn's disease

‫از ا ب‬ sickle cell anemia ‫ا ط ا‬ ‫ا‬ Hb .s34


Hb s. ‫ا د‬

35.medullary thyroid carcinoma ➡ calcitonin


‫اش‬ ‫دا ر ثا ر د ارس‬ ‫ر ار ر‬ ‫ت‬
calcitonin or thyroglobuline ‫بت‬

36.Hiv patient meningitis ➡


‫ ط ت‬Indian ink ‫ ب ا‬csf ‫صب‬ ‫دا ب ش ت د اتش ا‬
cryptococcus neoformans ‫ب زت د ط ا‬
Indian ink in case of capsule:

64
Bacteria. : Klbsela bromine--Fungi: cryptococcus neoformance
‫سحا ا؟‬ ‫ ت‬hiv ‫ا د‬ ‫ز‬ ‫ر ات بتج‬ ‫د‬
‫ر ات اذا ب ت ر ا ا ج اب بس‬ ‫د‬

37.Hypernatremia + hypokalemia ➡ diabetes


. ‫حا ة ا س ر‬ ‫ب‬ ‫د‬

‫راس‬ ‫ حسا ب ط ا‬donation , ‫اث اء ا‬ ‫جا تبر با د‬ ‫ز‬ ‫ا‬


‫ش‬ ‫ر ضا‬
38.Remove needle and stop donation

‫ت‬ ‫اج‬ ‫ا ت‬ ‫ش ب ا ش ؟ تبر ب ش‬ ‫جا تبر با د ا‬


.‫ا س‬ ‫تر ا ص ح ة د ا ج د‬ ‫حد ز د بش‬
39.patient with protein in urine and he has multiple myeloma
➡ ‫ا بر ت ش‬ bence jones protein

40.Sputum sample cultured in chocolate agar under


microscope appear filaments ➡h.influenzae
‫د ا ا داب‬ ‫ت‬ ‫ا تش‬ ‫بت‬
41.Hemolytic uremic syndrome caused by ➡ E.coli 0157
Note ; 0157 = serotype
65
42.Patient with jaundice
bilirubin ‫ا‬ ‫ارت ا‬ ‫اد‬ ‫سس‬ ‫حص‬
blood film ‫ا‬ CBC ‫ا‬ ‫ا‬
‫ش‬possible diagnosis ‫ا‬ microcytic hypochromic ‫ط ا‬
thalassemia ‫ا ج اب‬

‫ت‬ ‫ب ا‬Ketone in urine. 43


Fasting or starvation: such as with anorexia (an eating disorder)
High protein or low carbohydrate diet

Vomiting over a long period (such as during early pregnancy)

Acute or severe illnesses, such as sepsis or burns

High fevers

The thyroid gland making too much thyroid hormone (hyperthyroidism)


Nursing a baby, if the mother does not eat and drink enough

diabetic ketoacidosis (DKA).

44.B hemolytic catalase +ve coagulase + ve grame +ve in


cluster➡ staph aureus

45.Amebic dysentery ➡ entamoeba histolytica

46.Catalase_ ve and b hemolytic ➡ s.pyogen/s.group B

47.Toxoplasma ➡ protozoa
66
48.Sputum high neutrophil and high pus ➡ h.influenzae

49.B .hemolytic bacitracin sensitive ➡ s.pyogen

GGT = ‫ا‬ ‫ات ا زا‬ ‫ح‬ ‫ارئ شارب ا ثا‬ ‫جا ا‬ ‫ط‬ ‫ ا‬.50
51-Women has ALP 69 CA 18
hyperparathyroidism ‫ط ا‬ ‫اس‬ ‫د ا ا‬ ‫د‬

52.Eosinophilia➡ in skin disease and parasitic infection.

53.Endocarditis➡ staph aureus

54.Patient with Na k urea cl all high why ➡High intake of


protein
‫ا بر ت‬ ‫جر ات ب رة‬ ‫تا‬ ‫د‬ ‫ا ا‬ ‫ا حاجات د‬ ‫ز‬
55.Male has enlargement of breast and prostat??
‫ا سر‬ ‫د بر ستات‬ ‫ت دا‬ ‫شا‬ ‫ش‬ ‫حت‬
prostatic specific antigen PSA ‫ا ج اب‬
‫ا بر ست اد‬ ‫ج تض‬ ‫ا سر‬ ‫د بر ستات‬ ‫دا‬ ‫از‬

56.Tonsillitis caused by➡ s.pyogen


‫ش ؟ ا ت اب ا ز‬ ‫ا ت س تس ا‬

‫ برض‬high bilirubin ‫رة جاب ا د ا اص رار شد د د ا‬ ‫ ا‬.57


hepatic jaundice ‫د ا ش‬ ‫د‬ high liver enzymes

67
58.Patient eat full meal we do blood sugar for him after 2 hr
‫ا ر ضاس ر‬
‫ز‬ ‫حص ب د ج‬ ‫جا ب د سا ت‬ ‫ت ااا‬ ‫ا‬ ‫ا‬ ‫ا سؤا دا ز‬
**2hpp= less than 140mg/dl
**140-199 =GTT
**more than 200mg/dl=DM
‫ا رس‬ ‫ا ستا‬ ‫ ا بت‬Scalded skin syndrome. 59
skin scrub ‫ش‬ ‫ا حت‬ ‫ا‬
‫اس ا بار ا طر ح ا راش‬ ‫ت رحات ا سر را بتج‬ ‫ا‬
60.Hyperparathyroidism ➡ hypercalcemia
‫ا ا‬ ‫با ا س‬ ‫ا ا بربراثا ر د ز رتب‬
‫د اد ش‬ ‫ حتاج حدت‬A2 ‫رضد‬ ‫ ا‬.61
A2 or o_ve ‫حاد‬

62.Glucose high and cortisol high ➡ hypercortisolemia

63.ACTH high and Cortisol high ➡ cushing syndrome

64.Baby with high total WBCs neutrophil high➡ normal


conditions (In Neonate)

68
65.HIV+ meningitis ➡ tuberculosis mycobacterium
hiv ‫د اس ا‬ ‫ا تس‬ ‫ا‬ ‫با تر بت‬ ‫ا ا‬
‫ر ا سؤا د‬ ‫د سؤا‬ ‫ا تب‬
Hiv patient meningitis ➡

cryptococcus neoformans ‫د ط ا‬ ‫ ط ت ب زت‬Indian ink ‫ ب ا‬csf ‫صب‬ ‫د اتش ا‬ ‫دا ب ش ت‬

Indian ink in case of capsule:

Bacteria. : Klbsela bromine--Fungi: cryptococcus neoformance

66.UTI caused by➡ staph.saprophyticus

67.Hyperglycemia + ACTH high ➡ cushing syndrome


Adrenocorticotropic hormone (ACTH)

68.AST ALT BILIRUBIN high ➡ hepatic jaundice

**Primary hyper thyrideism➡T3T4 high& TSH low

***secondery hyper thyridism➡T3T4 high & TSH high

***primary hypo thyrodism➡T3T4 low & TSH high

*** secondery hypo thyrodeism➡T3T4 low & TSH low or


normal

69
69.donar feeling nausea
‫ش‬ ‫ش ص اث اء ا تبر حسا ب ا‬ ‫ا‬
Remove needle and stop donation

70-Normocytic normochromic anemia➡ hemolytic


anemia/aplastic anaemia
‫ا‬ ‫ا‬ ‫ت‬ ‫ش ا‬ ‫ر‬ ‫ر‬ ‫ر ست‬ ‫ا بت‬

70
71.Typhoid and fever➡ salmonella typhi

72.Urea nitrogen high amylase high ➡ acute pancreatitis

73.B. hemolytic in blood agar no growth in mackonky


catalase+ ve ➡ s.aurus

74.Suffering vomiting ➡ enteropathogenic


‫ت‬ ‫ت‬ ‫دا ا ت ر باث ج‬ ‫با رب‬

‫ا ضر ا ا ب ش‬ ‫ا اص ر‬ ‫ا‬ ‫ا س ص رة ب ت‬
‫رب ا ر ا ا ج اب‬

71
75.Pseudomonas

76.Acid fast parasite under oil immersion➡ cryptosporidium


zn ‫با‬ ‫بتصب‬ ‫دا ا براسا ت ا‬

77.PCO2 high ➡ respiratory acidosis


‫د ش‬TSH normal, T3 T4 high ‫ا‬
78. secondary hyper thyroidsm


‫ت ت بح د‬

72
1.high ei i CSF a e:

A. Ne a ce e e a

B. B ai ba ie da age

2.c ed CSF a e:

A. Se d che i

B. Rejec

3. a c ea ic ca ce a e:

A. CEA

B. CA19-9

4. c id c e, hich fac ?

A. Ca e

5.B ac i e

A. he a a i e deficie c

B.ca c i

B ac i e ca ed b A a ia , deficie c f i e e ab i a d
he a a i e

6.he hi ia A deficie c

A. Fac VIII

7.DNA i :

A. He

B. I f e a

8.c eac i

A. Reage eac i h a ige

B. Si i a e i
9.ch ic HDV

A. HBV ag +, HDV ab -, HBVc ag +

B.Igg a i HDV

10.DAT e :

A. HDN

11.Mi e a c ic id

A. A d e e

B. C i

Mi e a c ic id (a d e e), g c c ic id (c i ,c ic e e), a d e
e id (a d ge , e ge , ge e e)

12.B de e a e i ca e?

A. P ge

B. W i gc gh

13. a ie ih e ai a ga d

A. M b

14.U i e a e f ch ice f g e fi a i ae

A. 24 h i e
15. a ef f e ec i

A. F e h

B. F a i e fi ed

16. a ie ha a e ia Lab fi di g :

High TICP, fe i i , igh high hb A2, Pe ci ce :

A. I deficie c

B. I deficie c + G6PD

C. I deficie c + be a ha a e ia i

D. Sic e ce a e ia

17.bab ha i i g a d dia hea

A. R a i

18.Ag +Ab hich a h a ?

A. C a ica

B. A e a i e

19.MIC bac e ia b i ga ici i ga e 1 ga db e ci i g ga e 8 b MIC


af e i g

he ge he a 0.3 ha i hi c bi a i a ed?

A. A ag i

B. S ge

20.MCV

A. He a c i de ided RBC

B. He a c i e id b RBC

21.HIV e i e e

A. PCR

B. R c

22.SGAM

A. 42 da
23. a ec e :

A. XLD

B. XLD AND BA

C. BA + Ch c a e

24.bac e ia i i e

A. Ae bic

B. A ae bic

C. Ae bic + 10% c 2

D. A ae bic + 10% c 2

25.Which HIV a e ' ca e a i i

A. U i e

26. hich bac e ia ca ' g a ificia edia?

A. T a e a bi d

27.W a eg a i h he fi chi d ha a ca a h e hich e h d be d e

A. a aG di

28.f d i i g ca e/

A. C. Pef i ge

29. ha ha e i GVHD?

A. H figh a a i e

B. a a i e figh h

Vi ib e T h c e i d a b d a ac eci ie

30. e a e ?

A. Ph ica

B. Me a

31. e be a i ei :

A. LDL
B. HDL

C. VLDL

32.G a c ec ce ae

C. I adia ed RBC

D. N i adia ed RBC

O ce a b dc e ( ed ce , a ee a dg a c e ) eed be
i adia ed.

33. e i ab i g e e hi g f ,

A- a e 2 a e f he edge

B- a e 2 a e f he ce e

C- a e 3 a e f he ce e

D- a e 2 a e f he ce e

34.Wha d e e f ee a d f e e f he ce e b a e ba ie ?

A- CO2

B- A i acid

C- i id

D- ch e e

35.A e a e a b i h ic e ce a e ia b did e e i fe
h ae, h ?

A- highe HBF ha HBS c ce ai

36.Ca ed f db e e?

A- C. b i

B- C. diffici e

C- C. e a i

D-C. e f i ge

37.Ca e i h T3 a d T4 i c ea ed

A-H e aah di

B-h a h idi
C.h e h di

38.Ca e i h g c e e e a ed a d ic acid e e

A- diabe e a d G

39.Life i e f RBC ?

A-30 Da

B- 60 da

C-120 da

D- 160 da

40.Pa ie i h MI, af e h a i e i i i c ea e?

A-10

B-30

C-40

D-60

41.Wha i he f da e a e ec ef e ea d ce ?

A- S di

B- a i

C-Ch ide

D- Mag e i

42.Me ab ic acid i i a cia ed i h?

A- V ii g

B- Dia hea

Me ab ic a a i i a cia ed i h ➡ ii g

43.Wha a ibi ic i ib e a e i a ?

A ici i

44.Ge be i i i e, ha ga i ?

A- C.a bica
45.T a ii a e i he ia ce i i e e a ed ?

A- f He e

B- P i a ba

C- B adde

D- U e h a

46.O a a e?

A-CA-125

47.Ma e f h id? Th g b i

48.H e J b die a e a cia ed i h?

A-Tha a e ia

B-I deficie c a e ia

C- a a ic a e ia

Mega b a ic A e ia& Sic e ce a e ia& He ic a e ia

49.Wha b dg e e Ma a ia i fec i ?

A- F (a-b-)

B-F (a+b+)

50.I c ea e Ga a a d Be a i e ec h e i , ha c di i i
a cia ed?he a ic ci h i

51.K/A i h H2S g a ega i e?

A-

B-Sa e a

C- Shige a

52.AB eg a ie eed a a ee a fe , ha e h d ge ?

A-AB ega i e
53. a ie ha a i gj i

A i d ANA + e

Rh a id fac + e:

A. SLE

B. A c h ic ci h i

C. D g i d ced

D. He a i i

54. i e a e ca e af e h f c e

A. Acce

B. Rejec

55. hi e bi ai :

A. H&E

B. Pea ' e b e

56.be a he ic g a + e c cci a ca e ha gi i , e i ef e ici i ,


PYR + e:

A. S a h c cc a e

B. S e ge

57.high e ea e, ESR

A. Fa e e e a ed beca e bc a i g

B. Fa e e e a ed beca e e ea e edi e a i .

C. Fa e e e a ed d e ed c i i a a i c i

58.C c cc ai

A. G c

59.C c cc ai

A. I dia i

60. H. I f e a h g :

A. G a - e c cc bacci
61. i e a e h ed high g c ea d ic acid a d e e :

A. D +g

62.+ e f a e ia

A. Re he e

B.I fec i c e

63. i c a, hic fi h ed i g ce a d ba a i e, haa i he c i e


age ?

A. Bi e b a h d

64. ega i e f i a a

A. Re eg

B. Re

C. U e e e ii e eh d

65.ba hi ic a i g

A. DNA

B. RNA

C. He g bi

66.a a ic a e ia

A. Mic c ic h ch ich
B. N c ic ch ic

67.i deficie c a e ia i

A. Mic c ic h ch ic

B. N c ic ch ic

68.i deficie c a e ia:

A. Mic c i , B. H ch ia, C. Pe ci ce

69.Wha i da ge f he hea ?

A. H a e ia

B. H e a e ia

70.Red ig e a i edi

A. E. C i

B. Se ia

71.Sa e a hige a e b? c e

72.E.c i EMB? e a ic g ee hee


73.Rh a ige

A. D C E c e

74.H diffe e ia e be ee i eb a d di ea e a d He hi ia A

A.fac VIII a a

B.b eedi g i e

75.E a a e he i :

A- he g bi e ia

B- he c e

76.Hai ce a e a f hi e ce f ?

A- 103

B- 5

C- 25

D- 11c

,hai Ce e e ia CD19, CD20, CD22, CD11c, CD25, CD103

78.Diag i chi c c ga i

A. La ea i

B. Egg i

C. La ea a d egg i

D.h da id c

79.I chi d e , ha i e high hi e b d ce ? h c e

80.O he a ef g a e di ea e ? ic diff eg i e

81.c ea i i e c ea a ce ca c a ed

82.g a i i e bac e ia g a 43 C? i e ia c ge e

83.A be ai ai ed ? c e bac e i di h he ae

84.Re f agg i ai i ?c be e fi a i

85.C be a fi a i di ib i e a ? 56 C

86.Bac e ia e i a ce a ici i ? e i e e iae


87.S e e ha had a e e a ea de e hi g i a i he e b
a e ? e d - he c e ia

88.A ca e e ea e a 121 C f 15 i e ,15 e e

89. a a h di h e de ec i ?h e ca cae ia

90.diffe e ia i be ee he e f He hi ia ? fac a a

91.Wha d e e i i e he be ch i he ab a ?f a deh de

92.A ab i a d ca ab i a e he b ad c a e f bi che ica eac i


ha a e e ab i

93.Phe i di i fec a c g be ch

94.Fi h i e d ➡ e

95.H e a e ia: h e ad e i , e ce i e ea i g, b , diabe e


i i id .

96. eiha e -be e) P i ci e:

1. Bab c e , he i c

2. Bab i , he c ea

97. SITUATION: A e e ge c a a a ie e i e a f i . Si i fb d
a e de ed a . The e i i e d a a a ie a e. O- ega i e b d i
e ea ed. Whe i c a ibi i e i g be e f ed?

a. C a ibi i e i g be e f ed bef e b di i ed

b. C a ibi i e i g i be e f ed he a a ie a e i a ai ab e

c. C a ibi i e i g a be e f ed i edia e i gd e

d. C a ibi i e i gi ece a he b di e ea ed i e e ge c
i ai

98.Which e hige c ce ai c e e ( A.C5/B.C3/C.Ca )


99.Pa a i e ca e e igi i ➡Naeg e ia f ei

100.Va ia i i he i e f RBC i ➡( A i c i ) RDW

101. he i i ha e f ce di i i e e be ee ?!

1- G2 & G1

2-G1 & S

3- M & G1

4- S & G2

102.h h di ?

A. T3 T4 , high TSH

B.High T3 T4 , TSH

103.Ca a a e ii ea d ida e ega i e ?

A.P e

B. P e d ad

104.C ic fib e ?

A.a a ece i e

B.a a d i a

105.-Diffe e ia e be ee HB A a d HB F ?

A.HB e ec h e i

B.I a a
106.P ei i IDA ?

A. a fe i

B.Fe i i

107.MCV=

A.HCT RBC/10

B.HCT/RBC X 10

108.B12 deficie c

A.Mega b a ic a e ia

B.A a ic a e ia

109.H e e ii i e1?

A.IgA

B.IgE

C.IgG

110.Re ic c e ai ?

A.W igh ai

B. e h e e b e (S a i a)

111.-P ei ai i e ec h e i ?

A. cea S

B.a id b ac

C.C a ie B i ia b e

D.A

112.E e i dige i ?

A. a a e

B.Li a e

113.E e ae e ed ?

A. A idi i

B.i ac i i i
C.E a ic eh d

d.Ki e ic

E e Mea ed ➡ i e ic

114.Pa e hei e b d i c i ?

A.I

B.DNA

C.RNA

ba hi ic a i g ▶ RNA

H e J b die ▶ DNA

Hie ce ▶ De a ed HB

Pa e hei e ▶ I

D he b die ▶ RNA e id e

115.A i c i ?

A.RDW

B.Mc

c.MCH

D.Mchc

Mc ea e a e age i e f RBC

Va ia i i he i e f RBC i A i c i ea ed b RDW

116.Wha e f B ce a ef ed af e a ige i ai ?

A. P a a ce a d e B ce

B. Ma e B ce

C. A ige -de e de B ce

D. Rece -ac i a ed B ce
117.-Wha h d be d e if a f a d a d e e e ABO e a e ega i e?

A. Pe f addi i a e i g ch a i g i h a i-A1 ec i a d a i-A,

B. I c ba e a 22 C 4C e ha ce ea e e i

C. Re ea he e ih e eage

D. R a a ib d ide ifica i a e

118. Wha d i h hi ABO di c e a c :

Pa ie Ce Wi h Pa ie Se Wi h A i-A A i-B A1 ce B ce 4+ 4+ 4+ 4+

a. Mi a ea d e

b. Wa h RBC

118-Mic c cc a dS a h c cc ecie a e diffe e ia ed b hich e ( )?

A. Fe e ai fg c e (OF be)

B. Ca a a e e

C. G a ai

D. A f he e i

119.Pa ie Hi

118.A 1- ea - d i fa a ad i ed he h i a i h ec e e i a i f he
a 5 da .The a edica hi e ea ed ea b i i g a d a e e e eb eed
a 3 h f age, ece i a i g a f i he a . The he had had a e e e
eb eed 8 ea ag . The fa he a e ed b eed ea i af e ace a i . The
a ie a a f ed i h 2 i f ac ed ed ce ad i i .

Hgb: 4.5 g/dL N.R 13 15 g/dL

Pa ee c : 249 109/L N.R 150 450 109/L

PT: 11.2 ec N.R 11 13 ec

APTT: 34 ec N.R 28 37 ec

ADDITIONAL LABORATORY TESTS:

Fac VIII a a 70% N.R50% 150%

P a e e agg ega i : Ab a ADP, e i e h i e a d h bi ; a


i ce i The e c i ica a ife a i a d ab a e a ec i e ih
hich c di i ?
A. Wi eb a d di ea e

B. Be a d S ie d e

C. G a a h ba he ia

D. Fac VIII deficie c

119.bac e ia ca e e i ?

C idi b i ,C idi ea i

120.Which f he f i gi he be a e e he ead f ga i i a
hea h ca e e i g ?

A. Wea g e .

B.Wa h ha d ,.

C. Wea ab c a .

D. I a ed a ie .

121. e e diag e ea D

A.IAT

B.DAT

122.P i a i fec i f HIV bac e ia? Shige a


123.N a a ge f e e a a i ? 20 i i 200 i i

124.Ba hi ic i i g ee i ?

i ai ed Hb he i , a c h i ,a d ega b a ic a e ia , i i g

125.Mi a e ? ch ic

126.O e ai gi e da c ?c ce ai f he a i

127.Bac e ia ID a d e ii i ? Mic ca a a a

128.Ch e ? FISH ( f e ce ce i i h b idi a i

129.Ne hi e a ? che a i

130. ced e f Zehi Ni ai ?

131.F b dc eb e he a eb d ? 10 20

132.bac e ia a ae bic d ce e?C id

133. a ef a ae bic ? e a f id

134.Which f he f i g cha ac e i e i a i K?

A. I i e i ed f bi gica ac i i f fib i i

B. I ac i i i e ha ced b he a i he a

C. I i e i ed f ca b ai fg a a e e id e f e c ag a i fac

D. I i ade b he e d he ia ce

135.Which ae e i c ec ega di g a e age f he h bi i e


e ?

A. S ab e f 24 h if he a e i ca ed

B. S ab e f 24 h if he a ei ef ige a ed a 4 C

C. S ab e f 4h if he a ei ed a 4 C

D. Sh d be i hi 8 h
A. a a a e f PT e i g a e ab e f 24 h a

e e a e if ca ed/B.Ref ige a i g he a e ca e c d ac i a i f fac VII


a d, he ef e, h e ed PT e /C.The APTT a e a e ab e f 4 h if
ed a 4 C.

136.high e ea e f h e ,e ecia c i , i a ca e f face.

137.A he a ef G a e di ea e?

e h ha ic g i e .

Pa ' di ea e,

Begbie' di ea e,

F aja i' di ea e, F aja i Ba ed d e, a d

Ma h' di ea

138.ac e dia hea ca ed b

A.gia dia

B.e.hi icae

139. ea ha e a d ha e f age a

A.e.hi ica

B. (gia dia)

140.diffe e ia edia :

Macc e aga

B d aga

141.d be e be a he i :

A.c. e f i ge

B.c.b ii

142.bi e d g:

A. a e e a

B.B ce a

143.d g bi e :
A.R a i

B.Rabi i

144.B ce a ei he

A. b e a

B.Th

145.T h c e a ei he :

A. h

B.B e a

146.F gi ha ca g a ea a d d?

A.di hic f gi

B.M d f gi

147. ega i e feedbac f ad e c ic ic h e:

A.c i

B.G cag

148. a a e e ea e bc a :

A.MCV

B.Hc

149.T e e a ai :

A.Gi e a ai ed

B.Si e i eg a i ai a d R ' ai

150.gh ce

RBC i H ic U i e eci e

151. e ic c e high i

He ic a e ia

152.He a c i : Hc e e a i e 70-80% i h CPD/CPDA

153.Fac ac i a ed b h bi ➡ Fac V , fac VIII , fac IX , fac XIII


154. ecific g a i f i e

A.1.010

B.1.020

C.1.030

The a a ge f i e ecific g a i i 1.005 1.030

155.The e e a e ca be

A.di ed

B. i ed

156.C i e ia f d ai

Hb12.5g%

Hc 38%

157.A g d ai

Hb 11g/d

Hc 33%

158. hich ei ie

A.Shige a

B.Sa e a

159.b bb e i ca e ESR ?

A.I c ea e

B.Dec ea e

I e fi i g f he ESR be a ca e b bb e f ai a d a i c ea e i he
ESR a e.

160.h diffe e ia i be ee he hi ia A a d B

A.Fac a

B.Cbc

161. a ie c e he c i ic h i g S e j i he ab e ( he a id
fac )

A. he a id a h i i
B. e ic e he ah i

162. h e d ca ib a i ?

F he acc ac

The g a f ca ib a i i i i i ea ea e e ce ai b e i g he
acc ac f e e i e

163. ha i effec ESR ?

High e

He i

164.fac X deficie c he PT APTT e i be ?

A.P ga g

B.P a a g

c.P ga a

D.P a a g

165.be e f MI ?

A.T i I

B.C

C.C - b

166.W e ih ac i e he 200 ha i he be e d e?

A.P ac i a e

a ac i e e i < 20 g/ .,

167.18 ea d a ie a f HIV e a d i a i i e a d he a ed de e e
he e a d de ed he a e ha d ?

A.Re he e he ice

B.Re he e he fa he

C.Re he e ab di ec

D.De e e he e a d de ed he a ea he a ie a

168.Wha i he be ef a e ?

A.Sc ee i g
B.F af e he a

C.Diag ic

169.e i he ia ce i i e a c ef ?

ha c ef he agi a a d e h a.

170.Wha i he c f ai e f i deficie c a e ia ?

A.Fe i i

B.Tibc

171.Wha i he ce ee i a b d?

Ne hi

172.if a e ha e ia acid i ha i he h f i e?

A.4-7

B.7.4-7.8

C.7.9-8

173.-Pa ie ih i K deficie c ha i he be
gi e ?

A.FFP

B.C eci i a e

a ee c.

174.- a e f fac VIII i be d e ha i


he e . a e ?R e ea e

175. e e c

A.20000.

B.200000

C.2000000

D.20000000

N a e e ce c >20 i i e

176. he i h b dg AB- bab O+ h i g i c ea e i bi i bi


a d HDNF ?

A.B d e cha ge
B.P a e e

177.La e i ELISA add b ae

178.hb i 7 h ?hb A

Hb F 3_6 h

179.Be ic e? R a ic e

180.N a a ge f e e a a i ? 20 i i 200 i i

181. a a h id h e i c ea e ?ca ci e e

182.S ecific e ef i e ? ALT

183.Be a ce f a c ea d ce? I i

184.Re ib e f he e e i f Na + , C -, a d H20, he e c e i f K+ a d H+
a d, he ef e, he a f f id i he b d ? A d e e

185.ae bic a e? i e eci e

186. i e ic a ec ? e a ea i h id d e high i id

187.c i gi he e be PT a d APTT ? g he APTT PT .

188.RPR U ed f ? c ee i g a d ea e i i gf hi

189.Ca e f he a ce a ca ci ai a ie h ecie e a i fc a i ed
i h hich i i i e :

a- e ie i .

b-HCV.

c-H a a i a i

He a ce a ca ci a (HCC) ca ed b he a i i B he a i i C i fec i

190.110-Ge e f h e ?

RHD RHCE

191.Sa e f H. i f e a?

S a e

Th a

Ea

L e e ia ac , C f
192.A ha age ca a hi a ie i chi d?

12 ge

193.-Wha c ce ai %i di ci a e be?

3.2%

194.(Pic e f K\A +H2S i TSI) ha i he bac e ia ha h hi eac i a d


ca e dia h ea?

Sa e a e e i idi

195.-Ca e: a ie i h high i a d TIBC? ha a e ia

196.Ca e: a e ed Af ica a d he e he j e aga de ec ib i


ch e a. Wha aga h d e?

TCBS a d e e ae

197.-F he e a e i he c ai e ? 3 f idd e

198.Which f he e i a ie - e ibi i ?

Cha ge i he ea e a ih e i g he h icia ab i

199.Wha f he e i Ke Ag?

K( e )

(ce a )

200.Wha i fa i ?

G6 d deficie c

201.G a ega i e d ha e bac e ia ca e e i gi i ?

Ec i

202.Ca e: 4 da d i fa a d ec he ha e i gi i ha bac e ia i e ca e
hi ?

S e . Ag ac iae

203.Y i e gi e a ab a e e ha h d d ?

Reca ib a i

204.N a ca i i e?

H a i e ca

205.Wha ca e fa e ii e e i A ig b i e i g?
O e ce if ga i c a i a ed eage .I fficie a hi g f he a ie '
RBC .P e e ce f fib i .

206.Which f he e i L he a Ab?

A i-L a

A i-L b

207.Pa ie ih idd he ha ha Ab?

A i-J 3

208.B dg e de ed b i e fici a i e e ?D ff

209.Wha i he ca e f he edi a f c ei e a ce?

Deficie c fad a e

210.A idi defi i i ? ea e g h bi di g be ee Ab a d Ag ( i e)

211.Ne hi a d NK ce , he a e a e i ?

NK da aged a he ce The a e b h i i a ei e e

212.P ed f ?

E i ic a d c ah a

213.D i i e i h HBV?

Pe a e defe a

214.P i i e RF, ANA, a d d DNA diag i f

SLE

215.S ecific e ef he a bi ia di ea e?

ALP

216.TPHA i ci e?

P i ci e f he TPHA e S ecific a ib die e e i a a e f a a


e bi d he e a ige he he a e i i c ba ed i h he a ic e . Thi
ca e he a ic e agg i a e, he e e f a cha ac e i ic a e i he
e e

217.T a e f b ea ?

HER2

218.Wha e f be ch i ed i ab? A i i

219.H diffe e ia e he hi ia A f B b ce ?A e d

220.S he i fg c ef ei a d i id?

G c e ge e i

221.-I a d ich ELISA ha i a ached he id?

S ecific Ab

222.C a f d i acidic i e?

Ca ci aa e

223.S e id h e he i ed b ?

Ch e e

224. a ie i h high c ea i i e a d a BUN? Se e e i e di ea e , i i g,


dia hea , a ii

225. a ef diag ic E ei Ba i

B d

Ui e

Sa i a

226.VIBRIO ch ia g
A.TCBS a d A a i e e ae

B.TCBS a d ai e b h

227.I hibi bi

A. ca ci

B.G g e

C.A ic ag a

228.Wha fac ac i a e h bi ... ?X a d V


Part1:

1/normal found in urine?

Chloride.

2/measure of viral Ag?

ELISA.

3/function of prolactin hormone?

Milk secretion.

4/organism cause cutenous mycoses?

Trichohyton rubrum.
al
5/principle of ELISA?

Antigen and antibody reaction using labeled enzyme and specific substrate.
ya

6/beta lactamase(B-lactamase)producing and resistance staphlococcus aureus what the suit


able antibiotic?
ze
ed

+ ampicillin+sulbactam.

Also vancomycin, methicillin.

7/symdrome of inappropriate antiduiritic hormone,(SIADH) we found?

Hyperglycemia.
8/function of neutrophils?

Diapedesis of is for chemotaxis.

9/donor come for blood donation suffer from skin lesion?

Send to the blood bank doctor.

10/convert fibrinogen to fibrin?

Coagulase.

11/falsely sample increase?

Potassium(K).

12/staphylococcus resistant to novobiocin?


al
Staphylococcus saorophyticus.

13/how differentiate staphylococcus aureus from other staphylococci?


ya
By coagulase.

14/enzyme for carbohydrate digestion?


ze
Amylase.

15/what the internal control purpose?

For laboratory own internal standards.


ed

16/CPD (citrate-phosphatedextrose), store blood for?

14 to 21 days.

17/how differentiate streptococcus group B(Streptococcus agalactiae) from other streptoco


cci?

By CAMP test, (Christie, Atkins, and Munch-Peterson).

18/patient with enlarge spleen(splenomegaly) what the type of anemia?

Sickle cell anemia.

19/have flask shape and round cavity?

Nematodes.
20/large amount of protein found in serum?

Albumin.

21/cystic fibrosis inheritance?

Autosomal recessive.

22/avidity?

Overall strength of antigen-antibody complex.

23/leukemia found in child( 2-10) ages?

Acute lymphocytic leukemia (ALL).

24/germ tube test for?


al
Candida albicans.

25/media for candida albicans?


ya
Corn meal agar?

26/describe E.coli?
ze
Motile. Indole positive, lactose ferment.

27/measuring of CK-MP creatine kinase-MP by?

Electrophoresis.
ed

28/control the adrenal gland?

Adrenochorticotropic hormone(ACTH).

29/media inhibit the growth of some bacteria and allow the others to grow?

Selective media.

30/describe mold?

Produce hyphae.

31/best describe of fungi?

80 ribosome.

32/Heinz body it is?


A remnant of denatured hemoglobin Hb.

33/non nucleated red blood cell?

Reticulocyte and RBCs.

34/bence jones protein found in?

Multible myloma?

35/pyrimidin DNA?

Cytosine (C) and thymine (T).

36/active metabolic stage of Chlamydia trachomatis?

Reticulate body.
al
37/infective stage od Chlamydia trachomatis?

Elementary body.
ya
38/trachoma disease of the eyes caused by?

Chlamydia trachomatis.
ze
39/blood film shows immature RBCs and immature neutrophils?

Leucoerythroblast reaction.
ed
40/hormone cause glycogenolysis and gluconeogenesis?

Cortisol and glucagon.

41/amount of the blood that entre the kidney?

1200-1500 ml/min.

42/most common microorganism cause urinary tract infection(UTI)?

E.coli.

43/hemolytic disease casued by?

IgG cross the mother placenta to the baby circulation.

44/secondary structure of protein?

Alpha helix (α-helix).


45/difference between salmonella and shigella?

Motility (salmonella).

46/paradoxical aciduria?

Hypokalemia.

47/the intermediate host of clonorchis sinensis ?

2 intermediate hosts:
al
Snail >> Bithynia.
ya
ze
ed

Fish>>cyprinidae.

48/which of the following enzyme affected by hemolysis?

Aspartate aminotransferase (AST).

49/this device used for sedimentation in lab?

Centrifuge.

50/what the function of the Antigen presenting cells (APC)?

Capture and presentation of antigen and display it to the T lymphocyte by MHC-II.

(dendritic cell, B cell and macrophages).


al

51/ function of T cell in immune response?


ya

For cell mediated immunity.

52/what the type of chromosome in martin bell syndrome?


ze
X chromosome (fragil).

53/in Down syndrome what happen?


ed
(trisomy 21), icrease number of chromosome than normal .

54/to prevent spreading of pathogen?

Isolate the patient.

55/paroxysmal cold hemoglobiuria caused by?

Anti-p.

56/special media for neisseria spp?

Thayer martin media (TMT) or modified thayer martin media.

57/suitable specimen in chemistry?

Serum.

58/heterophil antibody test is for?


Infectious mononucleosis , episten barr virus, burkitt lymphoma.

59/the turbidity measured by?

nephelometer or turbidimeter.

60/hepatocellular carcinoma tumor marker?

Alpha feto protein (AFP).

61/folate and b12 deficiency type of?

Anemia (megaloblastic anemia)

62/b12 deficiency due to what?

Intrinsic factor deficiency (partial gasteroctomy).


al
63/sample for diagnosis of neisseria gonorrhea?

From cervix.
ya
64/urine after 2 hours?

Reject it.

65/fluke liver disease caused by?


ze

Faciola hepatica.

66/Hydatid cyct cause by?


ed
Echinococcus granulosus.

67/most extra cellular fluid electrolyte?

Sodium (Na).

68/the end point of the ammonia in the liver?

Urea.

69/the properdin(factor P) found in ?

Alternative pathway of complement system.

70/Gilbert disease characterized by ?

Increase unconjugated bilirubin(indirect bilirubin).


71/steps of PCR(polymerase chain reaction)?

Denaturation>>annealing>>extension.

72/cardiac disease increased ratio?

LDL:HDL.

73/ Loeffler Serum Medium or Loeffler Medium used for?

Corynebacterium diphtheria culture.

74/blood agar is?

Enriched and differential .

75/septicemia and gas gangrene caused by?


al
Clostridium perfrenges.

76/mycology is study of?


ya
Fungi.

77/n hemophilia A there is deficiency in ?

Factor VIII (8).


ze

78/in hemophilia B there is deficiency in ?

Factor IX (9).
ed
79/Hemophilus influanzae grow in?

Chocolate agar.

80/charcoal media used for?

Legionella pneumophilia.

81/myocardial infarction enzyme is ?

CK, LDL,AST(SGOT).

82/ Cholestasis characterized by ?

High Gamma-glutamyl transferase (GGT),

γ-glutamyl transferase.

83/type of leukemia in more than 40 years old? CML(chronic myeloid leukemia).


84/anticoagulant used in coagulation?

Sodium citratr (Na citrate).

85/type of water used in lab?

Type 2 (II).

86/hemophilia A due to?

X-linked recessive.

87/color of gram negative bacteria?

Red.

88/Rh blood group phenotype?

CDEce.
al

89/Rh positive?

DD hemozygous.
ya

90/whip worm ‫الدايدان السوطية عشان تشبه السوط‬

Whip= ‫سوط‬
ze
‫ممكن ي ي عنها كيز الم فتاح ح و‬

Patient with rectal prolapse or egg with bipolar plug.

91/pin worm ‫تشبه الدبوس‬


ed

Pin= ‫ الديدان الدبوسية‬Entrobius vermicularis

‫ممكن يس ل‬

Parasite or worm don't lay egg in intestine?

Or other test for entrobius vermicularis

‫ن ول‬scotch tape test

‫او ممكن ي ول‬scotch test use for?

92/heterophil test or paul pannel test, monospot test or mononuclear spot used for?

For infectious mononuclosis caused by Episten barr virus (EPV).

‫ر هدي اال ابة‬ ‫ممكن بر و نلف‬


Atypical lymphocytes or burkitt lymphoma ‫ح‬ ‫كلها‬

/93how to prepare 1% of agar for agarose?

1 gram of powder.

94/heavy metal measyred by?

Atomic absorption.

95/myocardial infarcion enzyme?

Ck, LDH and SGOT (AST).

96/osmatic fragellity test used for ?

Hereditary spherocytosis.

97/worm antibody?
al

IgG.

98/cold antibody ?
ya

IgM.

99/bacteria grow in chocolate agar?


ze
Hemophilus influanzae.

100/which wnzyme affected by meal?

Lipase and amylase.


ed

Part2:

1/sample error result in elevated potassium (K)?


Hemolysis.
2/calculate MCHC?
33.
3/determining the parathyroid hormones?
Hypercalcemia.
4/anticoagulant for CBC?
EDTA.
5/sample rejection criteria?
Insufficient quantity.
6/differentiation plasma from serum?
Fibrinogen in plasma
7/diagnostic test for HCV?
RIBA.
8/platelets clump in EDTA?
Use Na citrate (sodium citrate).
9/in complement fixation test the temperature of patient serum heated at?
56 C.
10/cells increase in allergy?
Basophils.
11/used to produce cheilumenenc?
Horseradish peroxidase (HRP).
12/in HDN, indirect comb test for?
Mother serum.
13/iron selective electrode use for? (electrolytes).
14/enzyme affected by meal?
Lipase.
15/mother Rh negative, infant Rh positive, what give for mother?
Anti-D.
16/life span of RBCs?
120 days.
17/selective media for vibrio cholera?
al
TCBS (thiosulfate-citrate-bile salts-sucrose).
18/universal recipient?
AB+ve.
ya
19/type of RNA take amino acid to the site of protein synthesis?
tRNA (transfer RNA)
20/clinitest use for?
Glucose.
21/anemia of student?
ze
HCT(PCV) and Hb.
22/rehmatoid fever diagnosed by?
ASO.
23/patient under antibiotic treatment and continuous diarrhea what the suspected organism?
ed
Clostridium difficle.
24/measure strength of interaction between epitops(Ag) and paratops(Ab)?
Affinity.
25/control of vaginal normal flora?
Low PH.
26/in early fasting the body take glucose from?
Hepatic glycogenolysis.
27/sample to diagnose the neonatal meningitis?
Neonatal CSF.
28/convert 0.025g?
25.
29/hemophilia inheritance?
X-linked recessive.
30/trisomy 21?
Down stndrome.
31/biosafty level of fungi?
Level 2.
32/what the last step of ELISA?
Add the substrate.
33/wright stain for?
WBCs count.
34/how to get or obtain the serum?
Put the sample at room temperature and the centrifuge it.
35/differentiate AS and SS?
39/DNA pyrimidine?
Cytosine(C) and thymine (T).
40/lamb used in spectrophotometer?
Tungestin.
41/antibiotic link with potassium (K)?
Valinomycin.
42/Hodgkin lymphoma?
Reed Sternberg cell.
43/this organism caused relapsing fever?
Borreliarecurrentis.
44/not found in eukaryotic cells?
Mesosomes.
45/not found in fungi?
Mesosomes.
al
46/time to diagnose bacteria in culture?
72 hours.
47/causes of HDN?
ya
IgG cross the palcenta from mother to baby circulation.
48/gene of familial hypercholesterolemia?
LDLr.
49/investigation assoiated with pregnancy?
Human chorionic gonadotropin (HCG).
ze

50/DNA stain in electrophoresis?


Ethidium bromide.
ed
51/type of the light that measured by nephlometer?
Scattered light.
52/metabolic alkalosis?
Vomiting.
53/important organs of buffering system?
Kidney and lung.
54/IgG present in a high concentration in serum?
IgG.
55/Cells present I parasitic infections?
Eosinophils.
56/mold produce?
Hyphae.
57/Acromegaly?
Over secretion or high secretion of Growth hormone in adult.
58/gigantism?
Over secretion or high secretion of growth hormone in child.
59/biosafety level for mycobacterium tuberculosis?
Level 3.
60/platelets in blood bank stored?
5 days.
61/burkitt lymphoma?
Epstein barr virus (EBV).
62/ketogenic amino acids?
Leucine and lysine.
63/Bordet-Gengou blood agar for?
Bordetella pertussis.
64/antibiotic inhibit RNA synthesis? (Rifampicin)
65/in PCR the denaturation temperature is?
94 to 96 C
66/Vibrio cholera in TCBS give yellow color due to?
Fermentation of sucrose and acid production.
67/mRNA to DNA?
Reverse transcriptase.
68/taq polymerase use in?
Polymerase chain reaction (PCR)
69/sample send to the lab immediately?
Cerebro spinal fluid (CSF) sample.
70/favismanemia caused by?
al
Deficiency of G6PD enzyme.
71/differentiation of hemophilia A and B?
By factor assay.
ya
72/the black center color of salmonella in XLD ( Xylose Lysine Deoxycholate) from?
Due to production of H2S (hydrogen sulfide).
73/hormone responsible of sodium (Na), potassium(K) and water reabsorption?
Aldosterone.
74/causes of Hemolytic disease of newborn (HDN)?
ze
IgG D (anti-D)
75/causes of HDN?
Anti-K.
76/carrier of hemophilia?
ed
Mother (female).
77/Accreditation is?
non governmental agency that concerning with evaluation of the labs and give certification.
78/ type of hemoglobin at 7 months?
HbA.
79/sample of indirect antiglobulin(IAT), in case of Hemolytic disease of newborn HDN?
Mother serum.
80/visceral lishmaniasis (kalazar) caused by ?
Lishmania donovani.
81/thrombocytopenia?
Megaloblastic anemia.
83/protein transport iron?
Transferrin.
84/function of flame in atomic absorption sectrophotometry?
Break the chemical bonds of molecules.
85/formation of glucose from protein and lipids called?
Gluconeogenesis.
86/Function of epinephrine?
Fight and flight.
87/beta hemolysis, (B-hemolysis) occure in?
Blood ager.
88/gram positive bacilli spore forming?
Clostridium.
89/gram positive bacilli non spore forming?
Listeria monocytogenes.
90/what the confirmatory test when found ketone in urine?
Acetest (acetotest).
92/Rh antigen?
Homozygous (DD).
93/most common method of automated immunoassay?
Chemiluminescence.
94/which of the following effect the sensitivity of the sample?
Low concentration of antigen.
95/common error or mistake in lab?
Sample clot in CBC.
96/which of the following read by low power field (LPF) microscope?
Casts.
97/what of the following sample suitable for anaerobic culture?
al
Pleural fluids.
98/suitable sample for pneumocystic carinii?
Lung.
ya
99/salmonella, shigella, and E.coli transmission?
Oral-faecal.
100/hormone cause glycogenesis and glycolysis?
Insulin. ‫ات ا ا‬ ‫ا‬ ‫ا‬ ‫ا‬ ‫ا‬
ze
Part3:

1/abundant hemoglobinin in infant?


ed
HbF.
2/parasites isolated from sputum?
Paragominus westermani and echinococcus granulosus.
3/Hemolytic disease of newborn caused by?
Anti-P ‫هدي ال ح‬
Anti-C
Anti- A
4/hypernatremia and hypokalemia is found in?
Diabetes mellitus.
5/ anticoagulant used for agglutination test?
Sodium heparin.
6/principle of spectrophotometer?
Measure light directional proprtional to the concentration.
7/type of malaria that form shizont rosset shape‫شكل وردة‬
Plasmodium malariae.
8/media that contain vancomycin cloistin and nystatin.(VCN)
Thayer martin media.
9/media that contain vancomycin cloistin and nystatin, trimethoprim(VCNT).
modified thayer martin media (MTM). Proteus.
10/causes of HDN?
Anti-k ‫ح‬ ‫هدي‬
Anti -p1
11/indian ink stain use for?
Encapsulated microorganisms.
12/ moher pipette pic.
13/sample for HCV labeled is ?
Red.
14/anticoagulant used in blood culture?
SPS, sodium polyanethol sulfonate.
15/nucleated RBCs NRBCs found in?
al
Hemolytic anemia and megaloblastic anemia.
16/highest parameter found in the blood?
Albumin.
ya
17/to differentiate between sickle cell disease (HbSS ) and sickle cell trait (HbAS) ??
Use of hemoglobin electrophoresis.
18/deficiency of vitamin B 12 and folate ?
ze
Megaloblastic anemia.
18/deficiency of vitamin B12?
Pernicious anemia.
ed
19/negri bodies?
Eosinophilic inclusion bodies in the cytoplasm of nerve cells.
20/hemoglobin function?
Carry the O2 from lungs to the tissues.
21/7 month old baby , what the type of hemoglobin? (HbA).
22/if you want to do indirect antiglobulin test (IAT) in case f HDN what is the sample?
Mother serum.
23/type of worm Ig (immunoglobulin) that cause HDN?
IgM
IgG ‫ال ح ح‬
IgA
24/HDN caused by?
IgG from mother serum to the infant blood stream through the placenta. ‫ال ح حه‬
‫با ي ال ارات معكوسة‬
/25ketonuria seen in ?
Type 1 diabetes mellitus.
26/why untreated diabetes mellitus (DM) causes ketones in urine (ketonuria)?
Cells can't utilize the glucose.
27/patient has high cortisol and high glucose?
Hyper cortisolism.(cushing syndrome).
28/bacteria that cause most of urinary tract infections (UTI)?
E.coli,
29/anticoagulant for whole blood?
EDTA.
30/albert stain used for?
Corynebacterium diphyheriae (volutin or metachromatic grnules).
31/obligate (strict) anaerobic bacteria?
Can't breath air
Colosteredium
Bacteroides
al
Actinomyces
32/direct sandwiche ELISA use to detect?
Antigens.
ya
33/mother infected with meningitis what sample take for diagnose the baby?
CSF baby.
34 /hemophilia gene?
ze
X chromosomes.
35/glucose 6 phosphate dehydrogenase (G6PDH) what type of anemia?
Hemolytic anemia.
ed
36/universal donor?
O negative.
37/all blood parameters are low?
Pancytopenia.
38/bacteria gram positive cocci, catalase +ve and coagulase +ve.
Staphylococcus aureus.
39/different between salmonella and shegilla?
Motility.
40/hormone important in fertility?
Progesterone.
41/systemic lupus erythromatous detected by?
Antinuclear antibodies(ANA).
41/protein pound in complement?
Properdin.
42/temperature of DNA storage.
4C or -80 C.
43/end point of NPN?
Protein (amino acids).
44/bacteria like drumstick?
Clostridium tetani (tennis rackets).
45/pinkish urine sediment after centrifugation?
Amorphus urates.
46/patient blood grouo A2 give blood?
A2 or O and A.
47/patient with high calcium (Ca)?
Hyperparathyrodism.
48/ketone found in urine?
Sugar more than 220 mg /dl.
49/platelets read low in EDTA?
Use sodium citrate.
al
50/use to identify virus antibody?
Hemagglutination test.
51/glucose in urine (+++) what that responsible?
ya
Glucagon.
52/media for fungi?
Sabroud dexterose agar (SDA).
ze
53/this organism sensitive to bacitracin?
Streptococcus pyogenes.
54/tryptophan and niacin deficiency cause?
ed
Pellagra.
55/organism produced urease?
Proteus.
‫وممكن بر و‬h.pylori
Klebsiella.
56/semi permable membrane?
Ultrafiltration.
57/type of PCR that produce mRNA to DNA
Reverse transcriptase PCR (RT-PCR).
58/asses quality in lab?
Reliability.
59/isulin produce from which gland?
Beta cells of islets of langerhans of pancreas.
60/Respiratory syncytial virus (RSV) cause?
Bronchitis in young infants.
61/group A streptococci(s.pyogenes) , what type of hemolysis?
B-hemolysis (beta).
63/group B streptococci,(s.agalactiae), type of hemolysis?
B-hemolysis (beta) clear zones (complete hemolysis).
64/s.pneumonia and viridans type of hemolysis?
Alpha hemolysis (green hemolysis). Partial hemolysis.
65/non hemolytic or gamma hemolysis?
Enterococci.
66/the solid phase of indirect ELISA coated with?
Specific Ag.
67/of direct ELISA coated with?
Specific Ab.
68/of sandwich ELISA?
Specific Ab.
69/competitive ELISA?
al
Specific Ab.
70/indian ink stain use for?
Cryptococcus neoformans.
ya
71/type of Ig that cross the placenta?
The only one (IgG) have placental receptor remember (Go ,Go)☺
72/in pregnancy we measure?
ze
Human choroinic gonadotropin(HCG).
73/meningitis in baby the best sample is ?
CSF sample.
ed
74/send immediately tolab?
Cereprospinal fluid (CSF).
75/most extra cellular cation?
Na.
76/most intracellular cation?
K.
77/the percentage (%) of bast cell in acute leukemia?
>20 %.
78/type of Hb in 7 montbs baby?
HbA.
79/high cortisol and high adrenocortecotropic hormone(ACTH)?
cushing syndrome.
Ameoba stain by?
Iodin.
80/one person come for nlood donation, his test show positive for hepatitis so?
Permanent deferral.
81/the urine containers contaion?
Acid (boric acid).
82/measure strength of the intraction between single epitop and single paratop?
Affinity.
83/hh bombay group?
hh not H and deffect in enzyme (L-fucose transferase).
84/glucogenesis and glycogenolysis?
Cortisol and glucagon.
85/dexamethasone role in cortisol?
Supressor.
86/CRH (corticotropic realasing hormone)is regulated by?
Pituitary gland..‫ال دة الن ام ة‬
/87dinitrophenol in a respiratory chain?
Uncoplar.
al
88/PHOX (phagocytic oxidase)?
NADPH oxidase kill by ROS reactive oxygen species. NO, H2O2, etc..
89/high Na low K?
ya
DM.
90/end point of glcolysis aerobic?
Pyruvate.
ze
91/Endpoint of glycolysis anaerobic ?
Lactate.
92/endpoint of glycolysis in RBCs?
ed
Lactate.
93/bacteria need CO2 carbon dioxide and chocolates?
Neisseria.
94/enzyme responsible for carbohydrates(CHO) digestion in stomach?
Amylase.
95/shigella on XLD colorof colonies?
Pink color.
96/shigella on macckonkey agar?
Colorless colonies.
97/sickling test is affected by or depend on?
Solubility of HbS.
98/different between ALL and AML?
Auer rods for AML.
99/bile acid in liver?
Cholesterol.
100/thrombus in myocardial infarction is called?
Mural thrombus.
Part4:

1/specific test for iron deficiency?


Serum ferritin.
2/CK-MB measured by?
Electrophoresis.
3/sample stay over night what parameter give false result?
K potassium.
4/cryoprecipitate use to treat?
5/lipimic sample affect result?
6/mother AO father BO?
7/avoid smudge cell?
8/neglar reaction used for(positive)?
Clostridium preferences .
9/Ag in HDN?
D antigen.
al
10/hepatitis B, C, A transmission?
Parental.
11/why untreated DM have ketonuria?
ya
Cells cant utilize the glucose.
12/gram negative?
Pink or red
Colorless
No stain
ze
13/warm Ab?
IgG.
14/cold Ab?
IgM.
ed
15/type of macCkonkey?
Selective and differential.
15/platelets at 22C?
5 days.
16/polychromatic non nucleated RBC?
Reticulocytes.
17/cryoprecipitate to treated what?
VWF and Fibrinogen deficiency.
18/rice watery diarrhea?
Vibrio cholera.
19/wright stain for ?
Differential count and reticulocytes (WBCs).
20/HBV close to which virus?
HDV.
21/virus sample?
Heparin.
22/accreditation ?
Non governmental agency.
23/hormone regulate adrenal cortx?
ACTH, adrenocorticotropic hormone.
24/produce color reaction?
Spectrophotometer.
25/lipemic sample?
Cholesterol.
26/o group plasm can given to?
O negative
27/platelets store temperature ?
22 – 27 C.
28/leukemia in more than 40 years old?
CML.
29/hocolate media contain antibiotics?
Thayer martin media.
30/fungi like drum stick?
31/hematocrite divided by RBCs?
MCV.
32/turbidimetry principle?
Measure the transmitted light.
33/ dimorphic fungi?
Candida albicans.
34/before take sample from penitent?
Assure the patient.
al
35/lactose fermenter?
e.coli, klebseilla and enterobacter.
36/hormone control adrenal gland?
ya
ACTH.
37/few amount of urine is called?
Oligouria.
38/method of protein detection in CSF?
Panday's test.
ze
39/in electrophoresis PH 8.6 protein?
Migrate to positive chord (anode).
40/common mistake in the lab?
CBC sample clot.
ed
41/best method to detect parasite on stool?
Wet preparation.
42/best method to detect intestinal protozoa?
Concentration techniques (formal ether concentration technique)
FECT.
43/intestine infection other sample than stool?
Intestinal aspiration.
44/blood in urine , this is called?
Hematuria.
45/hyperglycemic hoemone?
Glucagon.
46/CBC and blood flom shows macrocytic cell?
Partial gastrectomy.
47/tripple phosphate pic.
48/obligate anaerobic bacteria?
Colestredium oerfrenges.
49/gkucose is store in liver as ?
Gkycogen.
50/bacteria cause gas gangrane ?
C.perfrenges.
51/macckonkey use for?
Isolate lactose fermenter from non lactose fermenter bacteria.(enterobacteriacae).
52/storage of CSF sample ?
35 to 37 C
53/ in cross matching use?
IAT.
54/cylindurai?
Cast in urine.
55/enzyme cause favism anemia?
G6PD.
56/hormone casue glycosuria?
Glucagon.
57/in cloestredium difficile culture is better than serological test ? Why?
To differentiate the toxigenic strain from non toxogenic.
58/chagas disease caused by?
T.cruzi.
59/RBCs break down mainly in?
Spleen.
60/most common method of immunassay?
Chemillumisence.
al
61/what we collect for CBC?
Whole blood.
62/what the protein coated the nuclear material of virus?
ya
Capsid.
63/ELISA last step?
Add substrate.
64/sensitivity?
Measure the low concentrations substances.
ze
65/type of urine sample usually received?
Random.
66/DAT?
In vivo Ab coted RBCs.
ed
IAT?
In vitro Ab coated RBCs.
67/most common cause UTI?
E.coli.
68/test do for Rhematoid fever?
ASO.
69/Kernicterus in brain casued by elevation of ?
Unconjugated bilirubin .
70/substance promote glucose utilization?
Insulin.
71/substances affected by hemolysis?
K
Glucose.
72/electrolyte measure by?!
Iron selective electrode.
73/substance connect humoral immunity adpative and innate immunity?
Cytokines.
74/virus cause diarrhea?
Rota virus.
75/best describe s.arue?
Catalase positive.
76/function of flame?
Break the chemical bond of molecules.
77/coronary atherosclerosi?
LDL:HDl.
78/CKMB?
For myocardial infarction.
79/enzyme conjugate bilirubin?
uridine diphosphate (UDP)- glucuronosyltransferases (UGT) .
80/prolonged fasting.?
Lypolysis.
81/how differnatie aerobic from anaerobic bacteria?Chocolate blood agar with CO2.
82/ only take energy from glucose?
Brain.
83/virus sample?
Heparin.
84/E.coi cause diarrhea?
E.coli O157:H7.
85/orgnodm transmitted from human to human?
Treponema pallidum
al
86/blood glucose read 6.1 ,glucose in CSF equal?!
4 (6.1 x 2/3)
87/substance measure after excersise?
ya
Dopamine.
88/affected by excercis?
Ck..creatin.
89/PCR for virology?
Use EDTA.
ze
90/best describe zygomycet ?
Zygospore sexaul and asexual.
91/virus affect the newborn?
CMVand Herpes virus (HSV).
ed
92/automated immune assay?
Chemillimiscence.
93/high alpha beta bridge?
Hepatic and alcoholic cirrhosis.
94/gamma globulin very high in?
Malignancy, autoimmune disease, Multible myeloma
95/prolonged PT
VII, X,V, and II and fibrinogen(I). Extrinsic.
96/prolobged PTT or APTT?
VIII, IX, XI, and XII. Intrinsic.
97/corn meal agar +tween 80?
Chlamydospore of candida albicans.
98/diagnosis of viral Abs.
Immunoflurescence, heamagglutination or ELISA.
99/SIADH?
Hyponatremia and hyperglycemia or hyperproteinemia.
100/bacteria produce phenyle pyrovate?
phenylpyruvic acid from L-phenylalanine using an L-amino acid deaminase from Proteus mirabilis.
101/dimorphic fungi?
Histoplasma capsulatum.Coccidioides immitis.Paracoccidioides brasiliensis.Blastomyces
dermatitidis and candida.
102/lipemic sample?
Cholesterol.
103/cryopercipitte to treat?!
fibrinogen, von Willebrand factor, factor VIII, factor XIII and fibronectin.
104/MIC minimum inhibitory concentration?
Lowest concentration of Antimicrobial that inhibit bacterial growth.
105/MBC minmum bactericidal concentration?
Lowest concentration of antimicrobial that kill the bacteria.
106/most hormone secretd from?
Anterior puituitary gland.
107/lipemic sample affect result of?
Spectrophotometer.
108/sample rejection ?
Insuffeicent in quantity.
109/green vaginal discharge ?
T.vaginalis.
110/Antigene stimulate B cell without T cell?
Thymus-independent antigen. T independent Ag.
111/Ag stimulate both T and b cell?!
al
T dependent Antigen.
112/alpha HCG marker for ?
Testicular cancer.
ya
113/beta HCG?
pregnancy.
114/salmonella and shigella transmission?
Faecorally.
115/T cell costimulatory molecules?
ze
CD 28 and CD4, CD8.
116/CD 28 link to ?
B7-1 (CD80).
117/measure color of reaction between two substances?
ed
Spwctrophotometer.
118/Platelets read 228,000 next day read 85 ?
Put the sample in Na citrate
119/.best description of urination ?
Reabsorption, fitration and secretion. All
120/type of RNA take aminoacid to the site of protein synthesis?
t RNA.
121/Tuirbidtimetry?
Transmitted light.
122/thrombocytopenia caused by?
Megalblastic anemia.
123/hormone responsible of water reabsorption and hemostasis?
Vasopressin ADH same.
124/increase in beta gamma bridge ?
Cirrhosis.
125/whip worm.
Trichuris trichura.
126/metabolic alkalosis?
Sever vomiting.
127/clinitest for?
Glucose.
128/Women Rh negative baby Rh positive?
Give her Anti -D.
129/enzyme of digestion in intestine?
Pancreatic amaylase.
130/Megakaryocyte reduced in ?
ITP
Idiopathic thrombocytopenic purpura
131/meningitis associated with contaminated food.
Lesteri monocytogene.
132/when u apply the toriquet for more than recommended time what happen for blood.
Come more concentrated (hemconcentration)
134/for determination of parathyroid hormone PTH?
Hypercalcemia?
135/jaffe reaction used for ?
Creatinine.
136/protein in urine by heat at 45 C and disappear when boiling?
Bence jonse protein.
137/best statement describe yeast?
Round and ovale.
al
138/Hematuria caused by?
S.hematobium.
139/cell increase in viral infection ?
ya
Lymphocyte.
140/water pass through pores?
Ultrafiltration.
141/patient with high direct bilirubin and no urobilinogen?
Obstructive bile.
ze
142/cushing syndrome?
High cirtisol.
143/malaria diagnosed by?!
Peripheral blood.
ed
144/known amount of analayte?
STD standards.
145/enzyme to diagnose pancreatitis?
Lipase.
146/first coby cycle of PCR?
4 cobies.
147/organism with lab worker people?
Salmonella.
148/DNA triple bond ?
C&G
DOUBLE A&T.
149/insulin secret from ?
Pancreas. (beta cell) alpha (glucagon)
150/cause of hepatic comma ?
Ammonia.
151/Recent infection Ab?
IgM
152/tropic hormone secreted from?
Anterior puituitary.
153/normocytic normochromicanemia?
Aplastic anemia.
154/ pateint permanent deferral from donation
HCV
155/causative agent of infectious mononucleosis?
EBV.
156/protein carry the thyroid hormone?
Thyroglouline and thyroxin binding globulin TBG.
157/gasteroctomy?
Low vitamin B 12.
158/blood group AB what Ag found?
A and B Ags.
159/hypernatremia?
Conns disease.
160/AMA,ANA,ASMA?
Autoimmunit.
161/bombay hh?
L-fucose sugar absent not added.
162/hypocalcemia ?
Ca less than 7.5.
163/in DNA double helix?
al
5 to 3.
164/hyposegmented neutrophip found in?
Pelger huet anomaly.
ya
165/triglyceride transport in the blood by?
Chylomicron.
166/antibiotic inhibit the protein synthesis?
Aminoglycosides.
167/patient treatment with warfarin monitoring by?
ze
INR.
168/VWF can be found in?
Cryoprecipitate.
169/ammonia transport to liver in form of?
ed
Alanine.
170/in early fasting person the glucose taken from?
Hepatic(liver) glycogenolysis.
171/blood clotting in EDTA?
Use Na citrate.
172/hemophilus required factor X(hemin) and V(NAD).
Influanzae.
173/CO2 require to grow of?
S.penumonia.
174/bacteria cause meningitis in infants?
175/Mannose binding lectin?
Innnate immunity and bind to lectin of microorganisms.
176/creatinine clearance do for?
Glomerular filtration rate.(GFR).
177/antibiotic link to K of cell membrane?
Valinmycin.
178/MCV elevate in all except?
IDA.
179/mycoplasma resistance to ?
Penicillin (no cell wall).
180/viral transmitted by contaminated food and hand?
HAV.
181/hypersensitivity ?
IgE.
182/selective media.
XLD.
183/agar to identify lactose fermenter?
MaccKonky.
184/gene consists of promoter exon and entron.
185/bacillus dysentry caused by?
Shegilla.
186/end waste of creatin?
Creatinine.
187/antibiotic inhibit RNA synthesis?
Rifampicin.
188/baby with HDN need blood?
Trated with less than 7 days Rh negative blood group.
189/ASO negative?
No agglutination.
190/acetyl Co A in glucose bind to?
al
Oxaloacetate.
191/blood acidosis?
Low PH.
ya
191/best sample to diagnose n.gonorrhae in female?
Cervix.
192/blood clotted in EDTA?
Use Na citrate.
193/SAGM store blood for?
ze
42 days.
194/increase in gamma?
Liver cirrhosi.
195/bile soubility test for diffrentaion of ?
ed
S.pneumonia from other streptococci.
196/enzyme of alcoholic patient?
GGT.
197/false negatuve?
Sensitivity.
198/leukemia of 40-60.
CML.
199/temperatures of CSF delying?
35 to 37 C.
200/technique affected by lipemic sample?
Spectrophotometer.
Cases

1/Women has vaginal normal flora?

Candida albicans, gardenella, ureaplasma,mobiluncus, mycoplasma.

Candida albicans

2/patient with UTI, dysuria, we need to test the glomerular filtration rate, what type of sample
we take?

A.first morning.

B.fasting?

C.24 hour sample


al
3/patient came to emergency room, the following test:

High (ALT,AST,lipase and amylase) all high? What the suspected disease?
ya
A..acute pancreatitis

B.hepatitis.
ze
C.nephritis.

4/patient with diarrhea and vomiting , runny nose? What the causative agent?

A.HCV.
ed

B.HBV.

C.rota virus

5/high reticulocyte count(reticulocytosis)?

Hemolytic anemia and hemorrhage and blood loss:( thalassemia. Sickle)

reticulocytes

6/patient with urination dysuria, urine turbid, hematuria microscopic examination shows oval egg
with terminal spine, what the other sample if parasite not present in the urine?

A.24 hour sample.

B.catheter sample.99
C.bladder aspiration (and bladder biopsy).

D.Prostatic biopsy.

egg(terminal spine)

7/patient came to emergency room , CBC shows

HbA low.

HbF high.

HbS high.
al

What the type of anemia?

A/ sickle cell anemia.


ya

B/thalassemia.

C/iron deficiency anemia.


ze
8/atient tests shows:

ALP high
ed
AST high, and leucin crystal.

What type of jaundice?

A. Obstructive jaundice
B. Hemolytic jaundice
C. Hemolytic / obstructive.

9/case : the organism grow in charchol media?

Legionella pneumophilia.

10/case : peptic ulcer? The organism?

Helicobacter pylori(H.pylori).

11/patient with high direct bilirubin but no urobilinogen in urine?

A/hepatocellular disease.

B.obstuctive bile.(posthepatic).

c.infection liver.
12/organism associated with worker people ?

Salmonella.

al
ya

13/ D
ze
14/patient came to emergency room tests : ALT high

AST high and yellow mucous membrane, diagnosis?


ed
Hepatic jaundice.

15/women with UTI urination dysuria and vaginal discharge we need urine for culture and
sensitivity, what type of sample?

A/24 hours.

B/fasting.

C/clean catch .

D/first morning.

16/patient with high grade fever?

Salmonella typhi.

17/patient came to emergency room lab tests :

Total bilirubin high

Direct bilirubin high

ALT high
AST high

ALP high

GGT slight high? Suspected diagnosis?

Chronic hepatitis.

18/patient : ANA positive .

dsDNA Abs positive ?

systemic lupus erythromatous (SLE).

19/patient came to hospital with vomiting and diarrhea, eat egg and chicken ? suspected
oraganism caused disease?

Salmonella.
al
20/case then the organism cultured on loffler serum media?

c.diphtheriae.
ya

21/patient came to hospital with joint swelling and pain urinalysis shows +++ uric acid crystal?

Suspected disease?
ze
Gout.

22/case female with green vaginal discharges? Suspected organism?


ed
Trichomonas vaginalis.

23/patient came for donation then test : hepatitis?

Permanent deferral.

24/patient with high level of insulin and very low glucose?

Insulinoma.

25/patient with giardial infectio , what the cause of steatorrhae?

Malabsorption, low lipase.

Immune response????

26/case then : this organism cause relapsing fever?

A. Borrelia recurrentis.
B. Borrelia burgdorfer

27/case HIV patient have meningitis the make CSF staining with Indian ink? Appear encapsulated
organism? Suspected organism?
Cryptococcus neoformans.

28/7 years old with sorethroat?

Betahemolytic and bacitracin sensitive? Suspected organism?

Group A streptococci(streptococcus pyogenes)

29/case : patient blood group A2 need blood ? what type of blood u give?

A2 or O ‫ا‬

30/ ‫ا‬ fridge

Check the reagent , temperature indicator.

31/ ‫ا‬

Activate alarm.
al

32/child from Jazan with fever , found banana , crescent like parasite? Suspected ?
ya
A.malaria (p.falciparum).

B.trypanosoma.

33/high gamma and beta bridge (beta –gamma bridge)?


ze

Alcoholic cirrhosis.
ed

34/baby with HDN need blood?

Fresh blood 7 days O negative.

35/patient with high bence jonse protein?

Multible myeloma(MM).

36/case then : high HbF?

Thalassemia.
37/abdominal pain and diarrhea then pink worm earth worm like?

Ascaris lumbericoides.

/38man took antibiotics sufferd from diarrhea?

C.difficle.

/39female with UTI and discharges what the other sample if urine negative?!

Vaginal dischrges.

/40patient came to hospital car accident,

All enzyme elvated AST, CKMB,CK

Possibly diagnosis?

Myokardial infarction.
al

/41confirmatiry test for pin worm entrobius vermicularis.


ya
Scotch tape test.

/42women in hospital low Hb, HbF20?

Major thalassemia.
ze

/43patient get cat bite, ? Gram -ve catalase +ve oxidase+ve

Pasturella maltuscida
ed
/44pateint came to hospital UTI

Gram -ve.

Indole positive

Motile

E.coli.

/45childe with sore throt and enlarge lymph nodes..microscopically shows bacteria like chianese
letters ?

Corynebacterium diphtheriae.

/46patient with otitis media ..

Microscope

Gram positive in cluster

eCoagulase positive
Catalase positive

Staphylococcus aureus.

/47case of elevated ALP

And Ca?

Hyperparathyroidism.

48/ high Ca , ALP and AST?

Cancer pancreatitis.

49pateint with UTI

Catalse positive

Gram positive.
al

Coagulase negative.
ya
Staphylococcus saprophyticus.

/50pateint have typhoid and fever? Gram negative bacilli

Salmonella typhi.
ze

/51monitor the DM doctor askM

2hour urine sample.


ed
/52case the bacteria cause spreading growth on blood agar?

Proteus.

/53childe drink ethanol , best enzyme to test to know there is damage of liver?

GGT.

/54blood in urine then ultrasound is normal ?

S.hematobium.

/55fasting patient do glucose test after took dose of meal, then measure glucose after half hour
and 2 hour , what the name of test?

GTT glucose tolerance test.

56/pateint with urine sample , +++ RBCs hematuria, high eosinophils and prostititis, suspected
organnism?

S.hematobium.

57/pateint case; gram positive bacteria, catalase positive, coagulase negative.?


A.write the report.

B.write s.saprophyticus.

C.incubate the test over night coagulase tube method.

/58baby all WBCs high 5 month?

Normal at this age.

/59patient suffer from chest pain best test is?

Troponin.

/60case of how diagnose acute pancreatitis?

By lipase enzyme.

/61case then mention whip worm?


al

Trichuris trichura.
ya
/62case the blood film sows atypical lymphocytes?

EBV.

/63pateint with cryptococcal meningitis the CDF is collected, what the best confirmatory test?
ze

A.immunflurescence.

B.KOH prepartion.
ed
C.gram stain

D.latex agglutination (for cryptococcal antigen).

/٦٤case...

Egg of E.vermicularis. Pin worm in stool other test?

Schotch tape test.


65/leismania case!!!

66/16 old came to hospital took antibiotics and vit-C for 10 days and drink orange jauce ..

Microscopic examination show

RBCs 20 -30

WBCs 10-20

Calcium oxalate +

What the cause of presence of calcium oxalte crystal in urine?

A..calculi.

B.orange juice.

C.pyelonephritis.
al

67/Pateint , abdominal pain, and diarrhea stool sample collected for staining..
ya
The suitable stain is.?

Cold acid stain(acid fast stain)Kinyoun's Acid-Fast Stain (cold

68/for three days bilirubin test done for patient


ze

First day was 16

Second day was 9


ed
Third day was 15

Why the result for second day was low.?

The sample exposed to the light.

69/blood film show oval shape and shuffner dots.

Plasmodium ovale.

70/One patient came to hospital ..not passing stool for 2 days ..then the microscopic examination
shows 35 eggs

what the possible causes for not passing stool?

duodenum obstruction.

71/35 old male came to saudi arabia from india with abdominal pain ,Constipation, vomiting stool
examination show unfertilized egg ..what other sample?

Doudenal aspiration.
‫ذا‬ ‫من ل من د‪..‬ال مد ‪ ..‬ال ر ل ل من ا دن‬ ‫م ل ال مد‪ ..‬ال ر‬ ‫أ‬ ‫ال مد‬
‫ال د ا م‪..‬‬ ‫ال مل‪ ..‬ن نا من‬
‫أ م‪:‬‬
‫ال زيد‬
‫‪al‬‬
‫‪ya‬‬
‫‪ze‬‬
‫‪ed‬‬
ed
ze
ya
al
SLLE October 2020

1. What is the maximum volume of blood that can be collected from


a 110-lb donor, including samples for processing?
a. 450 mL
b. 500 mL
c. 525 mL
d. 550 mL

2. How often can a blood donor donate whole blood?


a. Every 24 hours
b. Once a month
c. Every 8 weeks
d. Twice a year

3. Which of the following anticoagulant preservatives provides a


storage time of 35 days at 1°C to 6°C for units of whole blood and
prepared RBCs if an additive solution is not added?
a. ACD-A
b. CP2D
c. CPD
d. CPDA-1

4. What are the current storage time and storage temperature for
platelet concentrates and apheresis platelet components?
a. 5 days at 1°C to 6°C
b. 5 days at 24°C to 27°C
c. 5 days at 20°C to 24°C
d. 7 days at 22°C to 24°C

5. RBCs can be frozen for:


a. 12 months.
b. 1 year.
c. 5 years.
d. 10 years
SLLE October 2020

6. Whole blood and RBC units are stored at what temperature?


a. 1°C to 6°C
b. 20°C to 24°C
c. 37°C
d. 24°C to 27°C

7. Additive solutions are approved for storage of red blood cells for
how many days?
a. 21
b. 42
c. 35
d. 7

8. What is the lowest allowable pH for a platelet component at


outdate?
a. 6
b. 5.9
c. 6.8
d. 6.2

9. Frozen and thawed RBCs processed in an open system can be


stored for how many days/hours?
a. 3 days
b. 6 hours
c. 24 hours
d. 15 days

10. Which of the following occurs during storage of red blood


cells?
a. pH decreases
b. 2,3-DPG increases
c. ATP increases
d. plasma K+ decreases
SLLE October 2020

11. Nucleic acid amplification testing is used to test donor blood


for which of the following infectious diseases?
a. Hepatitis C virus
b. Human immunodeficiency virus
c. West Nile virus
d. All of the above

12. Which of the following is the most common cause of


bacterial contamination of platelet products?
a. Entry of skin plugs into the collection bag
b. Environmental contamination during processing
c. Bacteremia in the donor
d. Incorrect storage temperature

13. Which of the following statements best describes mitosis?


a. Genetic material is quadruplicated, equally divided between
four daughter cells
b. Genetic material is duplicated, equally divided between two
daughter cells
c. Genetic material is triplicated, equally divided between
three daughter cells
d. Genetic material is halved, doubled, then equally divided
between two daughter cells

14. When a recessive trait is expressed, it means that:


a. One gene carrying the trait was present.
b. Two genes carrying the trait were present.
c. No gene carrying the trait was present.
d. The trait is present but difficult to observe.
SLLE October 2020

15. PCR technology can be used to:


a. Amplify small amounts of DNA.
b. Isolate intact nuclear RNA.
c. Digest genomic DNA into small fragments.
d. Repair broken pieces of DNA.

16. Which of the following is not involved in the acquired or


adaptive immune response?
a. Phagocytosis
b. Production of antibody or complement
c. Induction of immunologic memory
d. Accelerated immune response upon subsequent exposure
to antigen

17. Which cells are involved in the production of antibodies?


a. Dendritic cells
b. T lymphocytes
c. B lymphocytes
d. Macrophages

18. Which of the following cells is involved in antigen


recognition following phagocytosis?
a. B lymphocytes
b. T lymphocytes
c. Macrophages
d. Granulocytes

19. The role of the macrophage during an antibody response is


to:
a. Make antibody
b. Lyse virus-infected target cells
c. Activate cytotoxic T cells
d. Process antigen and present it
SLLE October 2020

20. Which of the following immunoglobulins is produced in the


primary immune response?
a. IgA
b. IgE
c. IgG
d. IgM

21. Which of the following immunoglobulins is produced in the


secondary immune response?
a. IgA
b. IgE
c. IgG
d. IgM

22. Which of the following MHC classes are found on antigen


presenting cells?
a. Class I
b. Class II
c. Class III
d. Class IV

23. Which of the following MHC classes encodes complement


components?
a. Class I
b. Class II
c. Class III
d. Class IV

24. Which of the following immunoglobulins is most efficient at


binding complement?
a. IgA
b. IgE
c. IgG
d. IgM
SLLE October 2020

25. Which portion of the immunoglobulin molecules contains


complement binding sites?
a. Heavy chain variable region
b. Light chain variable region
c. Heavy chain constant region
d. Light chain constant region

26. Which complement pathway is activated by the formation of


antigen-antibody complexes?
a. Classical
b. Alternative
c. Lectin
d. Retro

27. Which of the following immunoglobulin classes is capable of


crossing the placenta and causing hemolytic disease of the
newborn?
a. IgA
b. IgE
c. IgG
d. IgM

28. Which of the following refers to the effect of an excess


amount of antigen present in a test system?
a. Postzone
b. Prozone
c. Zone of equivalence
d. Endzone
SLLE October 2020

29. Which of the following refers to the presence of an excess


amount of antibody present in a test system?
a. Postzone
b. Prozone
c. Zone of equivalence
d. Endzone

30. Which of the following refers to a state of equilibrium in


antigen-antibody reactions?
a. Postzone
b. Prozone
c. Zone of equivalence
d. Endzone

31. Which one of the following properties of antibodies is NOT


dependent on the structure of the heavy chain constant region?
a. Ability to cross the placenta
b. Isotype (class)
c. Ability to fix complement
d. Affinity for antigen

32. Molecules that promote the update of bacteria for


phagocytosis are:
a. a. Opsonins
b. b. Cytokines
c. c. Haptens
d. d. Isotypes

33. Select the term that describes the unique confirmation of


the antigen that allows recognition by a corresponding antibody:
a. Immunogen
b. Epitope
c. Avidity
d. Clone
SLLE October 2020

34. Which of the following terms refers to the net negative


charge surrounding red blood cells?
a. Dielectric constant
b. Van der Waals forces
c. Hydrogen bonding
d. Zeta potential

35. The polymerase chain reaction (PCR):


a. Is carried out in vivo
b. Is used for peptide synthesis
c. Requires RNA polymerase
d. Is used for the amplification of DNA

36. A principle of the antiglobulin test is:


a. IgG and C3d are required for RBC sensitization.
b. Human globulin is eluted from RBCs during saline washings.
c. Injection of human globulin into an animal engenders
passive immunity.
d. AHG reacts with human globulin molecules bound to RBCs
or free in serum.

37. Polyspecific AHG reagent contains:


a. Anti-IgG.
b. Anti-IgG and anti-IgM.
c. Anti-IgG and anti-C3d.
d. Anti-C3d.

38. Monoclonal anti-C3d is:


a. Derived from one clone of plasma cells.
b. Derived from multiple clones of plasma cells.
c. Derived from immunization of rabbits.
d. Reactive with C3b and C3d.
SLLE October 2020

39. Which of the following is a clinically significant antibody


whose detection has been reported in some instances to be
dependent on anticomplement activity in polyspecific AHG?
a. Anti-Jka
b. Anti-Lea
c. Anti-P1
d. Anti-H

40. After the addition of IgG-coated RBCs (check cells) to a


negative AHG reaction during an antibody screen, a negative
result is observed. Which of the following is a correct
interpretation?
a. The antibody screen is negative.
b. The antibody screen needs to be repeated.
c. The saline washings were adequate.
d. Reactive AHG reagent was added.

41. RBCs must be washed in saline at least three times before


the addition of AHG reagent to:
a. Wash away any hemolyzed cells
b. Remove traces of free serum globulins
c. Neutralize any excess AHG reagent
d. Increase the antibody binding to antigen

42. An in vitro phenomenon associated with a positive IAT is:


a. Maternal antibody coating fetal RBCs
b. Patient antibody coating patient RBCs
c. Recipient antibody coating transfused donor RBCs
d. Identification of alloantibody specificity using a panel of
reagent RBCs
SLLE October 2020

43. False-positive DAT results are most often associated with:


a. Use of refrigerated, clotted blood samples in which
complement components coat RBCs in vitro.
b. A recipient of a recent transfusion manifesting an immune
response to recently transfused RBCs.
c. Presence of heterophile antibodies from administration of
globulin.
d. A positive autocontrol caused by polyagglutination.

44. Polyethylene glycol enhances antigen-antibody reactions


by:
a. Decreasing zeta potential.
b. Concentrating antibody by removing water.
c. Increasing antibody affinity for antigen.
d. Increasing antibody specificity for antigen.

45. Solid-phase antibody screening is based on:


a. Adherence.
b. Agglutination.
c. Hemolysis.
d. Precipitation.

46. A positive DAT may be found in which of the following


situations?
a. A weak D-positive patient
b. A patient with anti-K
c. HDN
d. An incompatible crossmatch

47. What do Coombs control cells consist of


a. Type A-positive cells coated with anti-D
b. Type A-negative cells coated with anti-D
c. Type O-positive cells coated with anti-D
d. Type O-negative cells coated with anti-D
SLLE October 2020

48. Which factor can affect AHG testing, yet is uncontrollable in


the lab?
a. a. Temperature
b. b. Antibody affinity
c. c. Gravitational force in the centrifuge
d. d. Incubation time

49. An ABO type on a patient gives the following reactions:

Patient Cells With Patient Serum With


Anti-A Anti-B A1 cells B cells
4+ 4+ Neg Neg

What is the patient s blood t pe

a. O
b. A
c. B
d. AB

50. The major immunoglobulin class(es) of anti-B in a group A


individual is (are):
a. IgM
b. IgG
c. IgM and IgG
d. IgM and IgA

51. The immunodominant sugar responsible for blood group A


specificity is:
a. L-fucose
b. N-acetyl-D-galactosamine
c. D-galactose
d. Uridine diphosphate-N-acetyl-D-galactose
SLLE October 2020

52. An ABO type on a patient gives the following reactions:

Patient Cells With Patient Serum With


Anti-A Anti-B Anti-A1 A1 cells B cells
4+ 4+ Neg 2+ Neg

53. The reactions above may be seen in a patient who is:


a. A1 with acquired B
b. A2B with anti-A1
c. AB with increased concentrations of protein in the serum
d. AB with an autoantibody

54. Which of the following ABO blood groups contains the least
amount of H substance?
a. A1B
b. A2
c. B
d. O

55. An example of a technical error that can result in an ABO


discrepancy is:
a. Acquired B phenomenon.
b. Missing isoagglutinins.
c. Cell suspension that is too heavy.
d. Acriflavine antibodies.

56. The Rh system was first recognized in a case report of:


a. A hemolytic transfusion reaction.
b. Hemolytic disease of the fetus and newborn.
c. Circulatory overload.
d. Autoimmune hemolytic anemia.
SLLE October 2020

57. What antigen is found in 85% of the white population and is


always significant for transfusion purposes?
a. d
b. c
c. D
d. E

58. Cells carrying a weak-D antigen require the use of what test
to demonstrate its presence?
a. Indirect antiglobulin test
b. Direct antiglobulin test
c. Microplate test
d. Warm autoadsorption test

59. How are Rh antigens inherited?


a. Autosomal recessive alleles
b. Sex-linked genes
c. Codominant alleles
d. X-linked

60. Biochemically speaking, what type of molecules are Rh


antigens?
a. Glycophorins
b. Simple sugars
c. Proteins
d. Lipids

61. Rh antibodies react best at what temperature (°C)?


a. 22
b. 18
c. 15
d. 37
SLLE October 2020

62. Rh antibodies are primarily of which immunoglobulin class?


a. IgA
b. IgM
c. IgG
d. IgD

63. Rh antibodies have been associated with which of the


following clinical conditions?
a. Erythroblastosis fetalis
b. Thrombocytopenia
c. Hemophilia A
d. Stomatocytosis

64. Which of the following methods may be employed to


remove IgG antibodies that are coating a patient s red blood cells
a. Adsorption
b. Elution
c. Neutralization
d. Titration

65. What type of blood should be given in an emergency


transfusion when there is no time to t pe the recipient s sample
a. O Rh0 (D)-negative, whole blood
b. O Rh0 (D)-positive, whole blood
c. O Rh0 (D)-positive, packed cells
d. O Rh0 (D)-negative, packed cells

66. A 26-year-old B Rh0 (D)-negative female patient requires a


transfusion. No B Rh0 (D)-negative donor units are available.
Which should be chosen for transfusion?
a. B Rh0 (D)-positive RBCs
b. O Rh0 (D)-negative RBCs
c. AB Rh0 (D)-negative RBCs
d. A Rh0 (D)-negative RBCs
SLLE October 2020

67. The purpose of the immediate spin crossmatch is to:


a. Ensure survival of transfused RBCs
b. Determine ABO compatibility between donor and recipient
c. Detect cold-reacting unexpected antibodies
d. Meet computer crossmatch requirements

68. Blood donor and recipient samples used in crossmatching


must be stored for a minimum of how many days following
transfusion?
a. 2
b. 5
c. 7
d. 10

69. Which is true regarding compatibility testing for the infant


younger than 4 months old?
a. A DAT is required.
b. A crossmatch is not needed with the infant s blood when
unexpected antibodies are present.
c. Maternal serum cannot be used for antibody detection.
d. To determine the infant s ABO group RBCs must be tested
with reagent anti-A, anti-B, and anti-A,B.

70. A crossmatch is positive at AHG phase with polyspecific


AHG reagent but is negative with monospecific anti-IgG AHG
reagent. This may indicate the antibody:
a. Is a weak anti-D
b. Is a clinically insignificant Lewis antibody
c. Can cause decreased survival of transfused RBCs
d. Is a Duffy antibody
SLLE October 2020

71. The prewarm technique is most useful in investigating


which types of blood bank problems?
a. ABO discrepancies
b. Rh discrepancies
c. Warm antibodies
d. Cold antibodies

72. Which of the following is a method for determining


approximate volume of fetal-maternal bleed?
a. Kleihauer-Betke test
b. Eluate testing
c. Nucleic acid amplification testing
d. Antibody screening

73. Which of the following may not be used as a patient


identifier?
a. Patient s full name
b. Patient s date of birth
c. Patient s medical record number
d. Patient s room number

74. Which of the following is not an enhancement media that


may be used in antibody screening and identification?
a. Albumin
b. Low ionic strength solution (LISS)
c. Normal saline
d. Polyethylene glycol

75. The endpoint of the gel test is detected by:


a. Agglutination
b. Hemolysis
c. Precipitation
d. Attachment of indicator cells
SLLE October 2020

76. The endpoint of the solid-phase immunosorbent assay


(ELISA) is:
a. Agglutination
b. Hemolysis
c. Color change in the substrate
d. Attachment of indicator cells

77. Mixed-field reactions can be observed in:


a. Gel
b. SPRCA
c. Protein A technology
d. Luminex

78. Which of the following information is not required for


whole blood donors?
a. Name
b. Address
c. Occupation
d. Sex
e. Date of birth

79. Which of the following would be cause for deferral?


a. Temperature of 99.2°F
b. Pulse of 90 beats per minute
c. Blood pressure of 110/70 mm Hg
d. Hematocrit level of 37%

80. Which of the following would be cause for permanent


deferral?
a. History of hepatitis after 11th birthday
b. Positive hepatitis C test result
c. Positive HTLV-I antibody
d. Positive anti-HBc test result
e. All of the above
SLLE October 2020

81. Immunization for rubella would result in a temporary


deferral for:
a. 4 weeks
b. 8 weeks
c. 6 months
d. 1 year
e. no deferral required

82. Which of the following donors is acceptable?


a. Donor who had a first-trimester therapeutic abortion 4
weeks ago
b. Donor whose husband is a hemophiliac who regularly
received cryoprecipitate before 1989
c. Donor who was treated for gonorrhea 6 months ago
d. Donor who had a needlestick injury 10 months ago

83. The required storage temperature for frozen RBCs using the
high-glycerol method is:
a. 4°C
b. 20°C
c. 18°C
d. 120°C
e. 65°C

84. How does irradiation affect the shelf-life of red blood cells?
a. Irradiation has no effect on the shelf-life.
b. The expiration date is 28 days from the date of irradiation
or the original outdate, whichever is later.
c. The expiration date is 28 days from the date of irradiation
or the original outdate, whichever is sooner.
d. The expiration date is 25 days from the date of irradiation
or the original outdate, whichever is later.
SLLE October 2020

85. Once thawed, FFP must be transfused within:


a. 4 hours
b. 6 hours
c. 8 hours
d. 12 hours
e. 24 hours

86. AHF concentrates are used to treat:


a. Thrombocytopenia
b. Hemophilia A
c. Hemophilia B
d. von Willebrand disease

87. Prothrombin complex concentrates are used to treat which


of the following?
a. Factor IX deficiency
b. Factor VIII deficiency
c. Factor XII deficiency
d. Factor XIII deficiency

88. Cryoprecipitate that has been pooled must be transfused


within ______ hours.
a. 24
b. 6
c. 4
d. 8

89. The most common anticoagulant used for apheresis


procedures is:
a. Heparin.
b. Sodium fluoride.
c. Warfarin.
d. Citrate
SLLE October 2020

90. Apheresis technology can be used to collect each of the


following components except:
a. Leukocytes.
b. Macrophages.
c. Hematopoietic progenitor cells.
d. Platelets

91. Which Of the following blood type is selected when a


patient cannot wait for ABO-matched RBCs?
a. A
b. B
c. O
d. AB
92. Which patient does not need an irradiated component?
a. Bone marrow transplant recipient
b. Neonate weighing less than 1,200 g
c. Adult receiving an RBC transfusion
d. Adult receiving an RBC transfusion from a blood relative

93. Which type of transplantation requires all cellular blood


components to be irradiated?
a. Bone marrow
b. Heart
c. Liver
d. Kidney

94. Select the appropriate product for a bone marrow


transplant patient with anemia:
a. RBCs
b. Irradiated RBCs
c. Leukoreduced RBCs
d. Washed RBCs
SLLE October 2020

95. Which blood product should be selected for vitamin K


deficiency?
a. Cryoprecipitate
b. Factor VIII
c. Factor IX
d. Plasma

96. Which fluid should be used to dilute RBCs?


a. 0.9% saline
b. 5% dextrose and water
c. Immune globulin
d. Lactated Ringers solution

97. What component is most frequently involved with


transfusion-associated sepsis?
a. Plasma
b. Packed red blood cells
c. Platelets
d. Whole blood

98. Fatal transfusion reactions are mostly caused by?


a. Serologic errors
b. Improper storage of blood
c. Clerical errors
d. Improper handling of the product

99. Pain at infusion site and hypotension are observed with


what type of reaction?
a. Delayed hemolytic transfusion reaction
b. Acute hemolytic transfusion reaction
c. Allergic reaction
d. Febrile nonhemolytic reaction
SLLE October 2020

100. Irradiation of blood is performed to prevent?


a. Febrile nonhemolytic transfusion reaction
b. Delayed hemolytic transfusion reaction
c. Transfusion-associated graft-versus-host disease
d. Transfusion-associated circulatory overload

101. The only presenting sign most often accompanying a


delayed hemolytic transfusion reaction is?
a. Renal failure
b. Unexplained decrease in hemoglobin
c. Active bleeding
d. Hives
102. Which transfusion reaction presents with fever,
maculopapular rash, watery diarrhea, abnormal liver function,
and pancytopenia?
a. Transfusion-associated sepsis
b. Transfusion-related acute lung injury
c. Transfusion-associated graft-versus-host disease
d. Transfusion-associated allergic reaction

103. Nonimmune hemolysis can be caused during transfusion by:


a. Use of small bore size needle.
b. Use of an infusion pump.
c. Improper use of a blood warmer.
d. All of the above

104. Transfusion reactions are classified according to:


a. Signs or symptoms presenting during or after 24 hours.
b. Immune or nonimmune.
c. Infectious or noninfectious.
d. All of the above
SLLE October 2020

105. Graft-versus-host disease (GVHD) is primarily caused by:


a. Neutrophils.
b. T lymphocytes.
c. B lymphocytes.
d. Monocytes.

106. The fecal-oral route is common in transmitting which of


these hepatitis viruses?
a. HAV
b. HBV
c. HDV
d. HCV

107. Which of the following is the most frequently transmitted


virus from mother to fetus?
a. HIV
b. Hepatitis
c. CMV
d. EBV

108. Jaundice due to HAV is seen most often in the:


a. Adolescent.
b. Adult.
c. Child younger than 6 years old.
d. Newborn.

109. HBV is transmitted most frequently:


a. By needle sharing among IV drug users.
b. Through blood transfusions.
c. By unknown methods
d. By sexual activity
SLLE October 2020

110. Which of the following is the most common cause of chronic


hepatitis, cirrhosis, and hepatocellular carcinoma in the United
States?
a. a. HAV
b. b. HBV
c. c. HCV
d. HDV

111. The first retrovirus to be associated with human disease


was:
a. HCV
b. HIV
c. HTLV-I
d. WNV

112. Tests for WNV include all of the following except:


a. ELISA.
b. NAT.
c. Plaque reduction neutralization test.
d. Immunofluorescent antibody assay.

113. Individuals exposed to EBV maintain an asymptomatic


latent infection in:
a. B cells.
b. T cells.
c. All lymphocytes.
d. Monocytes.

114. Screening for HIV is performed using the following


technique:
a. Radio immunoassay
b. WB
c. Immunofluorescent antibody assay
d. NAT
SLLE October 2020

115. Which disease is naturally caused by the bite of a deer tick?


a. Chagas disease
b. Babesiosis
c. Malaria
d. Leishmaniasis

116. HDFN is characterized by:


a. IgM antibody.
b. Nearly always anti-D.
c. Different RBC antigens between mother and father.
d. Antibody titer less than 32.

117. The main difference between the fetus and the newborn is:
a. Bilirubin metabolism.
b. Maternal antibody level.
c. Presence of anemia.
d. Size of RBCs.

118. Kernicterus is caused by the effects of:


a. Anemia.
b. Unconjugated bilirubin.
c. Antibody specificity.
d. Antibody titer.

119. Blood for intrauterine transfusion should be all of the


following except:
a. More than 7 days old.
b. Screened for CMV.
c. Gamma-irradiated.
d. Compatible with maternal serum.
SLLE October 2020

120. RhIG is indicated in which of the following circumstances?


a. Mother D-positive, infant D-positive
b. Mother D-negative, infant D-positive
c. Mother D-positive, infant D-negative
d. Mother D-negative, infant D-negative

121. ABO HDFN is usually mild because:


a. ABO antigens are poorly developed in the fetus.
b. ABO antibodies prevent the disease.
c. ABO antibodies readily cross the placenta.
d. ABO incompatibility is rare.

122. Immune hemolytic anemias may be classified in which of


the following categories?
a. Alloimmune
b. Autoimmune
c. Drug-induced
d. All of the above

123. Which of the following blood groups reacts best with an


anti-H or anti-IH?
a. O
b. B
c. A2
d. A1

124. With cold reactive autoantibodies, the protein coating the


patient s cells and detected in the DAT is
a. C3
b. IgG
c. C4
d. IgM
SLLE October 2020

125. Many warm reactive autoantibodies have a broad specificity


within which of the following blood groups?
a. Kell
b. Duffy
c. Rh
d. Kidd

126. Which of the following drugs has been associated with


complement activation and rapid intravascular hemolysis?
a. Penicillins
b. Quinidine
c. Alpha-methyldopa
d. Cephalosporins
1. Hyphae of this fungi branch at Aspergillus.
45-degree angle?
2. Hyphae of this fungi branch at Mucor
90-degree angle?
3. Simple test used for 1-Oxidative/fermentative media (O/F):
differentiation between S. micrococci (oxidative),
aureus and micrococcus? staphylococci (fermentative).
2-Micrococci are bacitracin sensitive).
4. Sandy spots on cells caused by Arenavirus.
which virus?
5. Patient present with EBV (Epstein Barr virus).
splenomegaly and positive
mono-spot test, which virus is
suspected?
6. Special stain used for amyloid? Congo red stain
7. Patient with swelling of salivary Mumps virus.
and parotid glands, and
al

anorexia what is the suspected


virus?
ya

8. Eye irritation & conjunctivitis Adenovirus.


after swimming, which virus?
9. Negri bodies are eosinophilic Rabies virus.
ze

inclusion bodies, it is caused


by?
ed

10. Organism cultured on blood Proteus spp.


agar (BA) cause swarming
phenomenon is?
11. Capsulated highly mucoid Klebsiella spp.
bacteria?
12. Urease test for Brucella spp, Red color.
what is the color of medium?
13. Special medium for Löwenstein Jensen medium (LJ).
Mycobacterium?
14. Which type of plasmodium Plasmodium falciparum.
cause malignant malaria?
15. Culture media used for fungi? Sabouraud dextrose agar (SDA).
16. Female with yellowish-green Trichomonas vaginalis.
frothy vaginal discharges
caused by?
17. What is the aim of Creatinine Estimate the glomerular filtration rate
clearance test? (GFR).
18. Type of bilirubin increase in Unconjugated (indirect) bilirubin.
hemolytic anemia?
19. Regarding Ag-Ab reaction, what Zone of excess Abs.
is the prozone?
20. The optimum temperature of 56 C°.
complement fixation test (CFT)
is?
21. Glycated Hb (hemoglobin) test HbA1c (average of glucose in 2-3 months).
what?
22. Auer rods found in which type Acute myeloid leukemia (AML).
of leukemia?
23. In Jaffe reaction the creatinine Picric acid in alkaline media.
reacts with?
24. When test is truly negative this Specificity.
is?
25. Function of primer? provides 3’OH end needed to grow
nucleotide chain, Provide free 3’-OH
al

group to which DNA polymerase can add


dNTP. primer binds upstream to the
ya

segment of interest on each strand, called


the
template, to initiate replication, produce
ze

ampIicon, bind to target and initiate


replication.
26. Germ tube test used for? Candida albicans.
ed

27. Wet preparation of stool show Egg of Trichuris trichura.


egg with bipolar plugs (barrel
shape)?
28. Function of microtome? Produce sections (cutting).
29. Using of patient samples in Patient should give informed consent.
research?
30. Fire? RACE & PASS check in google.
31. Review the crystals microscopy.
32. Histopathology, sample gross Pathologists
examination (Grossing) done
by?
33. Review the abnormal RBCs
morphology (schistocytes….),
babesia.
34. Review the morphology of
WBCs (blasts, myeloblasts and
mature) and their inclusions.
35. Review the tissue and
microtomy troubles and
troubleshooting.
36. Review the histopathology
stain.
37. Review the acid/base balance,
blood gases.
38. Review the thyroid and
parathyroid hormones.
39. Function of FSH.
40. Review the gonadotropic
hormones (FSH, LH).
41. Review the LFT.
42. Review the cases (giardia).
43. Review the parasites
microscopy.
al

44. Confirmatory test for VDRL? Fluorescent treponemal antibody


absorption (FTA-ABS).
ya

45. Tumor marker use for? Screening, follow up and monitoring.


46. Review Voges-Proskauer test Klebsiella +ve.
(VP)? Methyl red (MR test). E.coli +ve.
ze

47. Definition of allograft,


xenograft…etc
ed

48. Review the minor blood group.


49. Review the PLTs morphology, Platelets cold agglutinins
aggregation, clumping
pseudothrombocytopenia.
50. Review the transfusion
reactions.
51. Review the grouping of
acquired B Ag and Bombay
group.
52. Differentiate A1 from A2.
53. H substance amount for each
Ag,
54. Review HDN.
55. Review the immune-dominant
sugar for each blood group.
56. Review biochemical tests.
57. Deferral period of donor visiting 1 year
malaria area?
58. Which blood product is the Irradiated blood (RBCs)
most appropriate for
preventing GVHD?
59. Minimum HCT for autologous 33%
donation?
60. If both parents are A blood Blood group A or O
group, what is the possible
offspring blood group?
61. Double dose (homozygous) JK (a+b-)
expression of JK (a+) antigen?
62. Most immunogenic blood group D antigen
antigen after ABO antigens?
63. Chromosome of ABO genes Chromosome 9
location?
64. Chromosome of Rh genes Chromosome 1
location?
al

65. Define the directed donation? Donation for a specific recipient


66. Antibodies cause HDFN? Anti-D, anti-C, anti-E, anti-c, anti-e and
ya

anti-K.
67. One of applications of indirect Antibody screening, crossmatching.
antiglobulin test (IAT), indirect
ze

comb test?
68. Type of antibodies causes Clinically significant antibodies.
ed

transfusion reactions,
significant or insignificant?
69. Turbidity or milky color of urine WBCs (pus cells)
is caused by presence of?
70. Which antibodies are IgG? Anti-e, anti-P, anti-S, anti-s, anti-U, anti-K,
anti-JK(a,b)
71. Normal sperm count? 20 million to 250 million
72. Time of fresh semen sample? 2 hours
73. Urinalysis?
74. Case: gram stain of CSF show N. meningitidis (meningococcal infection)
kidney shape gram negative
bacteria, with low glucose and
elevated protein? Which
bacteria is suspected?
75. Patient with 3 days vomiting, Diabetic ketoacidosis (DKA)
with a distinctive smell of
breathing (like nail polish)?
76. The dark yellow color of bilirubin
amniotic fluid is due to
presence of?
77. Chemical strip of urinalysis Biliary obstruction (post-hepatic jaundice)
shows positive bilirubin and
decreased urobilinogen, what is
the diagnosis?
78. Non pathological condition Dehydration
shows a high blood albumin
level?
79. If the blood tube left for 5 Uric acid
hours what test not affected?
80. Specific enzyme elevated in GGT
biliary obstruction?
81. Patient with chest pain, Troponin T
suspected to have Acute
myocardial infarction, what is
al

the confirmatory test?


82. Special test for evaluation the TSH
ya

congenital hypothyroidism?
83. Tumor marker for colon Carcinoembryonic antigen (CEA)
cancer?
ze

84. Specific enzyme for Alkaline phosphatase (ALP)


hepatobiliary diseases?
ed

85. Suitable blood product for treat Cryoprecipitate


the von Willebrand disease?
86. Specific enzyme for acute Lipase
pancreatitis?
87. A protein electrophoresis, all Monoclonal gammopathy
fractions are normal except
Gamma fraction is elevated,
what is the diagnosis?
88. Bleeding after dental Hemophilia A
extraction, factor assay shows
factor VIII deficiency, prolonged
APTT, normal PT and normal
PLTs count, what is the
diagnosis?
89. Medication act as vitamin K Warfarin
antagonist?
90. Test used for differentiation Mixing studies
between factor deficiency and
factor inhibitor?
91. One patient with high level of Hyponatremia
ADH, what is the common
finding?
92. Test used to differentiate von Bleeding time (vWF low)
Willebrand disease from
hemophilia?
93. Peripheral smear shows Multiple myeloma (MM)
rouleaux formation, caused by?
94. Howell-jolly bodies seen in? Hemolytic anemia, splenectomy,
megaloblastic anemia
95. Type of G6PD anemia? Hemolytic anemia
96. The part in reticulocyte that RNA
stained with new methylene
blue is?
97. Type of WBCs elevated in E. Eosinophils
al

vermicularis infection?
98. CD markers for T lymphocytes? CD3, CD4
ya

99. Mutation in polycythemia vera JAK2 V617F


and essential
thrombocythemia?
ze

100. Mutation in CML? t(9:22)


101. Anticoagulant cause EDTA
ed

pseudothrombocytopenia?
102. Lab finding of TTP? Low PLTs count
103. Chemical movement of Chemotaxis
neutrophil called?
104. Liver macrophage is Kupffer cell
called?
105. Macrophage of which Liver and spleen
organ responsible of
elimination of bacteria?
106. Sever menorrhagia, with Von Willebrand disease
low PLTs and normal PT, APTT?
107. Clinical condition Hashimotos thyroiditis
associated with autoantibodies
against thyroglobulin and
thyroid peroxidase?
108. Protein produced by Interleukins
activated macrophages?
109. Administration of IgE
intravenous penicillin, then
rashes and fever, caused by?
110. Patient with dry mouth Sjogren syndrome
and gritty feeling eyes, high
ANA, anti SS Abs, caused by?
111. High sugar with normal Insulin independent diabetes milletus
insulin?
112. Histopathology, a section Un clean surface cut area
of liver found on a section of
lung?
113. Sample received in Reject the sample
histopathology with request but
no label, what is the proper
action?
114. Hematuria caused by? S. hematobium
115. Routine fixative of 10% buffered formalin
al

histopathology?
116. In gram stain, S. aureus Clusters
ya

arranged in?
117. Acids for urine Hydrochloric acid, boric acid
preservation?
ze

118. Special technique for Dark field microscope


treponema and spirochetes?
ed

119. Infant with meningitis, Listeria monocytogenes


caused by gram positive rods
with tumbling like motility?
120. Test differentiate Production of niacin (nicotinic acid)
M.tuberculosis from other
mycobacteria?
121. Gram positive rods, Listeria monocytogenes
infect fetus and pregnant
women, able to grow in 4 C?
122. Infant with diarrhea Rota virus
caused by a virus that can
prevented by national
immunization program?
123. Biosafety level (BCL) for BCL 3
MERS-C0V?
124. Influenza virus attached Heamagglutinin
to epithelial cells by?
125. Electron microscope Corona virus
shows virus with crown like
projections?
126. Detection of fungi in a 10% KOH
skin sample by?
127. Ringworm caused by? Trichophyton
128. See pic for CLL and CBC
129. Review the 704 M Used for hazards classification
identification system?
130. Suitable mask for MERS- N95
CoV?
131. Access to patient Patient confidentiality
information, results, diagnosis, ,
this state best described by?
132. Hb-electrophoresis Thalassemia minor
shows normal HbA and high
HbA2?
al

133. Treatment of patient Access to care


regardless of age, sex, or
ya

religion is called?
134. This is the only Chylomicron
lipoprotein synthesized and
ze

secreted in intestine?
135. Review cabot inclusion of
ed

RBCs
136. Protocol of positive Inform the infectious disease department
malaria in Saudi Arabia?
137. High lipase and amylase? Pancreatitis
138. Pleural fluid? Centrifugation
139. How differentiate By catalase test
Staphylococci from
streptococci?
140. Stain for reticulocytes Supravital stain (new methylene blue)
count?
141. Glucose in urine (renal Glucagon hormone
glycosuria), normal in blood?
142. Review protein
electrophoresis?
143. If creatinine is 5.9 Call physician
(critical)!!!
144. Review the problems and
solutions for paraffin wax
sectioning?
145. Trend and shift?
146. The specimens of 1 month
histopathology are discarded
after?
147. HIV, BBV, sickle cell Sickle trait
anemia and sickle cell trait, for
donation?
148. Abnormal chromosome Philadelphia chromosome
of CML?
149. Best test for IDA ferritin
150. Cause low ESR? Low fibrinogen
151. PT: 20 sec, APTT: 50 sec, Hypofibrinogenemia
TT: 18 sec??
152. Cause false positive DAT? Clotted sample
153. PT & APTT in Prolonged
al

polycythemia patient?
154. Result in factor X Prolonged PT & APTT
ya

deficiency?
155. Factor not measured by Factor XIII
PT & APTT?
ze

156. Prolonged PT, given IV Obstructive jaundice


vitamin K. the PT corrects to
ed

normal after 24 hrs,


157. A prolonged APTT is Factor IX
corrected with factor VIII
deficient plasma but not with
factor IX deficient plasma, the
deficiency factor is?
158. PT: prolonged, APTT: DIC
prolonged, low PLTs count?
159. Predisposing factor for Adenocarcinoma
DIC?
160. Factor XII deficiency Increased risk of thrombosis
associated with?
161. In factor VIII assay a Factor VIII deficient plasma
patient plasma is mixed with?
162. Protein of primary Alpha 2-antiplasmin
inhibitor of the fibrinolytic
system?
163. Plasminogen deficiency Thrombosis
associated with ?
164. After birth baby develops Neonatal alloimmune thrombocytopenia
petechiae and pupura and
hemorrhagic, PLTs count was
18 x 10 ˆ9/L, diagnosis?
165. Prolonged PT and APTT, Cryoprecipitate
and PLTs: 100 x 10ˆ9/L, and
fibrinogen is 40 mg/dL, what is
the suitable blood product?
166. Normal PT and Von Willebrand disease
prolonged APTT, increased
bleeding time and normal
platelet count, with abnormal
platelet aggregation to
ristocetin?
167. Which ration of 1:9
al

anticoagulant to blood is
correct for coagulation
ya

procedures?
168. Function of vitamin K in Required for carboxylation of glutamate
coagulation? residues of some coagulation factors
ze

169. The APTT is sensitive to Factor X


deficiency of: VII, X, PF3 or
calcium?
ed

170. Test for heparin APTT


monitoring?
171. Normal fasting blood Isolated postprandial hyperglycemia, type
sugar, and high 2-hour 2 diabetes Mellitus, cardiovascular risk.
postprandial glucose?
172. Amber color of urine, Bilirubin
presence of?
173. Photosensitive? Bilirubin
174. Hyaline cast vs waxy Hyaline cast is transparent and composed
cast? of mucoproteins and colorless, waxy cast
is a degeneration of cells contain, yellow
color with grooves on sides (review the
casts, urinalysis).
175. Picture of WBCs cast.
176. Osmolality of urine Na + specific gravity
depend on?
177. Phenylketonuria? Lack of phenylalanine hydroxylase.
178. Detection of ketones in sodium nitroprusside (nitrofen-icyanide,
urine? Acetest is a nitroprusside and glycine
tablet used to detect ketones, enzymatic
method uses beta-hydroxybutyrate
dehydrogenase to
detect the presence of beta-
hydroxybutyric acid.
179. Principle of ELISA? Antigen-Antibody complex
180. Type 4 hypersensitivity? delayed hypersensitivity take 24-72 hours
(days).
181. Define autoimmune An autoimmune disease occurs when an
disease? individual produces antibody or a T
cell response to his/her own antigens.
2. There is a loss of self-tolerance
182. Pic of microfilaria.
183. Pic of trichomonas
al

vaginlais
184. Pic of CML
ya

185. Pic of monocyte


186. Urine formed by? Filtration, reabsorption and secretion.
187. Pic of hemagglutination.
ze

188. Cause hemolytic uremic Enterohemorrhagic


syndrome? E. coli, Shigella dysenteriae
ed

189. Parasite cause acute Giardia


diarrhea?
190. Selective media for TCBS.
vibrio?
191. Anticoagulant in PT? Warfarin
192. Gas gangrene caused by? C. perfringens
193. Meningitis in neonates? S. agalactiae, L. monocytogenes and E.
coli
194. Media contain Thayer martin media, modified Thayer
vancomycin-colistin-nystatin martin media
195. Bile solubility test? Special test for S. pneumoniae, lysed by
bile salts,
196. virus cause intranuclear Cytomegalovirus
giant cytopathic effect as owl
eyes?
197. viral meningitis? Normal glucose with high protein
198. Neurotoxin produced by? C. tetani, C. botulinum, S. aureus, B. cerus.
199. Positive ANA, anti-ds- SLE
DNA?
200. media for fungi? SDA
201. fungal stain? Grocott methamine silver, periodic acid
Schiff, lactophenol cotton blue.
202. Wood lamp test? For hair skin infected by dermatophytes,
M. furfur, tinea versicolor, pityriasis
203. Acromegaly? Test of growth hormone (GH)
204. Niacin (nicotinic acid) Pellagra
and tryptophan deficiency
cause?
205. Cushing syndrome? High cortisol, Hyperaldosteronism, high
ACTH,
206. Double zone of beta C. perfringens.
hemolysis?
207. HbA1C ? Measure sugar for last 3 months.
208. Hypoalbuminemia Acute or chronic inflammation, nephrotic
al

caused by? syndrome, liver cirrhosis, heart failure,


malnutrition, allergy.
ya

209. INR for PT


210. Define MCV? Measure of average volume of RBC
211. Heinz bodies seen in? G6PD anemia
ze

212. Coomb control? Coomb cells (RBCs coated wit IgG)


213. Hypercalcemia? Hyperparathyroidism
ed

214. Cell blocks? Micro biopsies embedded in paraffin wax


for cytology specimens, small tissue
fragments from fine needle aspiration
(FNA),
215. Special stain for PAS
stomach?
216. Hematoxylin stains? The nucleus
217. Wilson disease? Low ceruloplasmin
218. Hemophilia A caused by? Deficiency of factor VIII
219. Life span of RBCs ? 120 days
220. Kleihauer-betke test? Pink RBCs of fetus, colorless RBCs of
mother
221. Antibiotic associated C.difficle
diarrhea?
222. Cat biting (bacteria) Pasteurella multocida
223. Calculate LDL? LDL= total cholesterol - HDL - TG/5
224. Media inhibit & prevent CLED (salt deficient)
the swarming of proteus spp?
225. Bacteria grow at 42 C? Campylobacter, Helicobacter,
Pseudomonas aeruginosa, Borkholderia
226. Bacteria grow at 4 - 43°C Listeria monocytogenes & Yersinia
227. Review RBCs indices vs
microcytic, macrocytic,
hypochromic…etc
228. What is the labeled Antibody labeled with an enzyme
material in ELISA?
229. Melting point of paraffin 56 C ±2
wax?
230. Tissue processing Solution viscosity, agitation, heat, vacume
machine depend on? & pressure
231. The proper thickness of 2 4 mm, 3 5 micron
tissue?
232. Tissue infiltration is done Paraffin wax, resin, agar, gelatin, celloidin
by?
al

233. Frozen section with It's an ice artifacts (water), due to the
holes, clefts and vacuoles? slow freezing of tissue, the solution is:
ya

freeze fast (flash/snap)


234. Exchange transfusion? Patient with ABO antibodies, HDFN,
remove bilirubin and maternal Abs, (O
ze

negative blood group or AB plasma,


negative for CMV, HbS negative, leukocyte
ed

reduced, irradiated, less than 7 10 days)


235. Intrauterine transfusion HDFN Rh-incompatibility (fetal anemia),
(IUT)? using the umbilical vein by high resolution
sonography, to maintain Hb above 10
g/dl, performed after 36 week, O Rh-
negative….same as exchange transfusion
criteria
236. Special stains for fungi Grocott (methanmine) silver (GMS) or
(histopathology) Periodic acid Schiff (PAS)
237. Type of lewis Abs? Cold antibodies
238. Special stain for Giemsa stain H&E
parasites?
239. Reed Sternberg cell Hodgkin lymphoma
found in?
240. Special stain for Best carmine
amoeba?
241. Enzyme responsible of glycolysis?
Pyruvate kinase

242. Differences between Hyaline cast: seen in normal individual &


hyaline and waxy cast? in renal disease (increased), congestive
heart failure, consists of tamm-horsfall
protein, colorless, translucent, low
refractive index
Waxy cast: renal diseases, tubular
inflammation and chronic renal failure,
nephrotic syndrome, consists of
degenerated granular cast and cells,
yellow, high refractive index, contain
cracks
243. Giant intranuclear CMV (cytomegalovirus)
inclusion like owl eyes
cytopathic effect?
al

244. Color of gram-negative Pink or red


bacteria?
ya

245. Test for monitoring of CD4 count


HIV?
246. RT-PCR? Produce DNA from RNA
ze

247. Review the FAB


classification of leukemia
M0…M7?
ed

248. Storage of positive For 10 years for glass slides, 30 years for
cytological specimen? paraffin wax blocks
249. Embedding done by? Filling of tissue with paraffin wax, using a
suitable size of mould or tissue cassettes
250. Tissue cassette not Cut the tissue into a proper size
closed?
251. Croup disease or barking Parainfluenza virus
cough caused by?
252. Enzyme affects with GGT
alcohol?
253. Hormone control the ACTH
adrenal gland?
254. This test differentiates group B from group A streptococci?
CAMP test
255. Bence jonse protein Multiple myeloma
found in?
256. Bacteria cause gas C. perfringens
gangrene?
257. Concentration of Na 3.2%
citrate for coagulation studies?
258. Device used to obtain Centrifuge
sediments?
259. Enzyme of obstructive ALP
jaundice?
260. Indian ink used for? Detect of capsule and cryptococcus
neoformans
261. Cause peptic ulcer, H. pylori
urease positive?
262. Anticoagulant for blood Sodium polyanethol sulphate
culture?
263. Cylindrical larvae? Nematodes (ascaris, hook worm)
al

264. Fungi vs bacteria? Have 80s ribosomes vs bacteria 70s


ribosomes
ya

265. Eukaryotes have no? Mesosomes


266. Parasite cause S. haematobium
hematuria?
ze

267. Cell in parasitic Eosinophil


infection?
ed

268. Cell in bacterial Neutrophil


infection?
269. CPD storage blood for? 14 21 days
270. CPDA for? 35 days
271. Plasma vs serum? Plasma contain fibrinogen
272. HDN caused by? Anti-D, anti-K
273. Shigella on XLD give? Red color
274. Platelets storage? 22- 27 C for 5 days
275. Nucleated RBCs found Hemolytic anemia ad megaloblastic
in? anemia
276. Anemia with Megaloblastic anemia and aplastic anemia
thrombocytopenia?
277. Fishy smell bacteria? Proteus spp
278. Life cycle of plasmodium 72 h
malariae?
279. Visceral leishmaniasis Leishmania donovani
caused by?
280. Patient with UTI, bacteria E. coli
indole positive, lactose
fermenter and motile?
281. G6PD anemia type of Hemolytic anemia
which anemia?
282. Bacteria oxidase Pseudomonas, vibrio, Neisseria and H.
positive? influanzae
283. Maturation of B In bone marrow
lymphocyte?
284. Maturation of T In thymus gland
lymphocyte?
285. Indirect comb test IAT Detect the Abs in vitro
used for?
286. Direct comb test used Detect the sensitized RBCs with Ab in vivo
for?
287. High growth hormone in Gigantism
children?
al

288. High growth hormone in Acromegaly


adult?
ya

289. How to differentiate By Hb-electrophoresis


HbSS from HbAS?
290. Shigella on macConky Colorless
ze

agar?
291. Salmonella on XLD? Pink color with black center
ed

292. Salmonella on Colorless or pale yellow


macConky?
293. Autosomal recessive? Two Copies of defected gene from both
parents should inherited
294. CA 125 marker used for? Ovarian and breast cancer
295. After u rescue and Contain the fire + close the doors
activate the alarm what is the
next step?
296. Disinfectant that kill the Glutaraldehyde
microorganisms and spores?
297. High CA 19-9 and high Pancreatic cancer
lipase and amylase?
298. Spill emergency code? Orange color
299. Person who directly Primary health care provider
contact with patents and
provide care?
300. Type 1 hypersensitivity? IgE
301. Indole test affect by Any media contain dyes, or macckonkey,
which media? EMB and mueller hinton agar
302. Howell-jolly bodies found Splenectomy
in?
303. Normal Hb and normal Normocytic normochromic RBCs
MCV?
304. The suitable time and 121 C for 15 mins
temperature for autoclave that
use 15 Ibs?
305. Patient with car accident Use O negative blood group (emergency)
need 4 units of blood but his
blood group is unknown?
306. Bacteria give a blue- Pseudomonas aeruginosa
green colony, positive fried egg
appearance?
307. Calculate BUN? Urea/2.14
308. Heterozygous Jk JK(a+b+)
al

reaction?
309. Protein S is cofactor for? Protein C
ya

310. Differentiate D-dimer positive


fibrinogenolysis from DIC?
311. Review the CD markers
ze

312. Phenotypes? The morphology, properties of an


organism
ed

313. Genotype? Genetic makeup of organism (DNA) and


hereditary information
314. Patient have vWF Cryoprecipitate
deficiency, the suitable blood
component is?
315. Laboratory reagents Inventory system
system?
316. Pic of leishmania
317. Food poisoning and C. perfringens
bacteria was a gram-positive
bacilli?
318. Method for leukemia Flow cytometry, bone marrow aspiration
diagnosis? and biopsy, cytogenetics (FISH, PCR),
immunohistochemistry, CBC, blood films,
MRI
319. Function of prime (PCR)?
RNA prime to detect genetic expression
320. Gram stain for gram Pink or red
negative bacteria?
321. Urine cast in case of Hemosiderin (hemoglobin) cast
Sickle cell anemia?
322. Familial hyper High LDL
cholesterolemia?
323. Lipid affected by meal? Triglycerides
324. Serology of hepatitis See blood bank ppt
325. Complete the template For DNA: TTCGTA
for AAGCAT For RNA: UUCGUA
326. HDV associated with HBV
which virus?
327. Atypical lymphocytes, EBV
heterophil tests, monospot
test, lymphadenopathy,
nasopharyngeal carcinoma?
328. Cases of traveler related HAV
al

hepatitis?
329. Unfractionated heparin APTT
ya

controlled by?
330. Role of crystal violet in Inhibit the growth of gram-positive
MacCkonkey agar? bacteria
ze

331. Urine with high WBCs? UTI & pyelonephritis


332. Define proficiency tests? External quality control, evaluates a
ed

laboratory testing results by comparing


them to those of similar laboratories.
333. Define elution? Remove of antibodies that is attached to
the surface of a red blood cell, used in
identification of antibodies
334. Review WBCs Pics
335. Missing epitopes of D Partial D antigen
antigen?
336. Iron deficiency anemia Microcytosis (microcytic hypochromic
features? anemia)
337. Philadelphia Chronic myelogenous leukemia (CML)
chromosome seen in?
338. Enzyme specific for Alkaline phosphatase (ALP)
osteoporosis?
339. HPV-18, risk? High risk
340. Fat stain?
Oil red O in histopathology section
341. Normal results of PT, PTT Factor XIII
what factor is deficient?
342. Vibrio cholerae grow on? TCBS media
343. Gram positive, alpha S. pneumoniae
hemolytic optochin sensitive?
344. Drumstick like bacteria? C. tetani
345. Albert stain used for Corynebacterium diphtheriae
which bacteria?
346. High urea, creatinine and High protein
ammonia, due to consuming
of?
347. Urine crystal in renal Calcium salts (Calcium phosphate, calcium
tubular acidosis (RTA)? carbonate)
348. Bacteria cause S. pyogenes
pharyngitis?
349. Female with vaginal Trichomonas vaginalis
discharge, itching, what is the
al

parasite?
350. X-linked diseases? Hemophilia A (factor VIII deficiency),
ya

Hemophilia B (factor IX deficiency),


Note!! Hemophilia C is autosomal
recessive (factor XI deficiency)
ze

351. Predominant WBCs? Neutrophil


352. Predominant C3, C3b
ed

complement?
353. B12 deficiency anemia? Megaloblastic anemia
354. Hypersegmented Megaloblastic anemia
neutrophil seen in?
355. Suitable place for safety In area available for all staff
data sheets?
356. Abduction code color? Pink color (infant)
357. Test for differentiate Hb- electrophoresis
between alpha and beta
thalassemia?
358. Hormone increase Cortisol, glucagon, epinephrine, T3, T4,
glucose? ACTH, growth hormone
359. Beta cell of pancreas Insulin
secretes?
360. Alpha cell of pancreas secrets?
Glucagon
361. Organ produce Kidney
erythropoietin?
362. Hormone responsible of Aldosterone
Na, K, water reabsorption and
excretion of K, H?
363. High liver enzyme, GGT? Cholestasis
364. Polyspecific antihuman IgG and C3 coated RBCs
globulin (AHG) detect?
365. Monospecific AHG Either IgG or C3
detect?
366. Review the morphology
of fungi
367. High ALP? Seen in Paget disease
368. From which DNA strand Positive strand DNA (+ve sense)
we can make RNA?
369. Normal urine pH? Acidic (4 to 8) average (5-6)
370. Positive ANA and positive SLE
al

RF?
371. Polyuria, weight loss, DM
ya

high glucose?
372. Coagulation studies Sodium citrate (Na citrate)
tube?
ze

373. Mendelian genetic for


blood groups
ed

374. Review pics about blood


grouping
375. Value of MCV in High
macrocytic anemia?
376. Beta thalassemia major hbF
which elevated Hb?
377. Abnormal hemoglobin in Hb Bart, Hb H
alpha thalassemia?
378. Sickle solubility test HbS
depend on?
379. Reticulocytosis seen in? Hemolytic anemia, hemorrhage, blood
loss, leukemia, sickle cell anemia,
autimmunehemolytic anemia, treatment
of vit. B12, IDA, folate deficiency
380. Paroxysmal cold Anti-P
hemoglobinuria (PCH) caused
by?
381. Innate immunity cell? Neutrophil, monocyte, basophil,
eosinophil, macrophage, dendritic cell
382. Patient blood group A2 A2 or O
with an anti-A1, the suitable
unit (group) for him?
383. Patient with anti-E need Same Rh phenotype (blood with no E
blood? antigen)
384. Review wiener Rh
terminology
385. Heterozygous of lutherna Lu (a+b+)
(Lu)?
386. Homozygous for lutheran Lu (a+b-)
Lu (a)?
387. Duffy group antigens? Fy (a), Fy (b)
388. Patient with bleeding, Platelets, coagulation factors!!!
give?
389. Organ produce T Bone marrow
al

lymphocyte?
390. Low CSF glucose, high Bacterial, fungal meningitis
ya

lactate and high protein?


391. Urea breath test for? H. pylori
392. The normal highest Sodium (Na 135 145)
ze

electrolyte?
393. Major cation of K
ed

intracellular fluid?
394. High prolactin cause? Infertility
395. Medium for Agarose gel
electrophoresis?
396. Ghost cell? RBCs in hypotonic urine
397. Corticosteroid hormone? Cortisol
398. Positive CEA? Carcinomas of the colorectal (colon),
pancreas, breast and stomach
399. Gram negative H. influenzae
coccobacilli grow on chocolate
agar?
400. Bacteria moves by? Flagellum
401. Parasitology: Sediment
concentration technique we are
looking for parasite in?
402. Pleural fluid sample High protein
exudate characterized by?
403. Most common type of Preanalytical errors
errors?
404. Air bubbles tissue due to poor floatation technique which
artifact? lead to inadequate adherence of tissue to
the slide, solved by using of distilled water
in the bath, also using of alcohol or
detergent to reduce the surface tension.
Placing of cover slip (mounting)
405. Temperature of water 10 C below the melting point of paraffin
bath in the bath? wax
406. Most common type of Hyaline cast
urine casts?
407. C3 cleaved by? C3 convertase
408. High thyroid hormone? Hyperthyroidism
409. While taking a blood Wash the wound
sample from a patient the lab
technician sustained in a
al

needlestick what the proper


action is?
ya

410. Catalase positive, S. aureus


coagulase positive?
411. Methods for detecting FISH, cytogenetic analysis, karyotyping,
ze

the chromosomal DNA microarray, PCR and


translocation? Immunohistochemistry (IHC)
ed

412. Mother O blood group, Codominant


father A blood group, the child
expression?
413. Enriched media? Medium containing natural proteins like
blood, hemoglobin, serum, growth factor
like blood agar and chocolate agar.
Required for fastidious organism like
Neisseria and streptococci
414. Bacillary dysentery Shigella
caused by?
ed
ze
ya
al
ed
ze
ya
al
ed
ze
ya
al
ed
ze
ya
al

You might also like